NR EMT FINAL EXAM 2023-2024 ACTUAL EXAM 200 QUESTIONS AND CORRECT DETAILED ANSWERS WITH RATIONALES (100% CORRECT AND VERIFIED ANSWERS) |ALREADY GRADED A+ (BRAND NEW!!)

National guidelines for EMS care are intended to __.

-provide more consistent delivery of EMS care across the United States

-unify EMS providers under a single medical director

-facilitate a national EMS labor group

-reduce expenses at the local and state levels
Provide more consistent delivery of EMS care across the United States

The EMT is legally obligated to protect a patient’s privacy according to _.

-HIPAA

-CQI

-APGAR

-DCAP
HIPAA

Which of the following descriptions MOST accurately portrays emergency medical services (EMS)?

-a system composed exclusively of emergency medical responders (EMRs) and emergency medical technicians (EMTs) who are responsible for providing care to sick and injured patients

-a team of paramedics and emergency physicians who are responsible for providing emergency care to critically injured patients

-a vast network of advanced life support (ALS) providers who provide definitive emergency care in the prehospital setting

-a team of health care professionals who are responsible for providing emergency care and transportation to the sick and injured
A team of health care professionals who are responsible for providing emergency care and transportation to the sick and injured

What is the EMT’s primary service area?

-the main area in which the EMS agency operates

-the area serviced by the EMT’s medical director

-the location where the EMT provides essential care during a 9-1-1 call

-the only area the EMT is certified to provide patient care
the main area in which the EMS agency operates

Online medical control requires _.

-written protocols approved by medical control

-a physicians presence on the scene of the call

-phone or radio contact with the medical director

-the presence of an advanced-level provider
phone or radio contact with the medical director

As an EMT, you may be authorized to administer aspirin to a patient with chest pain based on:

-medical director approval

-the patient’s condition

-the transport time to the hospita

-an order from a paramedic
medical director approval

What should an EMT do to limit errors in the field?

-deviate from established standards when necessary

-follow the agency’s written protocols

-carry an EMT text for reference at all times

-contact medical direction before initiating any treatments
follow the agency’s written protocols

Which of the following courses requires about 150 hours of training?

-AEMT

-Paramedic

-EMT

-EMR
EMT

The ability to understand others and have them understand you is known as __.

-communication

-scene leadership

-self-confidence

-teamwork and diplomacy
communication

Which of the following statements regarding the NREMT is correct?

-the NREMT is a governmental agency that certifies EMTs

-EMS training standards are regulated by the NREMT

-the NREMT is the exclusive body for EMTs

-the NREMT provides a national standard for EMS testing
the NREMT provides a national standard for EMS testing

The determination that prompt surgical care in the hospital is more important than performing time-consuming procedures in the field on a major trauma patient is based MOSTLY on:

-regional trauma guidelines

-the lead EMT’s decision

-local protocols

-EMS research
EMS research

Which of the following is a specific example of the Mobile Integrated Healthcare (MIH) model?

-paramedics pick up a patient from an acute care clinic

-EMTs respond to a woman who complains of high fever

-EMTs assess a patient whose lung disease is “acting up”

-the paramedic administers a patient’s flu vaccination
the paramedic administers a patient’s flu vaccination

Which of the following interventions is used by all levels if EMS providers?

-needle decompression

-automatic transport ventilators

-multilumen airways

-automated external defibrillator
automated external defibrillator

You are encouraging a patient to go to the hospital by ambulance. You know that a transport will keep you on duty several hours after your regular quitting time. This is an example of __.

-undue hardship

-scope of practice

-patient advocacy

-abuse of authority
patient advocacy

The ability to handle multiple tasks based on their priority is called __.

-time management

-patient empathy

-integrity

-patient advocacy
time management

Prehospital patient care decisions should be based on __.

-the experiences of EMTs

-the opinions of physicians

-an EMS agency’s fiscal resources

-EMS research
EMS research

You are caring for a driver who struck a light pole. She admits to drinking alcohol but orders you not to tell anyone. You should report the information to _.

-law enforcement personnel

-your medical director

-the state motor vehicle department

-the receiving nurse or doctor
the receiving nurse or doctor

The ability to take appropriate action with little direction is known as __.

-time management

-self-motivation

-self-confidence

-diplomacy
self-motivation

Which of the following statements regarding the Americans with Disabilities Act (ADA) of 1990 is correct?

-according to the ADA, EMT candidates with a documented disability are exempt from taking the NREMT exam

-the ADA applies only to individuals with a diagnosed and well-documented physical disability

-the minimum number of hours required to successfully complete an EMT course is less for candidates who are disabled

-the ADA prohibits employers from failing to provide full and equal employment to those who are disabled
the ADA prohibits employers from failing to provide full and equal employment to those who are disabled

American Heart Association protocols are based on __.

-opinions from a core group of cardiologists

-trends in the EMS community

-evidence-based research

-theories ready for field trial
evidence-based research

Which of the following agencies is the source of the EMT Agenda for the Future, the multidisciplinary national review of all aspects of EMS delivery?

-Federal Emergency Management Agency

-National Highway Traffic Safety Administration

-Department of Health and Human Services

-Department of Transportation
National Highway Traffic Safety Administration

Which of the following is an example of secondary prevention?

-providing cervical spine immobilization

-administering vaccines

-teaching safety in schools

-legislating the use of seat belts
providing cervical spine immobilization

Which of the following is the process in which a person, institution, or program is evaluated and recognized as meeting predetermined standards to provide safe and ethical care?

-continuous quality improvement

-licensure

-certification

-medical control
Certification

At what level is an individual who has training in basic life support, including automated external defibrillation, use of a definitive airway adjunct, and assisting patients with certain medications?

-EMT

-Paramedic

-AEMT

-EMR
EMT

What is the primary feature of the Health Insurance Portability and Accountability Act (HIPAA)?

-it enables patients to travel without losing health insurance

-it limits the availability if a patient’s health care information

-it prevents an emergency department from refusing to care for a critically ill patient

-it requires insurance companies to justify denying health care
it limits the availability if a patient’s health care information

Which organization developed and published the first curriculum that served as the guideline for EMT training?

-Department of Transportation (DOT)

-National Academy of Sciences

-Occupational Safety and Health Administration (OSHA)

-Department of Health and Human Services (DHHS)
Department of Transportation (DOT)

Which of the following entities controls the licensure of EMTs?

-the federal government

-the state in which the EMT practices

-community health organizations

-the medical director
the state in which the EMT practices

Which of the following statements about health care in the United States is true?

-health care in the United States is expensive but does not necessarily provide better health outcomes

-the United States has low health care costs as compared to other wealthy nations

-the average life expectancy in the United States is the highest in the world due to advances in health technologies

-the United States spends more money per capita on health care than most other countries, but the level of care is superior
health care in the United States is expensive but does not necessarily provide better health outcomes

Public heath examines the health needs of entire populations. In the United States, public health focuses on education and what else?

-surveillance

-prevention

-treatment

-evaluation
prevention

The reaponsibility of the medical director to ensure the appropriate medical care standards are met by EMTs on each call is called what?

-continuous quality improvement

-medical control

-primary prevention

-quality control
quality control

You are transporting an elderly woman who has possibly experienced a stroke. She is obviously scared but is unable to talk and cannot move the entire right side of her body. In addition to providing the medical care that she needs, you should:

-maintain eye contact and tell her that there is no need for her to be scared

-reassure her that after proper treatment in the hospital, she will regain her speech in time

-tell her that you understand why she is scared and that everything will likely be okay

-acknowledge that she is scared and tell her that you will take good care of her
acknowledge that she is scared and tell her that you will take good care of her

Which of the following is an effective stress management strategy?

-frequently reflecting on troublesome calls

-modestly increasing caffeine consumption

-developing a social network outside of EMS

-requesting overtime to occupy your mind
developing a social network outside of EMS

Which of the following statements regarding the different stages of the grieving process is correct?

-the stages of the grieving process may occur similtaneously

-the grieving process typically begins with severe depression

-bargaining is the most unpleasant stage if the grieving process

-it is rare that people will jump back and forth between stages
the stages of the grieving process may occur similtaneously

Vector-borne transmission of an infectious organism occurs via:

-inanimate objects

-smoke or dust

-direct contact

-animals or insects
animals or insects

Which of the following statements regarding HIV is correct?

-HIV is transmitted exclusively via blood

-HIV is far more contagious than hepatitis B

-HIV is easily transmittable in the EMS field

-there is no vaccine against HIV infection
there is no vaccine against HIV infection

When decontaminating the back of your ambulance after a call, you should:

-spray the contaminated areas and then immediately wipe them dry with a towel

-allow surfaces to air dry unless otherwise indicated in the product directions

-use a bleach and water solution at a 1:2 dilution ratio to thoroughly wipe all surfaces

-clean all surfaces and patient contact areas with a mixture of alcohol and water
allow surfaces to air dry unless otherwise indicated in the product directions

After assessing a patient’s blood glucose level, you accidentally stick yourself with the contaminated lancet. You should:

-discontinue patient care and seek medical attention

-report the incident to your supervisor after the call

-immerse your wound in an alcohol-based solution

-get immunized against hepatitis as soon as possible
report the incident to your supervisor after the call

The simplest yet most effective method of preventing the spread of an infectious disease is to:

-ensure that your immunizations are up-to-date

-undergo an annual physical examination

-wash your hands in between patient contacts

-undergo annual testing for tuberculosis and hepatitis
wash your hands in between patient contacts

At the scene of an automobile crash, a utility pole has been broken and power lines are lying across the car. The patients inside the car are conscious. You should:

-remove the lines with a nonconductive object

-mark off a danger zone around the downed lines

-advise the patients to carefully get out of the car

-proceed with normal extrication procedures
mark off a danger zone around the downed lines

Gloves, a mask, eye protection, and a face shield should be used:

-during routine cleaning of the ambulance

-when performing endotracheal intubation

-whenever you touch nonintact skin

-while handling needles or other straps
when performing endotracheal intubation

Vaccination against the hepatitis A virus is unnecessary if you:

-have been infected with hepatitis in the past

-received the hepatitis B vaccination

-are older than 35 years of age

-have a weak immune system
have been infected with hepatitis in the past

Which of the following would provide the EMT with the BEST cover in a situation involving active gunfire?

-stacked empty barrels

-a large cluster of shrubs

-a concrete barricade

-behind a car door
a concrete barricade

Quid pro quo, a type of sexual harassment, occurs when the harasser:

-requests sexual favors in exchange for something else

-touches another person without his or her consent

-stares at certain parts of another persons’s anatomy

-makes rude remarks about a person’s body parts
requests sexual favors in exchange for something else

Which of the following scenarios MOST accurately depicts a posttraumatic stress disorder (PTSD) reaction?

-a newly certified EMT becomes extremely nauseated and diaphoretic at the scene of an incident involving grotesque injuries

-an EMT with many years of field experience becomes irritable with her coworkers and experiences headaches and insomnia

-an EMT becomes distracted at the scene of a motor vehicle crash involving the same type of car in which a child was previously killed

-an EMT is emotionally exhausted and depressed after a school bus crash involving critical injuries and the death of several children
an EMT becomes distracted at the scene of a motor vehicle crash involving the same type of car in which a child was previously killed

The EMT’s first priority is_____.

-personal safety

-treatment and transport

-rapid response

-empathy for all patients
personal safety

The MOST serious consequence of drug or alcohol abuse among EMS personnel is:

-punitive action and the loss of a job

-low morale and frequently missed shifts

-tension among coworkers and supervisors

-substandard or inappropriate patient care
substandard or inappropriate patient care

General adaptation syndrome is characterized by which of the following phases?

-delayed reaction, alarm response, and physical recovery

-immediate reaction, psychological exhaustion, and recovery

-reaction and resistance, euphoria, and physical exhaustion

-alarm response, reaction and resistance, and recovery
alarm response, reaction and resistance, and recovery

Upon arriving at the scene of a motor vehicle crash, you note that two small cars collided head-on, the occupants are still in their vehicles. The fire department is in the process of stabilizing the vehicles and law enforcement personnel are directing traffic. After parking the ambulance at a safe distance, you and your partner should:

-put on high-visibility safety vests

-quickly begin the triage process

-report to the incident commander

-assist with vehicle stabilization
put on high-visibility safety vests

Common factors that influence how a patient reacts to the stress of an illness or injury include all of the following, EXCEPT:

-fear of medical personnel

-history of chronic disease

-assessment by the EMT

-mental disorders
assessment by the EMT

While providing care to a patient, blood got onto the ambulance stretcher. Because the stretcher was not properly cleaned afterward, a virus was transmitted to another emergency medical technician (EMT) several days later. What route of transmission does this scenario describe?

-direct contact

-indirect contact

-vector-borne transmission

-airborne transmission
indirect contact

Which of the following should you do when lifting something heavy?

-lift using the muscles of your lower back

-bend your legs, not your waist

-keep the weight at arms’s length from your body

-wait for assistance
bend your legs, not your waist

Which of the following involved the spread of infection by animals or insects that carry an organism from one person or place to another?

-bloodborne pathogens

-vector-borne transmission

-airborne transmission

-foodborne transmission
vector-borne transmission

Recently you started noticing that a coworker is disinterested in his work. He has started coming in late and has taken to sitting alone and not socializing with other members of the team. The job has been stressful lately. Call volume has increased and you rarely have any downtime between calls. What is your coworker most likely experiencing?

-delayed stress reaction

-posttraumatic stress disorder

-cumulatuve stress reaction

-acute stress reaction
cumulatuve stress reaction

What blocks the ability of the hemoglobin to transport oxygen to body tissues?

-smoke

-carbon monoxide

-carbon dioxide

-cyanide
carbon monoxide

While treating a patient with a high fever and cough, you accudentally expose yourself to illness. What should you do?

-do not report it but perform self-monitoring to be sure you are not infected

-report it to the hospital staff so they can isolate the patient

-ignore it. the risk of contamination is small

-report it to the infection control officer
report it to the infection control officer

What was the focus of Dr. Elizabeth Kübler-Ross’s On Death and Dying?

-it gave specific guidelines on how to deliver hospice care to critically ill patients

-it became the standard for EMS providers when dealing with violent death

-it introduced the stages of grief
it introduced the stages of grief

What are the three stages of general adaptation syndrome?

-an alarm response, reaction and resistance, and recovery/exhaustion

-acute stress, delayed stress, and cumulative stress

-discovering, defusing, and debriefing

-denial, bargaining, and acceptance
an alarm response, reaction and resistance, and recovery/exhaustion

The tactical use of an impenetrable barrier for protection that law enforcement agents use is called what?

-cover

-concealment

-infection control

-indirect contact
cover

You are responding to an accident scene involving an overturned semi truck carrying diesel fuel. Which reference source will be most helpful in this case?

-CDC standards for infection control

-ANSI safety standards

-OSHA guidelines

-DOT Emergency Response Guidebook
DOT Emergency Response Guidebook

What program, often available for EMS personnel, involves contracted agencies that provide a wide array of mental health, substance abuse, crisis management, and counseling services?

-a Voluntary Protection Program

-an employee assistance program

-an injury and illness prevention program

-a critical incident stress management program
an employee assistance program

Which type of consent is involved when a 39-year-old mentally competent female with a severe headache asks you to take her to the hospital?

-implied

-expressed

-formal

-informed
expressed

You arrive at the scene of a motor vehicle-versus-pedestrian accident. The patient, a 13-year old male, is unconscious and has multiple injuries. As you are treating the child, a law enforcement officer advises you that the child’s parents will be at the scene in approximately 15 minutes. What should you do?

-withhold transport until the parents arrive and guve you consent for treatment

-transport the child immediately and have the parents meet you at the hospital

-begin transport at once and have the parents meet you en route to the hospital

-treat the child at the scene and wait for the parents to arrive and give consent
transport the child immediately and have the parents meet you at the hospital

When performing his or her duties, the EMT is generally expected to:

-consistently exceed the standard of care

-exercise reasonable care and act prudently

-function above his or her scope of practice

-contact medical control on every EMS call
exercise reasonable care and act prudently

You respond to the home of a 59-year-old man who is unconscious; has slow, shallow breathing; and has a weak pulse. The family states that the patient has terminal brain cancer and does not wish to be resuscitated. They further state that there is a DNR order for this patient, but they are unable to locate it. You should:

-begin treatment and contact medical control as needed

-transport the patient without providing any treatment

-honor the patient’s wishes and withhold all treatment

-decide on further actiom once the DNR is produced
begin treatment and contact medical control as needed

Where would you MOST likely find information regarding a patient’s wishes to be an organ donor?

-driver’s license

-social security card

-insurance card

-voter registration card
driver’s license

Which aspect of the Health Insurance Portability and Accountability Act (HIPAA) MOST affects EMS personnel?

-ensuring access to insurance

-controlling insurance costs

-protecting patient privacy

-preventing insurance fraud
protecting patient privacy

In which of the following circumstances can the EMT legally release confidential patient information?

-a police officer requests a copy to place on file

-the patient is competent and signs a release form

-a media representative inquires about the patient

-the family requests a copy for insurance purposes
the patient is competent and signs a release form

During your monthly internal quality improvement (QI) meeting, you review several patient care reports (PCRs) with the staff of your EMS system. You identify the patient’s name, age, and sex, and then discuss the treatment that was provided by the EMTs in the field. By taking this approach to the QI process, you:

-are in violation of HIPAA because you did not remove the PHI from the PCR beforehand

-violated the patient’s privacy because you should have discussed the information only with the EMTs involved

-adequately safeguarded the patient’s PHI because the cases were discussed internally

-acted appropriately but must have each EMT sign a waiver stating that he or she will not discuss the case with others
are in violation of HIPAA because you did not remove the PHI from the PCR beforehand

When is forcible restraint permitted?

-when the patient poses a significant threat to self or others

-only if law enforcement personnel have witnessed threatening behavior

-only if consent to restrain is given by a family member

-anytime that the EMT feels threatened
when the patient poses a significant threat to self or others

Maintaining the chain of evidence at the scene of a crime should include:

-quickly moving any weapons out of the patient’s sight

-placing the patient in a private area until the police arrive

-making brief notes at the scene and then completing them later

-not cutting through holes in clothing that were caused by weapons
not cutting through holes in clothing that were caused by weapons

To help protect patients, EMS agencies are required to have __.

-public forums with their medical director

-an anonymous reporting system

-a privacy officer to answer questions

-online access to patient records
a privacy officer to answer questions

You are called to attend to an elderly patient with an extensive medical history who is now in cardiac arrest. The patient’s family tells you that the patient has a DNR order. There is no paperwork available but the patient does have a MedicAlert bracelet indicating Do Not Resuscitate. You should:

-initiate resuscitation in the absence of paperwork

-confirm the patient’s identity and honor the DNR order

-confirm the patient identity and then confirm the DNR by calling the MedicAlert Foundation number on the bracelet

-transport with minimal care
confirm the patient identity and then confirm the DNR by calling the MedicAlert Foundation number on the bracelet

Which of the following statements about the patient care report is correct?

-patient care cannot be discredited based on poor documentation

-it is difficult to prove actions were performed if they are not included on the report

-EMTs are not liable for any actions that are accurately documented

-incomplete reports are common and accepted in EMs
it is difficult to prove actions were performed if they are not included on the report

In which of the following situations does a legal duty to act clearly exist?

-a call is received 15 minutes prior to shift change

-a bystander encourages a victim who is not breathing

-the EMT witnesses a vehicle crash while off duty

-the EMT hears of a cardiac arrest after his or her shift ends
a call is received 15 minutes prior to shift change

Which of the following components are needed to prove negligence?

-duty to act, abandonment, breach of duty, and causation

-abandonment, breach of duty, damages, and causation

-duty to act, breach of duty, injury/damages, and causation

-breach of duty, injury/damages, adandonment, and causation
duty to act, breach of duty, injury/damages, and causation

An an EMT, the standards of emergency care are often partially based on:

-a consensus among paramedic supervisors

-the wishes of the general public

-locally accepted protocols

-the priorities of the medical director
locally accepted protocols

Which of the following general statements regarding consent is correct?

-expressed consent is valid only if given in writing by a family member

-a patient can consent to transport but can legally refuse to be treated

-all patients older than 18 years can legally refuse treatment or transport

-patients who are intoxicated are generally allowed to refuse treatment
a patient can consent to transport but can legally refuse to be treated

The EMT’s scope of practice within his or her local response area is defined by the:

-EMS supervisor

-local health district

-medical director

-state EMS office
medical director

Putrefication is defined as:

-profound cyanosis to the trunk and face

-blood settling to the lowest point of the body

-decomposition of the body’s tissues

-separation of the torso from the rest of the body
decomposition of the body’s tissues

You and your partner arrive at the scene of a major motor vehicle crash. The driver, a young male is severely entrapped in his car. He has an open head injury and massive facial trauma. He is unresponsive, is not breathing, and does not have a palpable carotid pulse. You should:

-request the fire department to extricate the patient so you can begin CPR

-stop any active bleeding and advuse dispatch to send a paramedic crew

-ventilate the patient for 5 minutes and then stop if there is no response

-have your partner check for a pulse to confirm that the patient is deceased
have your partner check for a pulse to confirm that the patient is deceased

Once your ambulance is dispatched to an emergency, you have an obligation to respond. What is this called?

-scope of practice

-mandatory reporting

-breach of duty

-duty to act
duty to act

You arrive at the scene of a call to find an elderly man complaining of chest pain. You introduce yourself and ask permission to examine him. He nods and extends his arm. Which type of consent is this?

-informed consent

-expressed consent

-mature consent

-implied consent
expressed consent

While evaluating a noncritical patient in the field, you receive a call from dispatch regarding a serious four-car accident. You inform the patient you are treating to have someone take him to his doctor to get checked out before leaving to attend to the more seriously injured people. This is an example of what?

-res iso loquitor

-implied consent

-defamation

-adandonment
adandonment

What is the best way to care for your patient once he tells you he does not want to go to the hospital?

-ask him to sign a refusal form, releasing you from liability

-get the police involved

-order him to go to the hospital against his wishes

-encourage him to call 9-1-1 again if his condition worsens
encourage him to call 9-1-1 again if his condition worsens

The right of a patient to make decisions concerning his or her health is called what?

-decision-making capacity

-competence

-implied consent

-patient autonomy
patient autonomy

An obvious sign of death caused by discoloration of the body from pooling of the blood to the lower parts of the body is called what?

-a presumptive sign

-dependent lividity

-rigor mortis

-putrefaction
dependent lividity

You belong to a small volunteer EMS company and are treating a patient with chest pain, and you feel that the administration of aspirin is indicated. You ask the patient if he is allergic to aspirin and he says no. Shortly after you administer the aspirin, the patient develops signs and symptoms of a severe allergic reaction. Later in the hospital, the doctor advises you that the patient’s medical history indicates that the patient has an allergy to aspirin. The patient later sues you. Which of the following is your best defense strategy?

-statute of limitations

-govenmental immunity

-contributory negligence

-gross negligence
contributory negligence

Which of the following describes a code of conduct that can be defined by society, religion, or a person, affecting character and conscience?

-morality

-ethics

-applied ethics

-bioethics
morality

Which of the following gives surrogates the right to make decisions for patients regarding their health care in the event that the patient is incapacitated and unable to make such decisions?

-advance directive

-emergency doctrine

-durable power of attorney for health care

-do not resuscitate (DNR) order
durable power of attorney for health care

You have responded to a vehicle accident call. Initially, the male patient allows you to splint his broken arm and complete a primary assessment. However, he rescinds comsent during transport and demands to be let out of the ambulance. If you refuse, which of the following might you be charged with?

-battery

-assault

-false imprisonment

-kidnapping
false imprisonment

Which of the following statements regarding a patient refusal is correct?

-documentation of proposed care is unnecessary if the patient refuses treatment

-a patient who comsumed a few beers will likely be able to refuse EMS treatment

-advuce given to a patient who refuses EMS treatment should not be documented

-a mentally competent adult has the legal right to refuse EMS care and transport
a mentally competent adult has the legal right to refuse EMS care and transport

Which of the following is NOT a function of the Federal Communications Commission (FCC)?

-maintaining communications equipment on the ambulance

-licensing base stations and assigning approriate radio call signs

-monitoring all radio traffic and conducting field spot checks

-allocating specific radio frequencies for use by EMS providers
maintaining communications equipment on the ambulance

Effective therapeutic communication skills require _.

-verbal and nonverbal communication techniques

-supervision by advanced life support personnel

-English-speaking patients or family members

-correct use of complex medical terminology
maintaining communications equipment on the ambulance

The official transfer of patient care does not occur until the EMT

-gives a radio report to the receiving medical facility

-notifies the admitting clerk of the patient’s arrival at the hospital

-informs dispatch of the patient’s arrival at the emergency department

-gives an oral report to the emergency room physician or nurse
gives an oral report to the emergency room physician or nurse

Immediately after being dispatched to a residence for an elderly patient with diabetic complications, you should:

-confirm with dispatch that you received the call information

-ask the dispatcher to obtain a medica history from the caller

-ask the dispatcher if the patient is conscious or unconscious

-request that law enforcement secure the scene before you arrive
confirm with dispatch that you received the call information

As you are wheeling your patient through the emergency department doors, you receive another call for a major motor vehicle crash. You should:

-inform the admissions clerk of the situatiom and then respond at once

-respond only after guving a verbal patient report to a nurse or physician

-leave a copy if the run form with a nurse and then respond to the call

-place the patient in a high-visibility area and then respond to the call
respond only after guving a verbal patient report to a nurse or physician

A patient’s refusal of EMS treatment and/or transport must be:

-witnessed by a notary

-an informed refusal

-authorized by a judge

-reported to the police
an informed refusal

You are caring for a 56-year-old male patient complaining of abdominal pain. Your service has recently switched to an electronic PCR system (ePCR). When completing the ePCR, it is important to be aware that:

-a written patient care report will need to be completed on arrival at the hospital

-the ePCR does not contain the same level of information as the written version and your verbal report should be expanded

-the ePCR allows patient information to be transmitted directly to the receiving hospital’s computers

-the ePCR is relatively unsecured and should not contain patient-specific information
the ePCR allows patient information to be transmitted directly to the receiving hospital’s computers

Which type of comminications equipment functions as a radio receiver and searches across several frequencies?

-scanner

-duplex station

-simplex station

-mobile repeater
scanner

During the alert and dispatch phase of EMS communications, the dispatcher’s responsibilities include all of the following, EXCEPT:

-screening and assigning a priority to each call based on local protocols

-selecting and notifying the correct EMS response units and personnel

-coordinating responding EMS units with other public safety personnel

-discouraging the caller from providing care until the EMS unit arrives
discouraging the caller from providing care until the EMS unit arrives

You and your partner arrive at the scene of a middle-aged female in cardiac arrest. Because of the remote geographic location, you are unable to contact medical control. What should you do?

-ask the husband if he wants to continue resuscitation

-make continuous attempts to contact medical control

-follow locally established protocols or standing orders

-perform CPR only and initiate immediate transport
follow locally established protocols or standing orders

During your assessment of a 20-year-old man with a severe headache and nausea, you ask him when his headache began, but he does not answer your question immediately. You should:

-ask him if he frequently experiences severe headaches and nausea

-allow him time to think about the question and respond to it

-tell him that you cannot help him unless he answers your questions

-repeat your question because he probably did not hear you
allow him time to think about the question and respond to it

Which of the following incidents does NOT require a report to be filed with local authorities?

-gunshot wounds

-animal bites

-cardiac arrest

-spousal abuse
cardiac arrest

After receiving online orders from medical control to perform a patient care interventiom, you should:

-perform the intervention as ordered

-confirm the order in your own words

-repeat the order to medical control word for word

-ask the physician to repeat the order
repeat the order to medical control word for word

Which of the following statements regarding a “dedicated line” is correct?

-it is a frequency that is used exclusively by EMTs to communicate with one another in the field

-it is a constantly open line of communication that cannot be accessed by outside users

-it is a designed frequency on a portable radio that provides direct access to medical control

-it is a constantly open line of communication that is under exclusive control of a single user
it is a constantly open line of communication that cannot be accessed by outside users

A 4-year-old boy had an apparent seizure. He is conscious and calm and is sitting on his mother’s lap. His father is sitting in a nearby chair. The child’s mother suddenly begins crying uncontrollably, which causes the child to start crying. You should:

-reassure the child’s mother that seizures in children are very common and that there is nothing to worry about

-give the child a favorite toy or blanket to hold onto and perform your assessment to the best of your ability

-attempt to calm the child’s mother, but avoid separating her from the child because this will increase her anxiety

-ask the father to hold the child so you can assess him while your partner tries to calm the mother
ask the father to hold the child so you can assess him while your partner tries to calm the mother

You are transporting a 54-year-old male in respiratory arrest. An EMR is driving the ambulance as you and your partner are caring for the patient. Which of the following is the MOST logical way of notifying the hospital?

-call the receiving hospital with your cel phone while providing patient care

-wait until you arrive at the hospital and then quickly apprise the staff of the situation

-have the driver contact dispatch and relay the patient information to the hospital

-request that a police officer respond to the hospital to apprise the staff of your arrival
have the driver contact dispatch and relay the patient information to the hospital

You could be sued for _ if your radio report to the hospital describes the patient in a manner that injures his or her reputation.

-libel

-negligence

-assault

-slander
slander

You are dispatched to the residence of an Asian family for a child with a high fever. When you assess the child, you note that he has numerous red marks on his back. The child’s parents explain that these marks represent coining—a traditional Asain healing practice in which hot coins are rubbed on the back. You should:

-advise the child’s parents that this is a harmful practice and is considered a form of child abuse in the United States

-acknowledge and respect this practice as a cultural belief, but advuse the child’s parents that it has no healing power

-advise the emergency department physician that you feel as though the child was intentionally abused by his parents

-document this finding on your patient care report and advise the emergency department staff of what the child’s parents told you
document this finding on your patient care report and advise the emergency department staff of what the child’s parents told you

Which of the following has the weakest transmission signal and, as a result, is LEAST likely to be heard by the party being called?

-mobile repeater station

-land-base repeater station

-hand-held portable radio

-multiplex base station
hand-held portable radio

As you radio the hospital to give a report on your patient, which of the following should you do?

-speak as quickly as possible

-provide as much detail as possible

-remain objective and impartial

-use code words to protect the patient’s identity
remain objective and impartial

After you submit the electronic PCR for a call, you realize that you accidentally documented the wrong vital signs. What should you do?

-say nothing and hope that the error goes unnoticed

-leave the record as it is and seek legal advice

-notify your agency’s information technology department

-follow your agency’s directions for making an amendment
follow your agency’s directions for making an amendment

What is therapeutic communication?

-anything that dampens or obscures the true meaning of a message

-verbal and nonverbal communication techniques that encourage patients to express their feelings and to achieve a positive relationship

-a means by which a patient can efficiently relate levels of pain

-the recorded portion of an EMT’s patient interaction, either written or electronic, that becomes part of the patient’s permanent medical record
verbal and nonverbal communication techniques that encourage patients to express their feelings and to achieve a positive relationship

Radio hardware containing a transmitter and receiver that is located in a fixed location is known as what?

-cell phone

-interoperable communications system

-mobile data terminal (MDT)

-base station
base station

When a person considers his or her own cultural values as more important when interacting with people of a different culture, what is this referred to?

-ethnicentrism

-stereotyping

-rapport

-cultural imposition
ethnicentrism

Encouraging this patient to provide more information about his injury is an example of which therapeutic communication technique?

-clarification

-facilitation

-reflection

-interpretation
facilitation

Which of the following is included in the narrative section in a PCR?

-time the EMS unit was notified

-supportable personal opinions

-refusal of care

-patient name and address
refusal of care

If you discover an error as you are writing your PCR, what should you do?

-erase the erroneous information and insert the correct information

-tell your supervisor about the error

-cover the error with correction fluid and initial it

-draw a single horizontal line through the error, initial it, and write the correct information next to it
draw a single horizontal line through the error, initial it, and write the correct information next to it

What type of mode does an interoperable communications system use?

-voice-over-Internet-protocol

-duplex

-simplex

-trunking
voice-over-Internet-protocol

What should you do if a patient refuses treatment or transport?

-immediately stop patient care

-contact medical control

-find another person to continue medical care

-ask law enforcement to take him into custody
contact medical control

Which of the following suffixes mean “two”?

-“bi-” and “null-“

-“dipl-” and “bi-“

-“primi-” and “dipl-“

-“primi-” and “bi-“
“dipl-” and “bi-“

The topographic term used to describe the location of body parts that are closer toward the midline of the body is:

-medial

-midaxillary

-midclavicular

-lateral
medial

The term “pericardiocentesis” means:

-the removal of fluid from around the heart

  • a surgical opening made in the heart

-narrowing of the arteries supplying the heart

-surgical repair of the sac around the heart
the removal of fluid from around the heart

You are transporting a 67-year-old female patient to the hospital for investigation of abdominal pain. The patient care record indicates that the patient has a history of AAA. Based on this information, the patient care record includes an abbreviation for:

-abdominal aortic aneurysm.

-acute abdominal assessment.

-acute abdominal aneurysm .

-against ambulance advice.
abdominal aortic aneurysm

After applying a tourniquet, the injury from a patient’s leg stops bleeding. This is called:

-hematemesis.
-hemolysis.
-hemiplegia.
-hemostasis.
hemostasis

Which of the following suffixes mean “pertaining to”?

  • “-ic” and “-ology”
  • “-al” and “-ic”
  • “-al” and “-ology”
  • “-ology” and “-oma”
    “-al” and “-ic”

“Gastro” in the word “gastroenteritis” means:

  • tongue.
  • bowel.
  • stomach.
  • intestine.
    stomach

An intoxicated 40-year-old male is found lying face down. How would you document his body’s position?

  • Prone
  • Supine
  • Dorsal
  • Recumbent
    prone

A patient has fractured both femurs. Anatomically, these injuries would be described as being:

  • bilateral.
  • medial.
  • proximal.
  • unilateral
    bilateral

The meaning “around” can have which of the following prefixes?

  • “peri-” and “circum-“
  • “infra-” and “peri-“
  • “epi-” and “sub-“
  • “sub-” and “infra-“
    “peri-” and “circum-“

You are dispatched to a motor vehicle crash involving a motorcycle. Upon your arrival, you find a 17-year-old male patient lying face down on the ground. What is the term for the position in which you found the patient?

  • supine
  • fowler
  • prone
  • recumbent
    prone

What is the root word for pain?

  • asthen
  • angi
  • alges
  • centesis
    alges

What term describes when only one lung is expanding with inhalation?

  • unilateral chest expansion
  • bilateral chest expansion
  • hemilateral chest expansion
  • lateral chest expansion
    unilateral chest expansion

Your partner makes the following symbol in a PCR:
Δ
What does this mean?

  • change
  • less than
  • greater than
  • per
    change

How would you identify a patient who does not have any apparent allergies?

  • NA
  • amb
  • AK
  • NKA
    NKA

What is the medical term for coughing up blood?

  • hematemesis
  • hematuria
  • hemogastritis
  • hemoptysis
    hemoptysis

What should you do when trying to define the term dysuria?

  • begin with the prefix and work forward
  • begin with the root word, move to the suffix, then to the prefix
  • begin with the suffix, move the the prefix, then to the root word
  • begin with the suffix and work backward
    begin with the suffix, move the the prefix, then to the root word

In general, which of the following does a prefix describe?

  • location and intensity
  • condition
  • movement
  • anatomic position
    location and intensity

What is adduction?

  • a directional term describing something farther from the trunk
  • something closer to or on the skin
  • motion of a limb toward the midline
  • motion of a limb away from the midline
    motion of a limb toward the midline

What does asthenia mean?

  • pain
  • weakness
  • paralysis
  • vomiting
    weakness

Capillary sphincter closure during internal or external bleeding is detrimental because:

  • high levels of oxygen remain in the cells and can cause significant damage.
  • waste products are not removed and nutrients are not delivered to the cells.
  • carbon dioxide and other waste can enter the cells, but oxygen cannot.
  • available blood is shunted off to another venule while still carrying oxygen.
    waste products are not removed and nutrients are not delivered to the cells

What is the function of platelets?

  • Transport of oxygen and nutrients
  • Transport of cellular waste materials
  • Defense against invading organisms
  • Initial formation of a blood clot
    Initial formation of a blood clot

Which part of the central nervous system is responsible for coordinating bodily movements such as writing or sewing?

  • Hypothalamus
  • Cerebrum
  • Cerebellum
  • Brain stem
    cerebellum

Which of the following is NOT a function of the skin?

  • Sensory reception
  • Metabolic coordination
  • Pressure and pain perception
  • Temperature regulation
    metabolic coordination

Urine is transported from the kidneys to the urinary bladder via the:

  • urethra.
  • ureters.
  • renal duct.
  • prostate
    ureters

Bile is produced by the liver and concentrated and stored in the:

  • stomach.
  • gallbladder.
  • kidneys.
  • pancreas.
    gallbladder

A patient has a blood pressure of 130/70 mm Hg. The “130” in this measurement represents:

  • atrial contraction.
  • ventricular filling.
  • ventricular contraction.
  • ventricular relaxation.
    ventricular contraction

What happens when blood volume is lost from the body?

  • The arteries contract to increase the blood pressure.
  • Widespread vasodilation causes blood pressure to decrease.
  • The veins dilate to increase systemic perfusion.
  • Arterial blood is diverted to the skin and muscles.
    The arteries contract to increase the blood pressure.

Trauma to the __ lobe of the brain would likely result in visual disturbances.

  • parietal
  • frontal
  • temporal
  • occipital
    occipital

A by-product of involuntary muscle contraction and relaxation is:

  • nitrogen.
  • oxygen.
  • lactic acid.
  • heat.
    heat

A patient has a large accumulation of blood in the sac surrounding the heart. Which type of shock would this condition cause?

  • Hypovolemic
  • Cardiogenic
  • Neurogenic
  • Obstructive
    obstructive

If a patient’s chest barely moves during inhalation, even if the patient’s respiratory rate is normal, you should suspect that:

  • expiratory reserve volume is decreased.
  • inspiratory reserve is increased.
  • minute volume is decreased.
  • overall tidal volume is increased.
    minute volume is decreased

The waste products of aerobic metabolism include:

  • ATP and glucose.
  • carbon dioxide and water.
  • uric acid and nitrogen.
  • glucose and lactic acid.
    carbon dioxide and water

The most superior section of the sternum is called the:

  • angle of Louis.
  • costal arch.
  • manubrium.
  • xiphoid process.
    manubrium

Large amounts of adenosine triphosphate (ATP) are generated when:

  • the cells function without oxygen.
  • carbon dioxide levels in the blood are high.
  • the cells function with adequate oxygen.
  • circulating blood glucose levels fall.
    the cells function with adequate oxygen.

At rest, the normal adult heart rate should not exceed:

  • 100 beats/min.
  • 90 beats/min.
  • 70 beats/min.
  • 80 beats/min.
    100 beats/min

Stimulation of the parasympathetic nervous system would result in:

  • a strong pulse.
  • a slower heart rate.
  • tachycardia.
  • vasoconstriction.
    a slower heart rate

The vocal cords are located in the:

  • oropharynx.
  • nasopharynx.
  • larynx.
  • pharynx.
    larynx

Pathophysiology is the study of the functional changes that occur when the body reacts to a particular:

  • assessment.
  • protocol.
  • disease.
  • medication.
    disease

The brain connects to the spinal cord through a large opening at the base of the skull called the:

  • vertebral foramen.
  • foramen ovale.
  • foramen magnum.
  • spinous foramen.
    foramen magnum

The distal aspect of the tibia forms the:

  • lateral condyle.
  • Achilles tendon.
  • lateral malleolus.
  • medial malleolus.
    medial malleolus

What is the function of the fallopian tubes?

  • To connect the ovaries
  • To transport a mature egg to the uterus
  • To supply blood to the uterine lining
  • To produce progesterone and estrogen
    To transport a mature egg to the uterus

Which of the following systems is responsible for releasing hormones that regulate body activities?

  • Skeletal
  • Reproductive
  • Endocrine
  • Nervous
    endocrine

The __ is a muscular dome that forms the inferior boundary of the thorax, separating the thorax from the abdomen.

  • costovertebral angle
  • costal arch
  • mediastinum
  • diaphragm
    diaphragm

Which organ lies in the lateral and posterior portion of the left upper quadrant of the abdomen?

  • Spleen
  • Stomach
  • Cecum
  • Liver
    spleen

Which of the following structures does NOT contain smooth muscle?

  • Blood vessels
  • Gastrointestinal tract
  • Skeletal system
  • Urinary system
    skeletal system

The bones that constitute the fingers and toes are called:

  • metatarsals
  • metacarpals.
  • carpals.
  • phalanges.
    phalanges

The _ is made up of the maxilla and zygoma, as well as the frontal bone of the cranium.

  • orbit
  • sphenoid
  • mastoid
  • occiput
    orbit

Which of the following is NOT true of the lymphatic system?

  • Its vessels closely parallel the major arteries in the body.
  • It circulates oxygen, nutrients and hormones to the cells.
  • It relies on muscle movement to circulate lymph.
  • It circulates waste products of metabolism away from cells.
    Its vessels closely parallel the major arteries in the body.

The central nervous system is composed of the:

  • spinal cord and sensory nerves.
  • brain and sensory nerves.
  • brain and spinal cord.
  • motor and sensory nerves.
    brain and spinal cord

You arrive at the scene of an accident and observe the patient lying face-up on the ground. As you look at the front side of his body, which plane of the body are you observing?

  • transverse
  • midsagittal
  • sadgittal
  • coronal
    coronal

After opening a patient’s airway and confirming spontaneous respirations, you check for a pulse on his neck just lateral to the trachea. What artery are you palpating?

  • radial
  • femoral
  • brachial
  • carotid
    carotid

Which of the following substances is the main element that the bone cells use to create a hard and resilient structure?

  • magnesium
  • sodium
  • potassium
  • calcium
    calcium

What is the proper name for the cheekbone?

  • maxilla
  • mandible
  • zygoma
  • sphenoid
    zygoma

The elbow is an example of what type of joint?

  • immovable
  • ball-and-socket

-hinge

  • saddle
    hinge

What structure of the brain is responsible for the level of consciousness and maintenance of vital signs?

  • brain stem
  • frontal lobe
  • occipital lobe
  • limbic system
    brain stem

Your patient’s initial pulse rate was 89 beats/min and now it is 116 beats/min. What part of the nervous system is responsible for this increase?

  • Somatic
  • Sympathetic
  • Parasympathetic

-Cerebellum
sympathetic

What type of muscle is involved when you hear your stomach growling?

  • voluntary muscle
  • skeletal muscle
  • smooth muscle
  • cardiac muscle
    smooth muscle

What do you call the amount of air that is moved into or out of the lungs during a single breath?

  • minute volume
  • tidal volume
  • inspiratory reserve volume
  • residual volume
    tidal volume

You are treating a patient who has lost a significant amount of blood. Which type of shock is this patient most likely to experience?

  • septic

-hypovolemic

  • cardiogenic
  • anaphylactic
    hypovolemic

At what age does separation anxiety typically peak in infants and small children?

  • 6 to 8 months
  • 24 to 36 months
  • 18 to 24 months
  • 10 to 18 months
    10 to 18 months

The decline in cardiac function that commonly occurs in late adulthood is MOST often related to:

  • atherosclerosis.
  • hypotension.
  • medication use.
  • kidney failure.
    atherosclerosis

Physical changes that typically occur in early adults include an:

  • increase in fatty tissue, which leads to weight gain.
  • increase in height because of spinal disc expansion.
  • increase in muscle strength and reflexes.
  • increase in respiratory rate due to increased metabolism.
    increase in fatty tissue, which leads to weight gain.

Which of the following statements regarding an infant’s vital signs is correct?

  • An infant’s normal heart rate increases by 10 beats/min each month.
  • An infant’s normal body temperature is typically higher than a preschooler’s.
  • An infant’s heart rate generally ranges between 70 and 110 beats/min.
  • By 6 months of age, an infant’s normal tidal volume is 2 to 4 mL/kg.
    An infant’s normal body temperature is typically higher than a preschooler’s.

Factors that contribute to a decline in the vital capacity of an elderly patient include all of the following, EXCEPT:

  • increased stiffness of the thoracic cage.
  • increased surface area available for air exchange.
  • decreased residual volume.
  • a loss of respiratory muscle mass.
    increased surface area available for air exchange.

An infant or small child’s airway can be occluded if it is overextended or overflexed because:

  • he or she has a long neck, which makes the trachea prone to collapse.
  • the back of the head is flat, which prevents a neutral position.
  • the tongue is proportionately small and can fall back into the throat.
  • the occiput is proportionately large and the trachea is flexible.
    the occiput is proportionately large and the trachea is flexible.

The areas of the infant’s skull that have not yet fused together are called __.

  • cranial valleys
  • fontanelles
  • sutures
  • ventricles
    fontanelles

Children of which age group are considered toddlers?

  • 1-3 years
  • 3-6 years
  • Over 6 years
  • 1 month to 1 year
    1-3 years

The respiratory rate of 30 breaths per minute in an infant is __.

  • too slow
  • too fast
  • too shallow
  • normal
    normal

Which of the following describes the Moro reflex?

  • When the sole of the foot is stroked with a blunt object, the big toe lifts upward and the other toes fan outward.
  • When something touches a neonate’s cheek, he or she instinctively turns his or her head toward the touch.
  • The neonate opens his or her arms wide, spreads his or her fingers, and seems to grasp at something after being startled.
  • An infant’s heart rate decreases secondary to hypoxia because he or she depends heavily on the heart rate to perfuse the body.
    The neonate opens his or her arms wide, spreads his or her fingers, and seems to grasp at something after being startled.

You are assessing a 13-month-old female who is running a fever and has been vomiting. While you are performing your physical examination on this child, you will MOST likely find that she:

  • will readily allow you to separate her from her mother.
  • responds to her name but is fearful of your presence.
  • is unable to track your movements with her eyes.
  • has bulging fontanelles secondary to severe dehydration.
    responds to her name but is fearful of your presence.

Age-related changes in the renal system result in:

  • a significant increase in filtration, which causes the excretion of large amounts of water from the body.
  • the formation of large amounts of urine secondary to an increase in kidney mass of up to 20%.
  • a decreased ability to clear wastes from the body and a decreased ability to conserve fluids when needed.
  • dilation of the blood vessels that supply the nephrons, which allows the kidneys to maintain their function.
    a decreased ability to clear wastes from the body and a decreased ability to conserve fluids when needed.

Atherosclerosis is defined as __.

  • dilation of the arteries
  • the blockage of a coronary artery
  • constriction of the blood vessels
  • the buildup of plaque inside blood vessels
    the blockage of a coronary artery

Which of the following is NOT a common factor that would affect a 75-year-old patient’s vital signs?

  • Increased weight
  • Overall health
  • Medical conditions
  • Medications
    Increased weight

Why does the incidence of diabetes mellitus increase with age?

  • Decreased physical activity, increased weight gain, and decreased insulin production
  • Decreased food intake, decreased weight gain, and decreased blood sugar levels
  • Increased physical activity, increased food intake, and increased insulin production
  • Decreased physical activity, increased weight gain, and decreased blood sugar levels
    Decreased physical activity, increased weight gain, and decreased insulin production

A 16-year-old female complains of vaginal bleeding and abdominal cramping that began several hours ago. During your assessment interview, you should:

  • inquire about the possibility of pregnancy in private, if possible.
  • avoid asking questions that she will feel uncomfortable answering.
  • obtain the majority of your information from one of her parents.
  • recall that patients in this age group prefer not to be treated as adults.
    inquire about the possibility of pregnancy in private, if possible.

According to the terminal drop hypothesis:

  • a person’s physical health begins to decline after the age of 75 years.
  • most late-stage adults retain high brain function until 1 month before death.
  • most elderly patients experience depression after the death of a loved one.
  • mental function is presumed to decline in the 5 years preceding death.
    mental function is presumed to decline in the 5 years preceding death.

A normal systolic blood pressure for a 30-year-old is between:

  • 70 and 140 mm Hg.
  • 80 and 120 mm Hg.
  • 90 and 140 mm Hg.
  • 60 and 120 mm Hg.
    90 and 140 mm Hg.

The risk of bleeding in the skull, which increases with age, is MOST directly related to:

  • blood vessel dilation.
  • a decrease in neurons.
  • shrinkage of the brain.
  • meningeal deterioration.
    shrinkage of the brain.

Which of the following are noticeable characteristics of a 9-month-old infant?

  • Places objects in the mouth, pulls himself or herself up
  • Responds to his or her name, crawls around efficiently
  • Walks without help, becomes frustrated with restrictions
  • Knows his or her name, can walk without any assistance
    Places objects in the mouth, pulls himself or herself up

Why is a neonate’s head more “moldable”?

  • Calcium growth in bones has not yet begun.
  • Fontanelles have not yet fused to form the skull.
  • The brain stem is less rigid.
  • There is more space between the skull and the brain.
    Fontanelles have not yet fused to form the skull.

Congenital abnormalities are the leading cause of death in which age group?

  • Toddlers
  • Adolescents
  • Older adults
  • Neonates
    Neonates

At what stage do vital signs begin to level off within adult ranges?

  • Neonate
  • Preschool
  • School-age
  • Adolescent
    Adolescent

Which of the following conditions is an infant most likely to have?

-Disk compression in the spinal column

  • Loss of passive immunity
  • Airway obstruction
  • Atherosclerosis
    Airway obstruction

Which age group is most likely to suffer from diabetes?

  • Infants
  • Adolescents
  • Middle adults
  • Older adults
    Older adults

Which of the following statements is true about the physiology of older adults?

  • Older adults require more sleep than younger adults.
  • The filtration function of the kidneys decreases and the size of the kidney increases.
  • The metabolic rate in the older brain does not change.
  • Vital capacity is at its highest in older adults.
    The metabolic rate in the older brain does not change.

Which of the following should be expected when you assess a 76-year-old patient’s pupils?

  • Vision deficit
  • Unequal pupils
  • Slower pupillary reaction
  • Fixed, dilated pupils
    Slower pupillary reaction

Which of the following affects vital signs the most in older adults?

  • Patient’s cardiovascular system

-Overall health of the patient

  • Patient’s cardiovascular system
  • Patient’s diet
    Overall health of the patient

Which life stage is apt to be characterized by antisocial behavior and peer pressure?

  • Toddler
  • Adolescent
  • Early adult
  • Middle adult
    Adolescent

Changes in which of the following body systems can result in the most debilitating of age-related illnesses?

  • Sensory
  • Nervous
  • Endocrine
  • Renal
    Nervous

You are called to a scene where a 40-year-old man was clearing trees in a remote area and was struck by a falling tree. When you arrive, he is conscious but the tree is laying across his lower legs. How should you remove the victim once the tree is removed?

  • power lift
  • extremity lift
  • direct lift
  • clothes drag
    Direct lift

When lifting the patient, it is imperative to use which proper body mechanics?

  • extend your arms out as far as possible
  • bend as far forward at the hips as possible
  • keep your feet close together
  • hold your back in an upright position
    Hold your back in an upright position

When lifting a backboard, you should use which of the following?

-power lift

  • lateral lift
  • diamond lift
  • dead lift
    Diamond carry

The scoop stretcher is also known as what?

-basket litter

  • orthopedic stretcher
  • flexible litter
  • ambulance stretcher
    Orthopedic stretcher

To avoid the strain of unnecessary lifting and carrying, which of the following should you use when moving a patient from the ground onto a stretcher?

  • direct carry
  • stair chair
  • draw sheet method
  • extremity lift
    Draw sheet method

You are dispatched to a house where an 80-year-old woman has fallen in an upstairs bathroom and hit her head on the sink. When you arrive she is unconscious. Which is the best equipment to use to move her to the ambulance?

  • stokes litter
  • stretcher
  • backboard
  • stair chair
    Backboard

When carrying a patient down a flight of stairs on a backboard, which person should be at the head end of the backboard?

  • Tina, who is 5’4″ and is the most flexible of the team
  • Jerry, who is 6’2″ and strong
  • Tom, who is 5’10” and has average upper body strength and superior lower body strength
  • Doug, who is 5’8″ and a weightlifter
    Doug, who is 5’8″ and a weightlifter

Which of the following is an example of an urgent move?

  • rapid extrication technique
  • one-person walking assist
  • clothes drag
  • extremity lift
    Rapid extrication technique

You are responding to an accident where a 25-year-old female fell 15 feet while rock climbing. The terrain is steep and there is not much space to work. Which device should you use?

  • vacuum mattress
  • flexible stretcher
  • scoop stretcher
  • Kendrick Extrication Device (KED)
    Flexible stretcher

You are transporting a 50-year-old patient who reports respiratory distress but has no other indications of other injury or illness. What is the best position in which to place this patient?

  • immobilized
  • fowler/semi-fowler
  • recovery
  • supine
    Fowler/semi-fowler

An unrestrained patient is sitting in his car after an automobile crash. He is conscious and alert, has no visible trauma, and is complaining of neck and back pain. Before removing him from his car, you should:

  • slide a scoop stretcher under his buttocks and rotate him laterally.
  • perform a detailed head-to-toe assessment and apply a cervical collar.
  • apply a cervical collar and immobilize him with a vest-style device.
  • maintain manual stabilization of his head and grasp him by the clothes.
    apply a cervical collar and immobilize him with a vest-style device.

Which of the following statements regarding an emergency patient move is correct?

  • It is not possible to perform an emergency move without injuring the patient.
  • An emergency move is performed before the primary assessment and treatment.
  • The patient is dragged against the body’s long axis during an emergency move.
  • The spine must be fully immobilized prior to performing an emergency move.
    An emergency move is performed before the primary assessment and treatment.

Which of the following statements regarding the scoop stretcher is NOT correct?

  • A scoop stretcher will provide adequate immobilization of a patient’s spinal column.
  • The construction of the scoop stretcher prohibits X-rays while the patient is on it.
  • You must fully secure the patient to the scoop stretcher before moving him or her.
  • Both sides of the patient must be accessible for a scoop stretcher to be used.
    A scoop stretcher will provide adequate immobilization of a patient’s spinal column.

In most instances, you should move a patient on a wheeled ambulance stretcher by:

  • slightly lifting the stretcher to prevent unnecessary patient movement.
  • pushing the head of the stretcher while your partner guides the foot.
  • retracting the undercarriage and carrying the stretcher to the ambulance.
  • pushing the foot of the stretcher while your partner guides the head.
    pushing the head of the stretcher while your partner guides the foot.

The extremity lift would NOT be appropriate to use on a patient:

  • with a deformed humerus.
  • without a spinal injury.
  • who complains of nausea.
  • with forearm lacerations.
    with a deformed humerus.

In which of the following situations would a direct ground lift be the MOST appropriate method of moving a patient?

  • A conscious patient complaining of abdominal pain
  • A pedestrian with back pain after being struck by a car
  • A patient who complains of hip pain following a fall
  • An unconscious patient with a possible ischemic stroke
    A conscious patient complaining of abdominal pain

You are attending to a 34-year-old male patient who requires transport to the hospital for assessment of his chronic back pain. The patient weighs over 750 pounds. Your bariatric stretcher has a wider surface area to allow for:

  • increased lifting capacity and patient weight load.
  • increased stability and leverage when lifting with more than two providers.
  • increased patient comfort and dignity.
  • better stability when moving the patient on uneven ground.
    increased patient comfort and dignity.

You are attending to a 26-year-old female who is 34 weeks pregnant with her first child. Your patient has been having lower abdominal pains and cramping for the past two hours. In placing your patient on the stretcher and preparing for transport, you should place her:

  • in the Fowler position.
  • supine with her legs elevated.
  • in a position of comfort.
  • on her left side
    on her left side

Which is the MOST appropriate method to use when moving a patient from his or her bed to a wheeled stretcher?

  • Log roll
  • Extremity carry
  • Draw sheet method
  • Direct carry
    Draw sheet method

When pulling a patient, you should extend your arms no more than __ in front of your torso.

  • 5 to 10 inches
  • 20 to 30 inches
  • 10 to 15 inches
  • 15 to 20 inches
    15 to 20 inches

The direct carry is used to transfer a patient:

  • from a bed to the ambulance stretcher.
  • who cannot be placed on a backboard.
  • with a possible cervical spine injury.
  • with multiple long bone injuries.
    from a bed to the ambulance stretcher.

The _ is both the mechanical weight-bearing base of the spinal column and the fused central posterior section of the pelvic girdle.

  • ischium
  • thorax
  • coccyx
  • sacrum
    sacrum

When carrying a patient up or down stairs, you should avoid:

  • using a wheeled stretcher whenever possible.

B. flexing your body at the knees.

C. the use of a long backboard or scoop stretcher.

D. the use of more than two EMTs.
using a wheeled stretcher whenever possible.

Upon arriving at the scene of a motor vehicle crash, you find a single patient still seated in his car. There are no scene hazards. As you approach the vehicle, you note that the patient is semiconscious and has a large laceration to his forehead. You should:

  • slide a long backboard under his buttocks and lay him sideways on the board.
  • apply a vest-style extrication device before attempting to move the patient.
  • apply a cervical collar and quickly remove the patient with a clothes drag.
  • direct your partner to apply manual in-line support of the patient’s head.
    direct your partner to apply manual in-line support of the patient’s head.

An EMT may injure his or her back, even if it is straight, if the:

  • shoulder is aligned over the pelvis.
  • hands are held close to the legs.
  • force is exerted straight down the spine.
  • back is bent forward at the hips.
    back is bent forward at the hips.

To protect a restrained patient and prevent him from using leverage to break free, the EMT should secure __.

  • both arms at the patient’s sides
  • only the patient’s torso
  • one arm above the head
  • both arms above the head
    one arm above the head

You should not attempt to lift a patient who weighs more than 250 lb with fewer than _ rescuers, regardless of individual strength.

  • five
  • four
  • three
  • six
    four

Situations in which you should use the rapid extrication technique include all of the following, EXCEPT:

  • a patient who blocks access to another seriously injured patient.
  • a patient who needs immediate care that requires a supine position.
  • a patient who can be properly assessed while still in the vehicle.
  • a patient whose condition requires immediate transport to the hospital.
    a patient who can be properly assessed while still in the vehicle.

The MOST serious consequence of a poorly planned or rushed patient move is:

  • causing patient anxiety or fear.
  • confusion among team members.
  • injury to you or your patient.
  • unnecessarily wasting time.
    injury to you or your patient.

It is essential that you __ your equipment to prevent the spread of disease.

  • decontaminate
  • incinerate
  • throw out
  • properly store
    decontaminate

What is the first concern when entering a scene?

  • vital signs
  • physical assessment
  • ABC’s
  • scene safety
    scene safety

You respond to a home where a 25-year-old woman is lying in bed. She is semiconscious but opens her eyes when you speak to her, and is lethargic. How do you rate her on the AVPU scale?

  • awake and alert
  • responsive to pain
  • unresponsive
  • responsive to verbal stimuli
  • responsive to verbal stimuli

Which of the following terms would be used to describe the patient’s pulse rate of 140 beats/min?

  • tachycardia
  • dyspnea
  • bradycardia
  • tachypnea
    tachycardia

What is a pertinent negative?

  • A negative finding that requires advanced life support
  • A negative finding that implies another condition may be present
  • A negative finding that requires further care and/or intervention
  • A negative finding that requires no further care or intervention
    A negative finding that requires no further care or intervention

What is the purpose of the primary assessment?

  • To perform a systematic physical examination
  • To identify and treat changes in a patient’s condition
  • To identify and begin to treat immediate life threats
  • To evaluate the conditions in which you are operating
    To identify and begin to treat immediate life threats

At what point in the patient assessment process do you investigate the chief complaint?

  • History taking
  • Reassessment
  • Secondary assessment
  • Primary assessment
    History taking

When treating a 6-year-old, you note a brassy crowing sound, especially when she breathes in. What is this?

  • Crackle
  • Snoring
  • Wheezing
  • Stridor
    Stridor

In what phase of patient assessment do you determine MOI/NOI?

  • Primary assessment
  • History taking
  • Secondary assessment
  • Scene size-up
    Scene size-up

What is the primary purpose of standard precautions?

  • Determining communication standards
  • Infection prevention
  • Ensuring scene safety
  • Streamlining incident command
    Infection prevention

Which of the following devices would you use to measure a patient’s ventilation, circulation, and metabolism?

  • Pulse oximetry
  • Capnography
  • Blood glucometry
  • Sphygmomanometer
    Capnography

A 29-year-old male with a head injury opens his eyes when you speak to him, is confused as to the time and date, and is able to move all of his extremities on command. His Glasgow Coma Scale (GCS) score is:

  • 12
  • 14
  • 13
  • 10
    13

An elderly patient has fallen and hit her head. Your initial care should focus on:

  • airway, breathing, and circulation.
  • obtaining baseline vital signs.
  • providing immediate transport.
  • gathering medical history data.
    airway, breathing, and circulation

For an adult, the normal resting pulse should be between:

  • 70 and 110 beats/min.
  • 50 and 70 beats/min.
  • 50 and 60 beats/min.
  • 60 and 100 beats/min.
    60 and 100 beats/min.

If a patient develops difficulty breathing after your primary assessment, you should immediately:

  • begin assisting his or her breathing.
  • determine his or her respiratory rate.
  • reevaluate his or her airway status.
  • auscultate his or her breath sounds.
    reevaluate his or her airway status.

After performing a head tilt-chin lift maneuver to open the airway of an unresponsive patient who has a pulse, you should:

  • assess respiratory rate, depth, and regularity.
  • place him or her in the recovery position.
  • provide positive-pressure ventilatory assistance.
  • suction as needed and insert an airway adjunct.
    suction as needed and insert an airway adjunct.

The chief complaint is MOST accurately defined as the:

  • condition that exacerbates an underlying problem.
  • most life-threatening condition that you discover.
  • gross physical signs that you detect on assessment.
  • most serious thing the patient is concerned about.
    most serious thing the patient is concerned about.

When auscultating the blood pressure in a patient’s upper extremity, you should place the head of the stethoscope over the _ artery.

  • femoral
  • radial
  • apical
  • brachial
    brachial

A decrease in blood pressure may indicate:

  • increased blood volume.
  • loss of vascular tone.
  • arterial constriction.
  • forceful cardiac contraction.
    loss of vascular tone.

Which of the following scenarios does NOT involve the presence of any symptoms?

  • A 44-year-old male with abdominal pain and severe dizziness
  • A 49-year-old female with blurred vision and ringing in the ears
  • A 55-year-old male with a severe headache and 2 days of nausea
  • A 61-year-old female who is unconscious with facial cyanosis
    A 61-year-old female who is unconscious with facial cyanosis

Which of the following is the MOST effective method of assessing the quality of air movement in the lungs?

  • Looking for the presence of accessory muscle use
  • Applying a pulse oximeter and monitoring the SpO2
  • Evaluating the patient’s chest for cyanosis
  • Auscultating breath sounds with a stethoscope
    Auscultating breath sounds with a stethoscope

Which of the following questions would you ask a patient to ascertain the “M” in the SAMPLE history?

  • “Have you ever had any major surgeries?”
  • “How long have you had your chest pain?”
  • “How much Tylenol do you take each day?”
  • “When was the last time you ate a meal?”
    “How much Tylenol do you take each day?”

When approaching a 32-year-old male who is complaining of traumatic neck pain, you should:

  • approach him from behind and ask him not to move.
  • assess his mental status by having him move his head.
  • stand behind him and immediately stabilize his head.
  • ensure that the patient can see you approaching him.
    ensure that the patient can see you approaching him.

Treatment and transport priorities at the scene of a mass-casualty incident should be determined after:

  • a physician arrives at the scene.
  • the number of patients is known.
  • all the patients have been triaged.
  • area hospitals have been notified.
    all the patients have been triaged.

Which of the following actions would NOT be performed during the scene size-up?

  • Asking a neighbor to secure the patient’s dog
  • Notifying the dispatcher to send fire personnel
  • Noting the position of a crashed motor vehicle
  • Rapidly assessing a patient’s respiratory status
    Rapidly assessing a patient’s respiratory status

Poor peripheral circulation will cause the skin to appear:

  • flushed
  • ashen
  • pink
  • cyanotic
    ashen

What part of the patient assessment process focuses on obtaining additional information about the patient’s chief complaint and any medical problems he or she may have?

  • Primary assessment
  • History taking
  • General impression
  • Secondary assessment
    History taking

During an EMS call, you should take standard precautions:

  • after it has been determined that the patient is bleeding.
  • before you load the patient into the ambulance.
  • before exiting the ambulance and before actual patient contact.
  • immediately after completion of your primary assessment.
    before exiting the ambulance and before actual patient contact.

A 39-year-old male sustained a stab wound to the groin during an altercation at a bar. As you approach the patient, you note that he is conscious. He is screaming in pain and is attempting to control the bleeding, which is bright red and spurting from his groin area. You should:

  • administer 100% supplemental oxygen.
  • elevate his legs and keep him warm.
  • apply direct pressure to the wound.
  • ensure that his airway is patent.
    apply direct pressure to the wound.

You are assessing a 72-year-old man with abdominal pain. The patient is sitting in a chair; he is conscious, alert, and calm. As you are talking to the patient, your partner discreetly directs your attention to a handgun, which is located on a nearby table. You should:

  • direct your partner to move the gun to a safe area and then advise the patient that his weapon has been secured.
  • document the presence of the weapon, including its specific location, and continue your assessment of the patient.
  • position yourself in between the patient and the gun and ask your partner to request law enforcement assistance.
  • immediately cease all patient care, carefully back out of the residence, and request law enforcement assistance.
    position yourself in between the patient and the gun and ask your partner to request law enforcement assistance.

In patients with deeply pigmented skin, changes in color may be apparent only in certain areas, such as the:

  • back of the neck.
  • lips or oral mucosa.
  • forehead and face.
  • dorsum of the hand.
    lips or oral mucosa.

You answer a call to a restaurant where you find an unresponsive 50-year-old woman laying on the floor. She appears cyanotic and you do not detect chest rise and fall. There are no signs of traumatic injury. What should you do first?

  • apply oxygen with a nonrebreathing mask and assess vital signs
  • prepare for immediate transport to a hospital
  • insert an oral airway and begin ventilations with a BVM
  • check the patient for any medical alert tags
    insert an oral airway and begin ventilations with a BVM

You are assessing ventilations with a BVM and notice the patient’s chest does not rise and fall with each ventilation. What should you do?

  • assume there is a foreign body airway obstruction and immediately begin chest compressions
  • assume there is a foreign body airway obstruction and attempt to ventilate using an alternative method
  • reposition the airway by bringing the head back to a neutral position, then reopen the airway and attempt to ventilate
  • reposition the airway by hyperextending the head to allow for better anatomic position, then attempt to ventilate
    reposition the airway by bringing the head back to a neutral position, then reopen the airway and attempt to ventilate

How can gastric distention be prevented when performing artificial ventilations?

  • provide slow, gentle breaths during artificial ventilation over 3 seconds
  • provide rapid, forceful breaths during artificial ventilations over 3 seconds
  • provid slow, gentle breaths during artificial ventilation over 1 second
  • provide rapid, forceful breaths during artificial ventilation over 1 second
    provide slow, gentle breaths during artificial ventilation over 1 second

Which of the following is contraindicted in a patient who has sustained a head injury?

  • insertion of a nasal airway
  • insertiom of an oral airway
  • jaw-thrust maneuver
  • mouth-to-mask ventilation
    insertion of a nasal airway

While you are performing artificial ventilations on a patient, he vomits. What should you do?

  • stop ventilations and wait for advanced life support
  • continue ventilations with increased force to prevent aspiration
  • roll the patient onto his side to allow for drainage of emesis
  • immediately stop ventilations and begin chest compressions
    roll the patient onto his side to allow for drainage of emesis

What is respiration?

  • the physical act of moving air into and out of lungs
  • the exchange of oxygen and carbon dioxide in the alveoli and tissues
  • the production of energy through nutrient and oxygen combination
  • the process of loading oxygen molecules into hemoglobin molecules in the bloodstream
    the exchange of oxygen and carbon dioxide in the alveoli and tissues

Excellent indicators of respiration include level of consciousness and what else?

  • posturing
  • skin color
  • flaring
  • wheezing or stridor
    skin color

What is the preferred method of ventilating a patient?

  • mouth-to-mask with one-way valve
  • two-person BVM with reservoir and supplemental oxygen
  • one-person BVM with reservoir

-manually triggered ventilation device
mouth-to-mask with one-way valve

What is the main problem with positive-pressure ventilation?

  • cardiac output drops
  • cardiac output

-there is a decrease in intrathoracic pressure

  • hemothorax
    cardiac output drops

Which of the following is an indication of poor air exchange?

  • stridor
  • pneumothorax
  • gastric distention
  • wheezing
    stridor

You are performing mouth-to-mask ventilations with oxygen connected and set at a flow rate of 15 L/min. What percentage of oxygen is your patient receiving?

  • 55%
  • 75%
  • 45%
  • 65%
    55%

What occurs when a patient is breathing very rapidly and shallowly?

  • air is forcefully drawn into the lungs due to the negative pressure created by the rapid respirations
  • the majority of tidal volume reaches the lungs and diffuses across the alveolar-capillary membrane
  • air moves primarily in the anatomic dead space and does not participate in pulmonary gas exchange
  • minute volume increases because of a marked increase in both tidal volume and respiratory
    air moves primarily in the anatomic dead space and does not participate in pulmonary gas exchange

Which of the following structures is NOT found in the upper airway?

  • pharynx
  • larynx

-orotharynx

  • bronchus
    bronchus

The physical act of moving air into and out of the lungs is called:

  • oxygenation
  • diffusion
  • respiration
  • ventilation
    ventilation

How does CPAP improve oxygenation and ventilation in patients with certain respiratory problems?

  • it pushes thick, infected pulmonary secretions into isolated areas of the lung
  • it forces the alveoli open and pushes oxygen across the alveolar membrane
  • it prevents albeolar collapse by pushing air into the lungs during inhalation
  • it decreases intrathoracic pressure, which allows more room for lung expansion
    it forces the alveoli open and pushes oxygen across the alveolar membrane

Irregular respirations characterized by an increasing rate and depth of breathing followed by periods of apnea are called:

  • eupneic respirations
  • agonal respirations
  • Cheyne-Stokes respirations
  • ataxic resporations
    Cheyne-Stokes respirations

Which of the following patients should you place in the recovery position?

  • a 40-year-old conscious fenale with a possible neck injury and regular respirations
  • a 31-year-old semiconscious male with low blood sugar and adequate breathing
  • a 24-year-old unconscious female who overdosed and has a reduced tidal volume
  • a 19-year-old conscious male with a closed head injury and normal respirations
    a 31-year-old semiconscious male with low blood sugar and adequate breathing

Which of the following statements regarding oxygen is correct?

  • Oxygen cylinders must always remain in an upright position.
  • Oxygen is flammable and may explode if under high pressure.
  • Oxygen supports the combustion process and may cause a fire.
  • Oxygen is most safely administered in an enclosed environment.
    Oxygen supports the combustion process and may cause a fire.

The nasopharyngeal airway is MOST beneficial because it:

  • effectively maintains the airway of a patient in cardiopulmonary arrest.
  • can maintain a patent airway in a semiconscious patient with a gag reflex.
  • is generally well tolerated in conscious patients with an intact gag reflex.
  • can effectively stabilize fractured nasal bones if it is inserted properly.
    can maintain a patent airway in a semiconscious patient with a gag reflex.

When testing a mechanical suctioning unit, you should turn on the device, clamp the tubing, and ensure that it generates a vacuum pressure of more than:

  • 00 mm Hg.
  • 400 mm Hg.
  • 200 mm Hg.
  • 300 mm Hg.
    300 mm Hg.

A patient who is suspected of being hypoxic and is breathing adequately should be given supplemental oxygen with a:

  • bag-valve mask.
  • nasal cannula.
  • mouth-to-mask device.
  • nonrebreathing mask.
    nonrebreathing mask.

In which of the following patients would the head tilt-chin lift maneuver be the MOST appropriate method of opening the airway?

  • A 50-year-old male who is unconscious following head trauma
  • A 24-year-old male who is found unconscious at the base of a tree
  • A 45-year-old male who is semiconscious after falling 20 feet
  • A 37-year-old female who is found unconscious in her bed
    A 37-year-old female who is found unconscious in her bed

A 19-year-old female is found unconscious by her roommate. Your primary assessment reveals that her breathing is inadequate. As you insert an oropharyngeal airway, she begins to gag violently. You should:

  • continue to insert the airway as you suction her oropharynx.
  • insert the airway no further but leave it in place as a bite block.
  • remove the airway and be prepared to suction her oropharynx.
  • select a smaller oropharyngeal airway and attempt to insert it.
    remove the airway and be prepared to suction her oropharynx.

Which of the following is the MOST reliable indicator of adequately performed bag-valve mask ventilations in an apneic adult with a pulse?

  • Twenty breaths/min being delivered to the adult
  • Adequate rise of the chest when squeezing the bag
  • Consistently increasing heart rate
  • Decreased compliance when squeezing the bag
    Adequate rise of the chest when squeezing the bag

Which of the following statements regarding normal gas exchange in the lungs is correct?

  • The oxygen content in the alveoli is highest during the exhalation phase.
  • Blood that returns to the lungs from the body has low levels of carbon dioxide.
  • Oxygen and carbon dioxide diffuse across the alveolar walls and capillaries.
  • The actual exchange of oxygen and carbon dioxide occurs in the capillaries.
    Oxygen and carbon dioxide diffuse across the alveolar walls and capillaries.

You are ventilating a patient with a stoma; however, air is escaping from the mouth and nose. To prevent this, you should:

  • ventilate with less pressure.
  • seal the mouth and nose.
  • thrust the jaw forward.
  • thoroughly suction the stoma.
    seal the mouth and nose.

Which of the following statements regarding breathing adequacy is correct?

  • A patient with slow respirations and adequate depth will experience an increase in minute volume.
  • The single most reliable sign of breathing adequacy in the adult is his or her respiratory rate.
  • Patients with a grossly irregular breathing pattern usually do not require assisted ventilation.
  • Patients breathing shallowly may require assisted ventilation despite a normal respiratory rate.
    Patients breathing shallowly may require assisted ventilation despite a normal respiratory rate.

During your assessment of a patient with respiratory distress, you hear wheezing when listening to breath sounds. This indicates:

  • a lower airway obstruction.
  • swelling of the upper airway.
  • secretions in the airway.
  • fluid in the alveoli.
    a lower airway obstruction.

Which of the following patients is breathing adequately?

  • A conscious female with facial cyanosis and rapid, shallow respirations
  • A conscious male with respirations of 18 breaths/min and reduced tidal volume
  • A conscious male with respirations of 19 breaths/min and pink skin
  • An unconscious 52-year-old female with snoring respirations and cool, pale skin
    A conscious male with respirations of 19 breaths/min and pink skin

A man was found unresponsive in his bed at home. There is no evidence of injury and the patient’s medical history is not known. The patient’s face is cyanotic, yet the pulse oximeter reads 98%. Which of the following would MOST likely explain this?

  • Increased body temperature
  • His extremities are cold
  • Severe pulmonary edema
  • Carbon monoxide poisoning
    Carbon monoxide poisoning

Tylenol is an example of what?

  • official name
  • generic name
  • trade name
  • chemical name
    trade name

You are treating a patient who tells you he was prescribed alprazolam (Xanax) for his anxiety. What would anxiety be considered?

  • intended effect
  • contraindication
  • side effect
  • indication
    indication

What information should you include on your PCR related to a patient’s medications?

  • Document the medication names and expiration dates.
  • The trade name and the generic name of each medication.
  • Do not document the medication names; these will be determined at the hospital.
  • Document the medication names and dosages.
    Document the medication names and dosages

If your patient takes nitroglycerin for a heart condition, which of the following routes of administration might he use?

  • inhaled
  • injection
  • sublingual
  • oral
    sublingual

Which of the following is an advantage of MDI drug administration over SVN drug administration?

  • The patient does not need to be conscious for MDI drug administration.
  • MDI medications have no side effects.
  • MDI medications do not expire.
  • The MDI route does not require an external oxygen source.
    The MDI route does not require an external oxygen source.

Activated charcoal comes in which medication form?

  • suspension
  • gel
  • solution
  • tablet
    suspension

Why should you not use an oral route to deliver medication in a patient with an altered LOC?

  • It takes too long for medicine to be delivered using the oral route.
  • The medication will not work as intended in a patient with altered LOC.
  • The patient will likely be combative so an oral route will be difficult.
  • There is a potential for airway compromise.
    There is a potential for airway compromise

During your treatment of a patient having a stroke whose breathing is normal and oxygen saturation is 96%, you administer oxygen via a nonrebreathing mask at 10 to 15 L/min. This is an example of what?

  • knowledge-based error
  • rules-based error
  • therapeutic effect
  • skills-based error
    knowledge-based error

You are treating a 4-year-old who has been vomiting for 2 days. Which route should you use to deliver anti-nausea medication?

  • intramuscular
  • intravenous
  • subcutaneous
  • per rectum
    per rectum

When administering a medication you are unfamiliar with, what is the first thing you should do?

  • Verify the form, dose, and route of the medication.
  • Obtain an order from medical control.
  • Verify the proper medication and prescription.
  • Check the expiration date and condition of the medication.
    Obtain an order from medical control

A 37-year-old male is found unresponsive in his car. His airway is patent and his respirations are rapid and labored. As you and your partner are assessing and treating the patient, a police officer hands you a medication named Alupent, which he found in the backseat of the patient’s car. This medication suggests that the patient has a history of:

  • allergic reactions
  • heart disease
  • asthma
  • hypertension
    asthma

Which of the following statements regarding the epinephrine auto-injector is correct?

  • The auto-injector delivers epinephrine via the subcutaneous route.
  • The adult auto-injector delivers 0.5 to 1 mg of epinephrine.
  • EMTs do not need physician authorization to use the auto-injector.
  • The epinephrine auto-injector delivers a preset amount of the drug.
    The epinephrine auto-injector delivers a preset amount of the drug.

Which of the following statements regarding glucose is correct?

  • Glucose is a complex sugar that rapidly absorbs into the bloodstream.
  • Glucose is usually administered by the EMT via the intravenous route.
  • Glucose is given to patients who are suspected of being hyperglycemic.
  • Glucose is a simple sugar that is readily absorbed by the bloodstream.
    Glucose is a simple sugar that is readily absorbed by the bloodstream.

EMTs respond to a known heroin abuser who is unresponsive. If they give naloxone (Narcan) to this patient, the EMTs should recall that:

  • naloxone should be administered in increments of 2 mg.
  • naloxone should not be given if the patient’s breathing is slow.
  • naloxone administration could cause seizures in this patient.
  • the effects of naloxone last longer than most opioid drugs.
    naloxone administration could cause seizures in this patient.

What is the route of administration for the EpiPen auto-injector?

  • Intraosseous
  • Sublingual
  • Intramuscular
  • Intravenous
    Intramuscular

The process of binding or sticking to a surface is called:

  • digestion.
  • adsorption.
  • suspension.
  • absorption.
    adsorption.

You are treating a 45-year-old woman who was stung by a hornet and has a rash. She tells you that she is allergic to hornets and has her own epinephrine auto-injector. She also tells you that she takes medication for hypertension. Her breath sounds do not reveal any wheezing, her breathing is unlabored, and her blood pressure is 154/94 mm Hg. What should you do if you are not able to make contact with medical control?

  • Begin transport to the hospital and closely monitor her condition while en route.
  • Begin immediate transport and request an intercept with a paramedic ambulance.
  • Give her half the dose of her epinephrine in case her allergic reaction is delayed.
  • Administer her epinephrine, reassess her condition, and transport her promptly.
    Begin transport to the hospital and closely monitor her condition while en route.

The amount of medication that is given is known as the _.

  • indication
  • dose
  • side effect
  • contraindication
    dose

In _ administration, you are administering medication to yourself or your partner.

  • EMT-administered
  • peer-assisted
  • paramedic-administered
  • patient-assisted
    peer-assisted

After taking diphenhydramine (Benadryl) for an allergic reaction, a person begins experiencing drowsiness and a dry mouth. These findings are an example of a(n):

  • unpredictable effect.
  • therapeutic effect.
  • side effect.
  • untoward effect.
    side effect.

Which of the following medication routes delivers a drug through the skin over an extended period of time, such as a nitroglycerin or nicotine patch?

  • Subcutaneous
  • Transcutaneous
  • Sublingual
  • Intraosseous
    Transcutaneous

Which of the following medication routes would be the MOST appropriate to use in an unresponsive patient when intravenous access cannot be obtained?

  • Intraosseous
  • Intramuscular
  • Subcutaneous
  • Transcutaneous
    Intraosseous

The term “pharmacology” is MOST accurately defined as:

  • the study of drug excretion from the human body.
  • the study of how medications affect the brain.
  • the study of drugs that are produced illegally.
  • the study of drugs and their actions on the body.
    the study of drugs and their actions on the body.

How is nitroglycerin usually given by the EMT?

  • Orally
  • Sublingually
  • Inhaled
  • Injected
    Sublingually

What medication form does oral glucose come in?

  • Suspension
  • Fine powder
  • Liquid
  • Gel
    Gel

Which of the following patient populations typically require a modified drug dose?

  • Patients with asthma
  • Females
  • Pediatric patients
  • Middle adults
    Pediatric patients

Advil, Nuprin, and Motrin are trade names for the generic medication:

  • aspirin
  • ibuprofen
  • nitrostat
  • acetaminophen
    ibuprofen

Which of the following is an example of a rules-based medication error?

  • The EMT administers a drug that is contraindicated for the patient.
  • The EMT accidentally gives a higher drug dose than what is indicated.
  • The EMT administers the correct drug, but gives it by the wrong route.
  • The EMT administers a drug that is not approved by the medical director.
    The EMT administers a drug that is not approved by the medical director.

Which of the following is an example of a generic drug?

  • Bayer
  • Aspirin
  • Excedrin
  • Advil
    Aspirin

Shortly after assisting a 65-year-old female with her prescribed nitroglycerin, she begins complaining of dizziness and experiences a drop of 30 mm Hg in her systolic blood pressure. The patient remains conscious and her breathing is adequate. You should:

  • transport her in a sitting position.
  • place her in a supine position.
  • assist ventilations with a bag-valve mask.
  • wait 5 minutes and reassess her blood pressure.
    place her in a supine position.

Which of the following terms is used to describe a balance of all body systems?

  • Hypothermia
  • Diffusion
  • Perfusion
  • Homeostasis
    Homeostasis

You arrive on scene to find a conscious 58-year-old woman sitting up and reporting severe chest pain and shortness of breath. She is anxious and tells you she feels like she is going to die. Physical examination shows that her skin is pale, cool, and clammy and her pulse is rapid, weak, and irregular. Her breathing is labored, with a respiratory rate of 28 breaths/min. Her SpO2 is 90%. Lung sounds show crackles in all fields. Blood pressure is 92/60 mm Hg. What is your differential diagnosis of this patient?

  • Septic shock
  • Hypovolemic shock
  • Neurogenic shock
  • Cardiogenic shock
    Cardiogenic shock

You are treating a patient presenting with labored breathing, absent peripheral pulses, and dilated pupils. These are indications of what?

  • Irreversible shock
  • Psychogenic shock
  • Decompensated shock
  • Compensated shock
    Decompensated shock

Which of the following is indicated in almost every type of shock?

  • Place the patient in a supine position
  • Request ALS assistance
  • Control bleeding
  • Administer high-flow oxygen
    Administer high-flow oxygen

Distributive shock is the result of which of the following?

  • Rapid deoxygenation
  • Poor vessel function
  • Pump failure
  • Low fluid volume
    Poor vessel function

What is perfusion?

  • A passive process in which molecules move from an area with a higher concentration of molecules to an area of lower concentration
  • The presence of abnormally large amounts of fluid between cells in body tissues, causing swelling of the affected area
  • The force or resistance against which the heart pumps
  • The flow of blood through body tissues and vessels
    The flow of blood through body tissues and vessels

What is generally the best position to place a patient with cardiogenic shock?

  • Supine
  • Fowler
  • Left lateral recumbent
  • Sitting/semi sitting
    Siiting/semi sitting

Which of the following is the last measureable factor to change in shock?

  • Blood pressure
  • Heart rate
  • Oxygenation
  • LOC
    Blood pressure

Which of the following is a potentially severe complication of neurogenic shock?

  • Pulmonary embolism
  • Hypothermia
  • Dehydration
  • Syncope
    Hypothermia

When treating a patient in shock from any cause, what is the first thing you should do?

  • Open and maintain the airway
  • Control life-threatening hemorrhage with direct pressure or tourniquet application.
  • Provide high-flow oxygen
  • Maintain normal body temperature
    Open and maintain the airway

Which of the following MOST accurately describes septic shock?

  • Bacterial infection of the nervous system with widespread vasodilation
  • Viral infection of the blood vessels, vascular damage, and vasoconstriction
  • Widespread vasoconstriction and plasma loss due to a severe viral infection
  • Bacterial damage to the vessel wall, leaking blood vessels, and vasodilation
    Bacterial damage to the vessel wall, leaking blood vessels, and vasodilation

Which of the following is the ONLY action that can prevent eventual death from a tension pneumothorax?

  • Early administration of high-flow oxygen
  • Decompression of the injured side of the chest
  • Rapid administration of intravenous fluids
  • Positive-pressure ventilation with a bag-valve mask
    Decompression of the injured side of the chest

You are transporting a 33-year-old male who was involved in a major motor vehicle crash. You have addressed all immediate and potentially life-threatening conditions and have stabilized his condition with the appropriate treatment. With an estimated time of arrival at the hospital of 20 minutes, you should:

  • take his vital signs in 15 minutes.
  • repeat your secondary assessment.
  • arrange for an ALS rendezvous.
  • reassess his condition in 5 minutes.
    reassess his condition in 5 minutes

To protect vital organs, the body compensates by directing blood flow away from organs that are more tolerant of low flow, such as:

  • the skin
  • the lungs
  • the brain
  • the heart
    the skin

Shock due to severe infection is called __.

  • neurogenic shock
  • hypovolemic shock
  • anaphylactic shock
  • septic shock
    septic shock

Your assessment of an unresponsive patient reveals that her breathing is inadequate. Your MOST immediate action should be to:

  • move her to the ambulance stretcher.
  • check her airway for obstructions.
  • administer high-flow oxygen.
  • ventilate her with a bag-valve mask.
    check her airway for obstructions.

Foods, medications, and insects are common causes of __.

  • psychogenic shock
  • septic shock
  • neurogenic shock
  • anaphylactic shock
    anaphylactic shock

A 20-year-old male has a large laceration to his wrist. He is holding a blood-soaked towel over the wound, but it continues to bleed rapidly. You should:

  • apply a tourniquet proximal to the wrist.
  • wrap the towel with pressure bandages.
  • administer high-flow supplemental oxygen.
  • apply pressure to the brachial artery.
    apply a tourniquet proximal to the wrist.

In an acute injury setting, neurogenic shock is commonly accompanied by:

  • tachycardia.
  • hypovolemia
    .
  • diaphoresis.
  • hypothermia.
    hypothermia.

You are dispatched to a residence for a 40-year-old female who fainted. Upon your arrival, the patient is conscious and alert, and states that she is fine. Her husband tells you that she fainted after receiving news that her sister was killed in a car crash. You offer oxygen to the patient, but she refuses to accept it. At this point, your primary concern should be to:

  • provide emotional support regarding her sister.
  • advise her that she needs to go to the hospital.
  • determine if she was injured when she fainted.
  • obtain baseline vital signs and a medical history.
    determine if she was injured when she fainted.

Clinical signs of compensated shock include all of the following, EXCEPT:

  • rapid, shallow breathing.
  • absent peripheral pulses.
  • cool and clammy skin.
  • restlessness or anxiety.
    absent peripheral pulses.

You respond to a residence for a patient with a severe leg injury following an accident with a chainsaw. When you arrive, you find the patient, a 44-year-old male, lying supine in the backyard. He has a partial amputation of his right lower leg that is actively bleeding. The patient is conscious and breathing adequately; however, he is restless and his skin is diaphoretic. You should:

  • apply direct pressure to the wound.
  • immediately evaluate his airway.
  • administer 100% supplemental oxygen.
  • assess the rate and quality of his pulse.
    apply direct pressure to the wound.

Neurogenic shock occurs when:

  • failure of the nervous system causes widespread vasodilation.
  • massive vasoconstriction occurs distal to a spinal cord injury.
  • the spinal cord is severed and causes massive hemorrhaging.
  • there is too much blood to fill a smaller vascular container.
    failure of the nervous system causes widespread vasodilation.

Which of the following statements regarding anaphylactic shock is correct?

  • Anaphylactic shock occurs immediately after a person is sensitized to an allergen.
  • Anaphylactic shock is caused by immune system failure due to a toxic exposure.
  • Sensitized people will experience less severe reactions upon subsequent exposure.
  • Subsequent exposure after sensitization often produces a more severe reaction.
    Subsequent exposure after sensitization often produces a more severe reaction.

When should nonlifesaving interventions be performed for your multisystem trauma patient?

  • Immediately after the injuries are discovered
  • Prior to transport
  • During the primary assessment
  • En route to the hospital
    En route to the hospital

A 56-year-old male is found semiconscious by his wife. Your assessment reveals that his respirations are rapid and shallow, his pulse is rapid and irregular, and his blood pressure is low. The patient’s wife states that he complained of left arm pain and nausea the day before, but would not allow her to call 9-1-1. The MOST likely cause of this patient’s present condition is:

  • acute myocardial infarction.
  • cardiogenic hypoperfusion.
  • severe septic hypoperfusion.
  • a ruptured aortic aneurysm.
    cardiogenic hypoperfusion.

When assessing a patient with signs and symptoms of shock, it is important to remember that:

  • irreversible shock often responds well to a prompt blood transfusion.
  • the patient’s respirations are deep during the early stages of shock.
  • multiple fractures are the most common cause of hypovolemic shock.
  • blood pressure may be the last measurable factor to change in shock.
    blood pressure may be the last measurable factor to change in shock.

Hypovolemic shock caused by severe burns is the result of a loss of:

  • plasma.
  • platelets.
  • whole blood.
  • red blood cells.
    plasma.

A 19-year-old male was stung multiple times by fire ants. He is experiencing obvious signs and symptoms of anaphylactic shock. You administer high-flow oxygen and give him epinephrine via intramuscular injection. Upon reassessment, you determine that his condition has not improved. You should:

  • transport him immediately and provide supportive care while en route.
  • repeat the epinephrine injection after consulting with medical control.
  • request a paramedic unit that is stationed approximately 15 miles away.
  • consider that he may actually be experiencing an acute asthma attack.
    repeat the epinephrine injection after consulting with medical control.

A 25-year-old unrestrained female struck the steering wheel with her chest when her car hit a tree while traveling at a high rate of speed. She has signs and symptoms of shock, which you suspect are the result of intrathoracic bleeding. Which of the following interventions will provide this patient with the greatest chance for survival?

  • High-flow oxygen administration
  • Full immobilization of her spine
  • Intravenous fluid administration
  • Rapid transport to a trauma center
    Rapid transport to a trauma center

Which intervention would have the MOST important impact on the cardiac patient’s outcome?

  • Cardiac medications
  • Early CPR and defibrillation
  • IV fluid administration
  • Advanced airway management
    Early CPR and defibrillation

The AED gives a “no shock” message to a patient who is in cardiac arrest. What should you do?

  • Resume chest compressions
  • Deliver two rescue breaths
  • Reanalyze the cardiac rhythm
  • Check for a carotid pulse
    Resume chest compressions

What is the maximum amount of time that should be spent checking for spontaneous breathing in an unresponsive child?

  • 15 seconds
  • 5 seconds
  • 10 seconds
  • 20 seconds
    10 seconds

When performing CPR on an adult, you should compress the chest to what depth and at what a rate of compressions per minute?

  • 1 to 1.4 inches (2.5 to 3.5 cm); 100 to 120 compressions per minute
  • 2 to 2.4 inches (5 to 6 cm); 100 to 120 compressions per minute
  • 1 to 1.4 inches (2.5 to 3.5 cm); 80 to 100 compressions per minute
  • 2 to 2.4 inches (5 to 6 cm); 80 to 100 compressions per minute
    2 to 2.4 inches (5 to 6 cm); 100 to 120 compressions per minute

What is the proper compression-to-ventilation ratio for adult two-rescuer CPR?

  • 15:2
  • 50:2
  • 60:2
  • 30:2
    30:2

When you are performing CPR on an adult or child, approximately how often should you reassess the patient for return of respirations and/or circulation?

  • Every minute
  • Every 5 minutes
  • Every 2 minutes
  • Every 3 minutes
    Every 2 minutes

What is the preferred method of removing a foreign body in an unresponsive child?

  • Back slaps
  • Chest compressions
  • Abdominal thrusts
  • Manual removal
    Abdominal thrusts

You are off duty and you come across an unresponsive child lying on the ground at a playground. She does not have a pulse and no one witnessed the collapse. What should you do?

  • Take the child to the nearest hospital in your vehicle
  • Call 9-1-1 immediately, then return to the child and begin CPR
  • Do 5 cycles of chest compressions, and then call 9-1-1.
  • Call for ALS backup immediately.
    Do 5 cycles of chest compressions, and then call 9-1-1

You respond to a call in which a 6-year-old was hit by a car. She is responsive but struggling to breathe. What position should you place her in?

  • Fowler
  • Supine
  • Any position of comfort
  • Recovery
    Any position of comfort

When performing CPR on an infant, which of the following is correct?

  • Place the heel of one hand on the lower half of the sternum and the other hand over the first hand
  • Place two fingers in the middle of the sternum, just below the nipple line
  • Use your index fingers on the lower half of the sternum
  • Place the heel of one or two hands in the center of the chest, in between the nipples, avoiding the xiphoid process.
    Place two fingers in the middle of the sternum, just below the nipple line

Several attempts to adequately open a trauma patient’s airway with the jaw-thrust maneuver have been unsuccessful. You should:

  • carefully perform the head tilt-chin lift maneuver
  • try opening the airway by lifting up on the chin
  • tilt the head back while lifting up on the patient’s neck
  • suction the airway and reattempt the jaw-thrust maneuver
  • carefully perform the head tilt-chin lift maneuver

You are off duty at a park when you witness an apparently healthy 12-year-old child suddenly collapse. There are no bystanders around and your mobile phone is in your car. After confirming that the child is in cardiac arrest, you should:

  • perform CPR for 2 minutes and then call 9-1-1
  • perform chest compressions only until a bystander arrives
  • deliver five rescue breaths before starting chest compressions
  • call 9-1-1 and then return to begin CPR on the child
    call 9-1-1 and then return to begin CPR on the child

Between each chest compression, you should __.

  • remove your hands from the chest
  • check for a pulse
  • allow full chest recoil
  • administer a breath
    allow full chest recoil

A 60-year-old man is found to be unresponsive, pulseless, and apneic. You should:

  • start CPR and transport immediately
  • begin CPR until an AED is available
  • determine if he has a valid living will
  • withhold CPR until he is defibrillated
    begin CPR until an AED is available

What is the correct compression-to-ventilation ratio for adult CPR?

  • 3:2
  • 30:2
  • 30:1
  • 5:1
    30:2

If an object is visible in the unconscious patient’s airway, you should __.

  • comtinue chest compressions
  • remove it
  • place the patient on his or her side
  • leave it in place
    remove it

A young male is unresponsive after overdosing on an opiod. He is not breathing and his pulse is weak. The EMT should immediately:

  • ventilate with a BVM
  • administer naloxene
  • request an ALS ambulance
  • begin chest compressions
    ventilate with a BVM

The proper depth of chest compressions on a 9-month-old infant is:

  • one-half to two-thirds the diameter of the chest
  • two-thirds the diameter of the chest, or about 2 inches
  • one-third the diameter of the chest, or about 1 1/2 inches
  • one-half the diameter of the chest, or about 1 1/2 inches
    one-third the diameter of the chest, or about 1 1/2 inches

You should deliver chest compressions to an unresponsive adult patient in cardiac arrest by:

  • compressing the sternum between the nipples
  • depressing the sternum more than 2.5 inches in depth
  • compressing quickly and releasing slowly
  • placing the heel of your hand on the xiphoid
    compressing the sternum between the nipples

What is the correct ratio of compressions to ventillations when performing two-rescuer child CPR?

  • 3:1
  • 30:2
  • 5:1
  • 15:2
    15:2

The MOST appropriate treatment for a patient with a mild upper airway onstruction includes:

  • performing five back blows and five abdominal thrusts
  • administering oxygen and transporting immediately
  • visualizing the airway and removing the obstruction
  • advising the patient not to make any attempts to cough
    administering oxygen and transporting immediately

Which of the following techniques should you ise to dislodge a foreign body airway obstruction in a patient who is an advanced stage of pregnancy or who is very obese?

  • Back blows
  • Abdominal thrusts
  • Finger sweeps
  • Chest thrusts
    Chest thrusts

Your conscious patient has a mild partial airway obstruction. You should:

  • perform abdominal thrusts
  • encourage the patient to cough
  • administer back blows
  • place the patient supine
    encourage the patient to cough

Most prehospital cardiac arrests occur as the result of:

  • an acute ischemic stroke
  • obstruction of the airway
  • a cardiac dysrhythmia
  • severe blunt trauma
    a cardiac dysrhythmia

CPR will NOT be effective if the patient is:

  • horizontal
  • on a firm surface
  • supine
  • prone
    prone

CPR retraining is the MOST effective when it:

  • is delivered by computer
  • involves hands-on practice
  • occurs every 24 months
  • is self-paced and brief
    involves hands-on practice

CPR should be initiated when:

  • the carotid pulse is very weak
  • signs of putrefaction are present
  • rigor mortis is obvious
  • a valid living will is unavailable
    a valid living will is unavailable

When assessing the pulse of an unresponsive infant, you should palpate the _ artery.

  • carotid
  • radial
  • femoral
  • brachial
    brachial

The impedance threshold devide (ITD) may improve circulation during active compression-decompression CPR by:

  • maximizing the amount of air in the lungs following chest recoil, which hyperinflates the lungs and forces more blood from the ventricle during each compression
  • drawing all of the air out of the lungs in between chest compressions, which causes positive intrathoracic pressure and a reduction of blood return to the right side of the heart
  • maintaining increased intrathoracic pressure during the downward stroke of each chest compression, which forces more blood from both of the ventricles
  • limiting the amount of air that enters the lungs during the recoil phase between chest compressions, which results in negative intrathoracic pressure and improved cardiac filling
    limiting the amount of air that enters the lungs during the recoil phase between chest compressions, which results in negative intrathoracic pressure and improved cardiac filling

If gastric distention begins to make positive-pressure ventilation difficult, you should:

  • reposition the patient’s airway
  • increase the rate of ventilation
  • suction the patient’s oropharynx
  • insert an oropharyngeal airway
    reposition the patient’s airway

An anaphylactic reaction constitutes what type of medical emergency?

  • Cardiovascular
  • Toxicologic
  • Hematologic
  • Immunologic
    Immunologic

At what point should you make a preliminary transport decision?

  • During reassesment
  • During scene size-up
  • After secondary assessment
  • Once the preliminary assessment is complete
    Once the preliminary assessment is complete

You are dispatched to a home where you find a 45-year-old woman in apparent respiratory distress. She is using accessory muscles to help breathe and can only respond with short answers. What is the first step in your assessment process?

  • Obtain vital signs
  • Determine scene safety
  • Manage the airway
  • Perform a secondary assessment
    Determine scene safety

What mnemonic is used to determine a patient’s chief complaint?

  • AVPU
  • OPQRST
  • SAMPLE
  • TACOS
    OPQRST

HIV is caused by what type of organism?

  • Virus
  • Fungus
  • Bacteria
  • Protozoa
    Virus

Jaundice is a sign of which disease?

  • Hepatitis
  • Meningitis
  • HIV
  • Herpes simplex
    Hepatitis

Why is tuberculosis not more common than it is?

  • Infected air is easily diluted with uninfected air, so transmission is inefficient
  • Droplet nuclei that spread the infection have a very short lifespan
  • The BCG vaccine is 95% effective
  • Human beings have natural immunity
    Infected air is easily diluted with uninfected air, so transmission is inefficient

Which of the following diseases is the most virulent?

  • HIV
  • Herpes simplex
  • Hepatitis B
  • Tuberculosis
    Hepatitis B

What is an index of suspicion?

  • The ability to determine how contagious an infectious disease is
  • An outcome of scene size-up that indicates whether law enforcement should be called
  • An awareness that unseen life-threatening injuries or illness may exist
  • A level of understanding whereby you can determine multiple NOIs
    An awareness that unseen life-threatening injuries or illness may exist

Which of the following best describes a communicable disease?

  • A disease that can be spread from one person or species to another
  • Presence of infectious organisms on or in objects
  • The growth and spread of small harmful organisms within the body
  • A disease that is transmitted through contaminated drinking water
    A disease that can be spread from one person or species to another

The secondary assessment of a medical patient:

  • is not practical if the patient is critically ill or your transport time is short
  • should routinely include a comprehensive examination from head to toe
  • is typically limited to a focused exam for patients who are unconscious
  • should be performed at the scene, especially if the patient is critically ill
    is not practical if the patient is critically ill or your transport time is short

Which of the following medications would the EMT be LEAST likely to administer to a patient with a medical complaint?

  • Oral glucose
  • Aspirin
  • Albuterol
  • Ibuprofen
    Ibuprofen

Which of the following assessment findings is MOST indicative of a cardiovascular problem?

  • Jugular venous distention
  • Unequal breath sounds
  • Use of the accessory muscles
  • Palpable pain to the epigastrium
    Jugular venous distention

When caring for a patient who takes numerous medications, it is best to:

  • let the hospital staff retrieve the patient’s medical records, which should show a list of his or her current medications
  • take all of the patient’s medications with you to the hospital and document them on your patient care report
  • send the patient’s medications to the hospital with a family member or other person who will safeguard them
  • document the medications on your patient care report, but leave them at home so they do not get misplaced
    take all of the patient’s medications with you to the hospital and document them on your patient care report

Reassessment of a patient with a medical complaint should begin by:

  • taking another set of vital signs.
  • reassessing the nature of illness.
  • reviewing all treatment performed.
  • repeating the primary assessment.
    repeating the primary assessment.

The BEST way to prevent infection from whooping cough is to:

  • get vaccinated against diphtheria, tetanus, and pertussis.
  • routinely place a surgical mask on all respiratory patients.
  • wear a HEPA mask when treating any respiratory patient.
  • ask all patients if they have recently traveled abroad.
    get vaccinated against diphtheria, tetanus, and pertussis.

In 2009, the H1N1 virus accounted for over 200,000 deaths worldwide in the form of the swine flu. In 1919, a similar outbreak of the H1N1 occurred in the form of the Spanish flu. Starting in Kansas City, the virus spread rapidly worldwide, claiming up to 50 million lives. These are both examples of:

  • epidemics.
  • parasitic infection.
  • pandemics.
  • uncontrolled outbreaks.
    pandemics.

When performing a secondary assessment on a conscious patient with nontraumatic abdominal pain and stable vital signs, you should:

  • prepare the patient for transport first.
  • only palpate tender areas of the abdomen.
  • examine the patient from head to toe.
  • focus on his or her chief complaint.
    focus on his or her chief complaint.

After sizing up the scene of a patient with a possible infectious disease, your next priority should be to:

  • take standard precautions.
  • contact medical control.
  • notify law enforcement.
  • quickly access the patient.
    take standard precautions.

In addition to looking for severe bleeding, assessment of circulation in the conscious patient should involve:

  • applying a pulse oximeter probe to the finger to determine if peripheral perfusion is adequate.
  • taking a blood pressure and determining if the patient is alert and oriented or confused.
  • checking the radial pulse and noting the color, temperature, and condition of the skin.
  • palpating the carotid pulse to determine the approximate rate and checking capillary refill time.
    checking the radial pulse and noting the color, temperature, and condition of the skin.

In contrast to viral hepatitis, toxin-induced hepatitis:

  • can be prevented with a vaccination.
  • is not a communicable disease.
  • typically does not cause yellow skin.
  • is a far more transmittable disease.
    is not a communicable disease.

You and your EMT partner arrive at the residence of a 50-year-old man who complains of weakness. Your primary assessment reveals that he is critically ill and will require aggressive treatment. The closest hospital is 25 miles away. You should:

  • administer oxygen via nonrebreathing mask and obtain as much of his medical history as possible.
  • manage all threats to airway, breathing, and circulation and consider requesting an ALS unit.
  • load him into the ambulance, begin transport, and perform all treatment en route to the hospital.
  • perform a detailed secondary assessment, assess his vital signs, and then transport rapidly.
    manage all threats to airway, breathing, and circulation and consider requesting an ALS unit.

When forming your general impression of a patient with a medical complaint, it is important to remember that:

  • the majority of medical patients you encounter are also injured.
  • most serious medical conditions do not present with obvious symptoms.
  • the conditions of many medical patients may not appear serious at first.
  • it is during the general impression that assessment of the ABCs occurs.
    the conditions of many medical patients may not appear serious at first.

We have an expert-written solution to this problem!
Which of the following conditions is NOT categorized as a psychiatric condition?

  • Alzheimer disease
  • Schizophrenia
  • Substance abuse
  • Depression
    Substance abuse

Most patients with an infectious disease will have _.

  • abdominal pain
  • a low blood glucose level
  • a fever
  • seizures
    a fever

Ten days after treating a 34-year-old patient with tuberculosis, you are given a tuberculin skin test, which yields a positive result. This MOST likely indicates that:

  • the disease is dormant in your body, but will probably never cause symptoms.
  • you were exposed to another infected person prior to treating the 34-year-old patient.
  • you are actively infected with tuberculosis and should be treated immediately.
  • you contracted the disease by casual contact instead of exposure to secretions.
    you were exposed to another infected person prior to treating the 34-year-old patient.

Assessment of a patient’s blood pressure with an automatic BP cuff reveals that it is 204/120 mm Hg. The patient is conscious and alert and denies any symptoms. The EMT should:

  • conclude that she has hypertension.
  • obtain a manual blood pressure.
  • reassess her blood pressure in 5 minutes.
  • prepare for immediate transport.
    obtain a manual blood pressure.

Your primary assessment of an elderly woman reveals that she is conscious and alert, but is experiencing difficulty breathing. She has a history of emphysema, hypertension, and congestive heart failure. As you assess the patient’s circulatory status, you should direct your partner to:

  • retrieve the stretcher and prepare for transport.
  • perform a head-to-toe secondary assessment.
  • assess her oxygen saturation and blood pressure.
  • administer oxygen with the appropriate device.
    administer oxygen with the appropriate device.

It is especially important to assess pulse, sensation, and movement in all extremities as well as pupillary reactions in patients with a suspected _ problem.

  • respiratory
  • cardiac
  • endocrine
  • neurologic
    neurologic

An infectious disease is MOST accurately defined as:

  • the invasion of the human body by a bacterium that cannot be destroyed by antibiotics or other drugs.
  • a disease that can be spread from one person or species to another through a number of mechanisms.
  • any disease that enters the body via the bloodstream and renders the immune system nonfunctional.
  • a medical condition caused by the growth and spread of small, harmful organisms within the body.
    a medical condition caused by the growth and spread of small, harmful organisms within the body.

Which of the following medications can be used for the treatment of an acute asthma attack?

  • Cromolyn
  • Albuterol
  • Fluticasone
  • Salmeterol
    Albuterol

What is the most appropriate method for oxygen delivery to an adult patient experiencing breathing difficulty?

  • Venturi mask at 8 L/min
  • Nasal cannula at 2 to 6 L/min
  • BVM at 15 L/min
  • Nonrebreathing mask at 15 L/min
    Nonrebreathing mask at 15 L/min

When assisting an asthmatic patient with a small-volume nebulizer attached to oxygen, what is the appropriate flow rate for the oxygen?

  • 2 L/min
  • 10 L/min
  • 4 L/min
  • 6 L/min
    6 L/min

In addition to asthma, which of the following conditions is associated with wheezing?

  • Bronchitis
  • Pulmonary embolism
  • Epiglottitis
  • Croup
    Bronchitis

What is atelectasis?

  • The buildup of excess acid in the blood or body tissues that results from a primary illness
  • The buildup of excess base (lack of acids) in the body fluids
  • Collapse of the alveolar air spaces of the lungs
  • An extreme, life-threatening, systemic allergic reaction that may include shock and respiratory failure
    Collapse of the alveolar air spaces of the lungs

Which type of breath sound are you more likely to hear in a person with congestive heart failure?

  • Crackles
  • Rhonchi
  • Decreased/absent
  • Stridor
    Crackles

You arrive at a residence where you find a woman in her early 60s. She is anxious, tachycardic, and her fingertips and lips are blue. As you assess her, she coughs up frothy sputum, and you hear crackles and some wheezing as you check for breath sounds. What condition do these findings indicate?

  • Emphysema
  • Bronchitis
  • Congestive heart failure
  • Spontaneous pneumothorax
    Congestive heart failure

In the mnemonic PASTE, what does the “S” stand for?

  • Sputum
  • Synchope
  • Severity
  • Symptoms
    Sputum

CPAP is generally used for which condition?

  • Pleural effusion
  • Hyperventilation
  • Spontaneous pneumothorax
  • Acute pulmonary edema
    Acute pulmonary edema

Which of the following statements is true regarding asthma?

  • Asthma involves excessive mucus production.
  • Asthma involves accumulation of air in the pleural space.
  • Asthma involves a collection of fluid in the alveoli.
  • Asthma involves a collection of fluid in the pleural space.
    Asthma involves excessive mucus production.

Your patient has a chronic respiratory condition. His stimulus to breathe is triggered by low oxygen levels in the blood. This is known as the _.

  • hypoxic drive
  • CO2 drive
  • COPD drive
  • alternate drive
    hypoxic drive

You are assisting an asthma patient with his prescribed metered-dose inhaler. After the patient takes a deep breath and depresses the inhaler, you should:

  • advise him to exhale forcefully to ensure medication absorption.
  • allow him to breathe room air and assess his oxygen saturation.
  • instruct him to hold his breath for as long as he comfortably can.
  • immediately reapply the oxygen mask and reassess his condition.
    instruct him to hold his breath for as long as he comfortably can.

Crackles (rales) are caused by _.

  • narrowing of the upper airways
  • severe bronchoconstriction
  • air passing through fluid
  • mucus in the larger airways
    air passing through fluid

When auscultating the lungs of a patient with respiratory distress, you hear adventitious sounds. This means that the patient has:

  • abnormal breath sounds.
  • an absence of breath sounds.
  • normal breath sounds.
  • diminished breath sounds.
    abnormal breath sounds.

Treatment with continuous positive airway pressure (CPAP) would MOST likely be contraindicated in which of the following situations?

  • Conscious and alert patient with an oxygen saturation of 85%
  • Difficulty breathing, two-word dyspnea, and tachycardia
  • Pulmonary edema, history of hypertension, and anxiety
  • Shortness of breath and a blood pressure of 76/56 mm Hg
    Shortness of breath and a blood pressure of 76/56 mm Hg

His parents tell you that their son has had a chest infection for the past two days and when they took him to their family doctor, they were told it was likely due to the respiratory syncytial virus (RSV). They have kept him well hydrated, but the infection seems to have gotten worse. On auscultation, you hear decreased air entry bilaterally with fine expiratory wheezes and the occasional coarse wet crackle. Based on this information, your patient is most likely suffering from:

  • pertussis.
  • epiglottitis.
  • croup.
  • bronchiolitis.
    bronchiolitis.

A 30-year-old male presents with acute shortness of breath, widespread hives, and facial swelling. He denies any past medical history and takes no medications. During your assessment, you hear wheezing over all the lung fields. His blood pressure is 90/50 mm Hg and his heart rate is 110 beats/min. In addition to giving him high-flow oxygen, the MOST important treatment for this patient is:

  • epinephrine.
  • a beta-antagonist.
  • albuterol.
  • an antihistamine.
    epinephrine.

Which of the following must be assessed in every respiratory patient?

  • Blood glucose levels
  • Lung sounds
  • Distal pulse, motor, sensation
  • Orthostatic vital signs
    Lung sounds

Asthma is caused by a response of the:

  • cardiovascular system.
  • immune system.
  • respiratory system.
  • endocrine system.
    immune system.

A pleural effusion is MOST accurately defined as:

  • a bacterial infection of the lung tissue.
  • diffuse collapsing of the alveoli.
  • a unilaterally collapsed lung.
  • fluid accumulation outside the lung.
    fluid accumulation outside the lung.

Which of the following conditions would be LEAST likely to result in hypoxia?

  • Narcotic overdose
  • Pleural effusion
  • Severe anxiety
  • Pulmonary edema
    Severe anxiety

Which of the following is MOST characteristic of adequate breathing?

  • 30 breaths/min with supraclavicular retractions and clammy skin
  • 24 breaths/min with bilaterally equal breath sounds and pink skin
  • 22 breaths/min with an irregular pattern of breathing and cyanosis
  • 20 breaths/min with shallow movement of the chest wall and pallor
    24 breaths/min with bilaterally equal breath sounds and pink skin

“PASTE” is an alternate assessment tool for _.

  • respiratory patients
  • stroke patients
  • cardiac patients
  • seizure patients
    respiratory patients

While auscultating an elderly woman’s breath sounds, you hear low-pitched “rattling” sounds at the bases of both of her lungs. This finding is MOST consistent with which of the following conditions?

  • Aspiration pneumonia
  • Early pulmonary edema
  • Acute asthma attack
  • Widespread atelectasis
    Aspiration pneumonia

In what area of the lungs does respiration occur?

  • Alveoli
  • Capillaries
  • Trachea
  • Bronchi
    Alveoli

A 59-year-old male with a history of emphysema complains of an acute worsening of his dyspnea and pleuritic chest pain following a forceful cough. Your assessment reveals that he has a barrel-shaped chest, unilaterally diminished breath sounds, and tachycardia. What is the MOST likely cause of this patient’s condition?

  • Acute pulmonary embolism
  • Rupture of the diaphragm
  • Exacerbation of his COPD
  • Spontaneous pneumothorax
    Spontaneous pneumothorax

You are attending to a 54-year-old female patient in a homeless shelter. The patient tells you that she had the flu a couple of weeks ago, and she has not gotten over it. She has been tired and keeps waking up at night, sweating. She has been coughing up green sputum occasionally and has been experiencing episodes of chest pain that get worse when she breathes. Based on this information, your patient is most likely suffering from:

  • influenza Type A.
  • chronic obstructive pulmonary disease (COPD).
  • pneumonia.
  • tuberculosis.
    tuberculosis.

You are attending to a 3-year-old male patient who is presenting with severe shortness of breath. His parents report that he has had a cough and cold with a low grade fever for the past two days. They became worried today, as his level of distress has increased dramatically. On assessment, the patient is sitting upright and making high-pitched noises with each breath. Based on this information, the patient is most likely suffering from:

  • inflammation of the bronchioles.
  • bacterial infection of the epiglottis.
  • inflammation of the lower respiratory tract and bronchospasm.
  • viral infection of the upper respiratory tract.
    viral infection of the upper respiratory tract.

Which of the following is a genetic disorder that predisposes the patient to repeated lung infections?

  • Cystic fibrosis
  • Celiac sprue
  • Multiple sclerosis
  • Severe acute respiratory syndrome
    Cystic fibrosis

In order for efficient pulmonary gas exchange to occur:

  • the percentage of inhaled carbon dioxide must exceed the percentage of inhaled oxygen.
  • the pulmonary capillaries must be completely constricted and the alveoli must be collapsed.
  • there must be low quantities of pulmonary surfactant to allow for full alveolar expansion.
  • oxygen and carbon dioxide must be able to freely diffuse across the alveolar-capillary membrane.
    oxygen and carbon dioxide must be able to freely diffuse across the alveolar-capillary membrane.

You are dispatched to a 60-year-old man reporting chest pain and shortness of breath. The patient has angina and is taking nitroglycerin, furosemide, and atorvastatin. You hear crackles when listening to his breath sounds. The patient’s difficulty breathing and crackles are due to blood backing up in which part of the body?

  • The vessels
  • The heart
  • The arteries
  • The lungs
    The lungs

What is cardiac output?

  • The amount of venous flow return to the right atrium
  • The number of times the heart contracts in 1 minute
  • The volume of blood pumped out by the left ventricle in one contraction
  • The amount of blood pumped out of the left ventricle in 1 minute
    The amount of blood pumped out of the left ventricle in 1 minute

Most often, low blood flow to heart tissue is caused by what?

  • Acute myocardial infarction
  • Atherosclerosis
  • Thromboembolism
  • Ventricular fibrillation
    Atherosclerosis

What is the most effective way to assist a person with CHF to breathe effectively and prevent an invasive airway management technique?

  • Nasal cannula
  • BVM with 100% oxygen
  • Nonrebreathing mask
  • CPAP
    CPAP

You are treating a 50-year-old woman who has just finished a 10k run. She is complaining of crushing pain in the chest radiating down her left arm and nausea, which came on suddenly. She is sweating profusely. What condition should you suspect?

  • Cardiogenic shock
  • Acute myocardial infarction
  • Angina pectoris
  • Congestive heart failure
    Angina pectoris

Which dysrhythmia is the most common cause of sudden death in a cardiovascular emergency?

  • Bradycardia
  • Ventricular fibrillation
  • Ventricular tachycardia
  • Tachycardia
    Ventricular fibrillation

You are treating a 63-year-old patient whose vital signs are as follows: pulse, 140 beats/min and irregular; respiratory rate, 28 breaths/min; and blood pressure, 90/50 mm Hg. He is complaining of chest pain. Given this information, why would nitroglycerin be contraindicated?

  • His blood pressure is too low.
  • He may have taken Viagra in the last 24 hours.
  • He may have already exceeded the dosage limit
  • He may be allergic to it
    His blood pressure is too low.

You are transporting a patient with angina and he loses consciousness on the way to the hospital. When you check, you cannot feel a pulse. What should you do?

  • Tell your partner to drive faster as you begin CPR on the patient.
  • Have your partner pull over the ambulance and come back to help you with CPR and the AED.
  • Immediately apply the AED and wait for it to analyze the rhythm before taking any other action.
  • Begin CPR and after 2 minutes stop to apply the AED while your partner continues driving to the hospital.
    Have your partner pull over the ambulance and come back to help you with CPR and the AED.

Which of the following represents the MOST appropriate method of assisting a patient with his or her prescribed nitroglycerin tablet?

  • Wait 15 minutes and reassess the patient’s blood pressure prior to administering another dose.
  • Place the medication under the tongue and have the patient swallow it.
  • Administer the medication sublingually and allow it to dissolve or absorb.
  • Encourage the patient to chew the tablet to increase its effectiveness.
    Administer the medication sublingually and allow it to dissolve or absorb.

Acute coronary syndrome is a group of symptoms caused by what?

  • Thromboembolism
  • Occlusion
  • Ischemia
  • Bradycardia
    Ischemia

Most AEDs are set up to adjust the voltage based on the impedance, which is the:

  • direction that the electrical flow takes in the body.
  • distance between the two AED pads on the chest.
  • resistance of the body to the flow of electricity.
  • actual amount of energy that the AED will deliver.
    resistance of the body to the flow of electricity.

Nitroglycerin is contraindicated in patients:

  • with a systolic blood pressure less than 120 mm Hg.
  • who have taken up to two doses.
  • with a history of an ischemic stroke.
  • who have experienced a head injury.
    who have experienced a head injury.

Risk factors for AMI that cannot be controlled include:

  • excess stress.
  • lack of exercise.
  • family history.
  • hyperglycemia.
    family history.

A patient with atherosclerotic heart disease experiences chest pain during exertion because:

  • the ragged edge of a tear in the coronary artery lumen causes local blood clotting and arterial narrowing.
  • the lumen of the coronary artery is narrowed and cannot accommodate increased blood flow.
  • tissues of the myocardium undergo necrosis secondary to a prolonged absence of oxygen.
  • the coronary arteries suddenly spasm and cause a marked reduction in myocardial blood flow.
    the lumen of the coronary artery is narrowed and cannot accommodate increased blood flow.

A 49-year-old male presents with an acute onset of crushing chest pain and diaphoresis. You should:

  • administer up to three doses of nitroglycerin.
  • administer up to 324 mg of baby aspirin.
  • assess the adequacy of his respirations.
  • obtain vital signs and a SAMPLE history.
    assess the adequacy of his respirations.

When preparing to obtain a 12-lead ECG, the “LL” and “RL” electrodes should be placed:

  • on either side of the chest.
  • anywhere on the arms.
  • on the lower abdomen.
  • on the thighs or ankles.
    on the thighs or ankles.

Angina pectoris occurs when:

  • one or more coronary arteries suddenly spasm.
  • a coronary artery is totally occluded by plaque.
  • myocardial oxygen supply exceeds the demand.
  • myocardial oxygen demand exceeds supply.
    myocardial oxygen demand exceeds supply.

Which of the following signs is commonly observed in patients with right-sided heart failure?

  • Pulmonary edema
  • Labored breathing
  • Dependent edema
  • Flat jugular veins
    Dependent edema

A 66-year-old female with a history of hypertension and diabetes presents with substernal chest pressure of 2 hours’ duration. Her blood pressure is 140/90 mm Hg, her pulse is 100 beats/min and irregular, her respirations are 22 breaths/min, and her oxygen saturation is 92%. The patient does not have prescribed nitroglycerin, but her husband does. You should:

  • give her one nitroglycerin and reassess her systolic blood pressure.
  • obtain a SAMPLE history and contact medical control for advice.
  • administer oxygen, give her 324 mg of aspirin, and assess her further.
  • give her high-flow oxygen, attach the AED, and transport at once.
    administer oxygen, give her 324 mg of aspirin, and assess her further.

A dissecting aortic aneurysm occurs when:

  • the inner layers of the aorta become separated.
  • the aorta ruptures, resulting in profound bleeding.
  • all layers of the aorta suddenly contract.
  • a weakened area develops in the aortic wall.
    the inner layers of the aorta become separated.

After the AED has delivered a shock, the EMT should:

  • re-analyze the cardiac rhythm.
  • immediately resume CPR.
  • assess for a carotid pulse.
  • transport the patient at once.
    immediately resume CPR.

Prior to attaching the AED to a cardiac arrest patient, the EMT should:

  • dry the chest if it is wet.
  • assess for a pulse for 20 seconds.
  • contact medical control.
  • perform CPR for 30 seconds.
    dry the chest if it is wet.

The EMT should use an AED on a child between 1 month and 8 years of age if:

  • his or her condition is rapidly progressing to cardiac arrest.
  • pediatric pads and an energy-reducing device are available.
  • he or she is not breathing and has a weakly palpable pulse.
  • special pads are used and the child has profound tachycardia.
    pediatric pads and an energy-reducing device are available.

Signs and symptoms of a hypertensive emergency would MOST likely be delayed in patients who:

  • have had a stroke in the past.
  • regularly take illegal drugs.
  • are older than 40 years of age.
  • have chronic hypertension.
    have chronic hypertension.

A patient in cardiac arrest is wearing an external defibrillator vest, which is interfering with effective chest compressions. The EMT should:

  • remove the battery from the monitor and leave the vest in place.
  • remove the battery from the monitor and then remove the vest.
  • leave the battery attached to the monitor and remove the vest.
  • perform ventilations only and allow the vest device to defibrillate.
    remove the battery from the monitor and then remove the vest.

The electrical impulse generated by the heart originates in the:

  • sinoatrial node.
  • atrioventricular node.
  • bundle of His.
  • coronary sinus.
    sinoatrial node.

Deoxygenated blood from the body returns to the:

  • right ventricle.
  • left ventricle.
  • left atrium.
  • right atrium.
    right atrium.

Which of the following is NOT a common sign or symptom associated with malfunction of an implanted cardiac pacemaker?

  • Generalized weakness
  • Heart rate less than 60 beats/min
  • A rapid heart rate
  • Syncope or dizziness
    A rapid heart rate

You and your partner arrive at the scene of a middle-aged man who collapsed about 5 minutes ago. He is unresponsive, apneic, and pulseless. Bystanders are present, but have not provided any care. You should:

  • have your partner perform CPR while you question the bystanders.
  • begin high-quality CPR and apply the AED as soon as possible.
  • perform two-rescuer CPR for 5 minutes and request ALS backup.
  • immediately apply the AED pads and analyze his cardiac rhythm.
    begin high-quality CPR and apply the AED as soon as possible.

A patient tells you that he has a left ventricular assist device (LVAD). Which of the following conditions should you suspect that he has experienced?

  • Uncontrolled hypertension
  • Thoracic aortic aneurysm
  • Acute myocardial infarction
  • Obstructive lung disease
    Acute myocardial infarction

You are treating a patient who is exhibiting slurred speech, facial droop, and an inability to move his left arm. Which neurologic examination emphasizes these possible stroke signs?

  • 3-Item Stroke Severity Scale
  • Glasgow Coma Scale
  • Chicago Prehospital Stroke Scale
  • Cincinnati Prehospital Stroke Scale
    Cincinnati Prehospital Stroke Scale

What happens when blood flow to a particular part of the brain is cut off by a blockage, resulting in tissue damage?

  • Status epilepticus
  • Ischemic stroke
  • Seizure
  • Hemorrhagic stroke
    Ischemic stroke

What is the name of the condition when the patient forgets about the injured side after a stroke?

  • Neglect
  • Hemiparesis
  • Ataxia
  • Aphasia
    Neglect

What criteria must be met for a patient to have status epilepticus?

  • A seizure lasting more than 30 minutes
  • A seizure that was preceded by an aura
  • The presence of incontinence during the seizure
  • Multiple seizures with normal consciousness between each event
    A seizure lasting more than 30 minutes

Which of the following is a metabolic cause for a seizure?

  • Brain tumor
  • High fever
  • Head trauma
  • Hypoglycemia
    Hypoglycemia

Which of the following mimics a stroke and also causes a seizure?

  • Meningitis
  • Migraine headache
  • Postictal state
  • Hypoglycemia
    Hypoglycemia

What is the difference between a stroke and a transient ischemic attack?

  • A TIA affects a limited portion of the brain
  • A TIA has longer-lasting effects than a stroke
  • A TIA involves surges of electrical activity in the brain
  • A TIA resolves completely within 24 hours of onset
    A TIA resolves completely within 24 hours of onset

You are dispatched to a home where you find a 70-year-old man. He is exhibiting facial drooping and, when you ask him to tell you what day it us, he says “January.” Which part of the brain is most likely to have been affected?

-Right hemisphere

  • Left hemisphere
  • Cerebellum
  • Brain stem
    Left hemisphere

You are examining a patient whose signs include lip smacking, jerking of the left arm, and agitation. Based on these signs, what type if seizure is this patient experiencing?

  • Status epilepticus
  • Complex partial
  • Simple partial
  • Generalized
    Complex partial

You are performing a Glascow Coma Scale on a patient who opens her eyes in response to pain, is confused in her responses to questions, and has abnormal flexion. What is this patient’s GCS score and corresponding state of dysfunction?

  • 6- severe dysfunction
  • 9- moderate dysfunction
  • 12-mild dysfunction
  • 3- severe dysfunction
    9- moderate dysfunction

you arrive at the residence of a 33-year-old woman who is experiencing a generalized (tonic-clonic) seizure. She has a small amount of vomitus draining from the side of her mouth. After protecting her from further injury, you should:

  • restrain her extremities to prevent her from injuring herself, suction her airway to remove the vomitus, and assist her ventulations with a bag-mask device
  • maintain her airway with manual head positioning, suction her airway to remove the vomitus, insert a nasopharyngeal airway, and administer high-flow oxygen
  • wait for the seizure to stop, manually open her airway, insert an oropharyngeal airway, and assess her oxygen saturation with a pulse oximeter
  • place a bite block in between her teeth, apply high-flow oxygen via a nonrebreathing mask, and consider inserting a nasopharyngeal airway
    maintain her airway with manual head positioning, suction her airway to remove the vomitus, insert a nasopharyngeal airway, and administer high-flow oxygen

A transient ischemic attack (TIA) occurs when:

  • a small clot in a cerebral artery causes temporary symtoms
  • a small cerebral artery ruptures and causes minimal damage
  • medications are given to dissolve a cerebral blood clot
  • signs and symptoms resolve spontaneously within 48 hours
    a small clot in a cerebral artery causes temporary symtoms

Which of the following is a metabolic cause of a seizure?

  • Massive stroke
  • Brain tumor
  • Poisoning
  • Head trauma
    Poisoning

You arrive at a local grocery store approximately 5 minutes after a 21-year-old female stopped seizing. She is confused and disoriented; she keeps asking you what happened and tells you that she is thirsty. Her brother, who witnessed the seizure, tells you that she takes phenytoin (Dilantin) for her seizures, but has not taken it in a few days. He also tells you that she has diabetes. In addition to applying high-flow oxygen, you should:

  • administer one tube of oral glucose and prepare for immediate transport.
  • place her in the recovery position and transport her with lights and siren.
  • monitor her airway and breathing status and assess her blood glucose level.
  • give her small cups of water to drink and observe for further seizure activity.
    monitor her airway and breathing status and assess her blood glucose level

Individuals with chronic alcoholism are predisposed to intracranial bleeding and hypoglycemia secondaryto abnormalities in the:

  • liver
  • pancreas
  • kidneys
  • brain
    liver

Febrile seizures:

  • are usually benign but should be evaluated
  • occur when a child’s fever rises slowly
  • often result in permanent brain damage
  • are also referred to as absence seizures
    are usually benign but should be evaluated

You are caring for a 70-year-old female with signs and symptoms of an acute stroke. She is conscious, has secretions in her mouth, is breathing at a normal rate with adequate depth, and has an oxygen saturation of 96%. You should:

  • suction her oropharynx and transport immediately
  • assist her ventilations with a bag-valve mask
  • administer one tube of oral glucose and transport
  • insert an oral airway, apply oxygen, and transport
    suction her oropharynx and transport immediately

A patient whose speech is slurred and difficult to understand is experiencing:

  • dysphagia
  • aphasia
  • dysarthria
  • paraplegia
    dysarthria

The spinal cord exits the cranium through the:

  • cauda equina
  • foramen magnum
  • foramen lamina
  • vertebral foramen
    foramen magnum

Which of the following symptoms would lead the EMT to believe that a patient’s headache is caused by sinus congestion?

  • There is associated neck stiffness
  • The pain is worse when bending over
  • There is numbness in the extremities
  • The headache began suddenly
    The pain is worse when bending over

When transporting a stable patient with a paralyzed extremity, place the patient in a:

  • supine position with the legs elevated 6 feet to 12 feet
  • sitting position with the head at a 45• to 90• angle
  • recumbent position with the paralyzed side up
  • recumbent position with the paralyzed side down
    recumbent position with the paralyzed side down

When assessing for arm drift of a patient with a suspected stroke, you should:

  • ask the patient to hold his or her arms up with the palms down
  • ask the patient to close his or her eyes during the assessment
  • observe movement of the arms for approximately 2 minutes
  • expect to see one arm slowly drift down to the patient’s side
    ask the patient to close his or her eyes during the assessment

When caring for a patient with documented hypoglycemia, you should be MOST alert for:

  • a seizure
  • an acute stroke
  • a februle convulsion
  • reapiratory distress
    a seizure

A 30-year-old male experienced a generalized (tonic-clonic) seizure, which stopped before you arrived at the scene. The patient is conscious, is answering your questions appropriately, and refuses EMS transport. Which of the following would be the MOST compelling reason to disagree with his refusal of transport?

  • He is currently not prescribed any medications
  • He has experienced seizures since he was 20
  • His wife states that this was his “usual” seizure
  • His Glascow Coma Scale (GCS) score is 15
    He is currently not prescribed any medications

Which of the following conditions would be the LEAST likely to mimic the signs and symptoms of a stroke?

  • Hypoglycemia
  • Intracranial bleeding
  • Hypovolemia
  • A postictal state
    Hypovolemia

You are dispatched to a residence for a 66-year-old male who, according to family members, has suffered a massive stroke. Your primary assessment reveals that the patient is unresponsive, apneic, and pulseless. You should:

  • perform CPR for 5 minutes before applying the AED
  • assess the patient for a facial droop and hemiparesis
  • obtain a blood glucose sample to rule out hypoglycemia
  • initiate CPR and attach an AED as soon as possible
    initiate CPR and attach an AED as soon as possible

Which of the following conditions would MOST likely affect the entire brain?

  • Respiratory failure or cardiopulmonary arrest
  • Reduced blood supply to the left hemisphere
  • Ruptured cerebral artery in the occipital lobe
  • Blocked cerebral artery in the frontal lobe
    Respiratory failure or cardiopulmonary arrest

The MOST significant risk factor for a hemorrhagic stroke is:

  • heavy exertion
  • hypertension
  • diabetes mellitus
  • severe stress
    hypertension

Your patient opens his eyes, moans, and pulls away from you when you pinch his trapezius muscle. You should assign a Glasgow Coma Scale (GCS) score of:

  • 9
  • 8
  • 7
  • 6
    8

The three major parts of the brain are the:

  • cerebrum, cerebellum, and brain stem
  • midbrain, cerebellum, and spinal cord
  • cerebellum, medulla, and occiput
  • brain stem, midbrain, and spinal cord
    cerebrum, cerebellum, and brain stem

In cholecystitis, where is the patient likely to feel referred pain?

  • Right shoulder
  • Around the navel
  • Right upper quadrant
  • Left lower quadrant
    Right shoulder

You are assessing a patient reporting severe pain in his upper left and right abdominal quadrants radiating to the back. The patient reports that the pain got worse right after he ate lunch an hour ago. He also has nausea and vomiting, abdominal distention, and tenderness. Based on these signs and symptoms, which of the following conditions do you suspect?

  • Pancreatitis
  • Diverticulitis
  • Esophagitis
  • Appendicitis
    Pancreatitis

Rebound tenderness is a result of what?

  • GERD
  • Ileus
  • Esophageal varices
  • Peritoneal irritation
    Peritoneal irritation

Which of the following statements is true about acute renal failure?

  • Kidney tissue shrinks and function diminishes over a period of years
  • Often caused by diabetes or hypertension
  • Requires dialysis or a kidney transplant
  • Can be reversed with prompt diagnosis and treatment
    Can be reversed with prompt diagnosis and treatment

What is the medical term for blood in the vomit?

  • Uremia
  • Melena
  • Hematemesis
  • Cholecystitis
    Hematemesis

What is it called when a patient tenses his abdominal muscles during your assessment?

  • Guarding
  • Splinting
  • Tetany
  • Rebound tenderness
    Guarding

Which of the following does NOT produce upper intestinal bleeding?

  • Ulcerative colitis
  • Esophageal varices
  • Esophagitis
  • Mallory-Weiss tear
    Ulcerative colitis

You are assessing a 76-year-old man complaining of severe tearing radiating into his lower back. He is hypotensive and tachycardic. Which of the following should you suspect?

  • Gallstones
  • Cystitis
  • Diverticulitis
  • AAA
    AAA

When assessing a patient’s abdomen, what is the best position to place the patient in?

  • Tradelenburg
  • Supine
  • Fowler
  • Prone
    Supine

Which of the following is a sign of peptic ulcer disease?

  • Melena
  • Diarrhea
  • Cystitis
  • Vomiting
    Melena

A 29-year-old pregnant woman has had severe vomiting for the last 2 days. Today, she is vomiting large amounts of blood. Her skin is cool and pale and she is tachycardic. The EMT should suspect:

  • esophageal varices
  • esophagitis
  • acute pancreatitis
  • Mallory-Weiss tear
    Mallory-Weiss tear

Solid abdominal organs include the:

  • gallbladder and large intestine
  • stomach and small intestine
  • spleen, kidneys, and pancreas
  • urinary bladder, colon, and ureters
    spleen, kidneys, and pancreas

In contrast to the parietal peritoneum, the visceral peritoneum:

  • lines the walls of the abdominal cavity and is stimulated when the solid abdominal organs contract.
  • is less likely to become inflamed or infected because it lines the abdominal organs themselves.
  • is supplied by the same nerves from the spinal cord that supply the skin of the abdomen.
  • is supplied by nerves of the autonomic nervous system, which are less able to localize pain or sensation.
    is supplied by nerves of the autonomic nervous system, which are less able to localize pain or sensation.

A 30-year-old woman with a history of alcoholism presents with severe upper abdominal pain and is vomiting large amounts of bright red blood. Her skin is cool, pale, and clammy; her heart rate is 120 beats/min and weak; and her blood pressure is 70/50 mm Hg. Your MOST immediate action should be to:

  • protect her airway from aspiration.
  • keep her supine and keep her warm.
  • rapidly transport her to the hospital.
  • give her high-flow supplemental oxygen.
    protect her airway from aspiration.

An important aspect in the treatment of a patient with severe abdominal pain is to:

  • encourage the patient to remain in a supine position.
  • administer analgesic medications to alleviate pain.
  • give 100% oxygen only if signs of shock are present.
  • provide emotional support en route to the hospital.
    provide emotional support en route to the hospital.

Urinary tract infections are more common in __.

  • active adults
  • sedentary adults
  • women
  • men
  • women

Pain that may be perceived at a distant point on the surface of the body, such as the back or shoulder, is called:

  • visceral pain.
  • remote pain.
  • referred pain.
  • radiating pain.
    referred pain.

Which of the following statements regarding dialysis is correct?

  • Hemodialysis is effective but carries a high risk of peritonitis.
  • Acute hypertension is a common adverse effect of dialysis.
  • The purpose of dialysis is to help the kidneys retain salt and water.
  • Patients who miss a dialysis treatment often present with weakness.
    Patients who miss a dialysis treatment often present with weakness.

Functions of the liver include:

  • secretion of bile and filtration of toxic substances.
  • release of amylase, which breaks down starches into sugar.
  • production of hormones that regulate blood sugar levels.
  • absorption of nutrients and toxins.
    secretion of bile and filtration of toxic substances.

The parietal peritoneum lines the:

  • surface of the abdominal organs.
  • retroperitoneal space.
  • lungs and chest cavity.
  • walls of the abdominal cavity
    walls of the abdominal cavity

Which of the following is characteristic of peptic ulcer disease (PUD)?

  • Burning or pain in the stomach that subsides immediately after eating
  • The passage of bright red blood in the stool or coughing up blood
  • Symptom relief after taking nonsteroidal anti-inflammatory drugs
  • Sharp pain that is typically located in both lower abdominal quadrants
    Burning or pain in the stomach that subsides immediately after eating

A 47-year-old male presents with severe abdominal pain of 3 hours’ duration. His abdomen is distended and guarded. Your MOST important consideration for this patient should be to:

  • be alert for signs and symptoms of shock.
  • transport him in a supine position.
  • assess his blood pressure to determine perfusion adequacy.
  • determine the exact location and cause of his pain.
    be alert for signs and symptoms of shock.

A strangulated hernia is one that:

  • spontaneously reduces without any surgical intervention.
  • is reducible if surgical intervention occurs within 2 hours.
  • can be pushed back into the body cavity to which it belongs.
  • loses its blood supply due to compression by local tissues.
    loses its blood supply due to compression by local tissues.

A 60-year-old female presents with a tearing sensation in her lower back. Her skin is sweaty and she is tachycardic. The EMT should suspect:

  • kidney stones.
  • aortic aneurysm.
  • strangulated hernia.
  • acute pancreatitis.
    aortic aneurysm.

Most patients with abdominal pain prefer to:

  • lie in a supine position with their knees in a flexed position.
  • lie on their side with their knees drawn into the abdomen.
  • sit in a semi-Fowler position with their knees slightly bent.
  • sit fully upright because it helps relax the abdominal muscles.
    lie on their side with their knees drawn into the abdomen.

Peritonitis may result in shock because:

  • fluid shifts from the bloodstream into body tissues.
  • severe pain causes systemic dilation of the vasculature.
  • intra-abdominal hemorrhage is typically present.
  • abdominal distention impairs cardiac contractions.
    fluid shifts from the bloodstream into body tissues.

When assessing a patient with abdominal pain, you should:

  • observe for abdominal guarding, which is characterized by sudden relaxation of the abdominal muscles when palpated.
  • ask the patient to point to the area of pain or tenderness and assess for rebound tenderness over that specific area.
  • palpate the abdomen in a clockwise direction, beginning with the quadrant after the one the patient indicates is painful.
  • visually assess the painful area of the abdomen, but avoid palpation because this could worsen his or her condition.
    palpate the abdomen in a clockwise direction, beginning with the quadrant after the one the patient indicates is painful.

The MOST important treatment for a patient with severe abdominal pain and signs of shock is:

  • giving oral fluids to maintain perfusion.
  • positioning the patient on his or her side.
  • transporting the patient without delay.
  • administering high-flow oxygen.
    transporting the patient without delay.

Which of the following organs assists in the filtration of blood, serves as a blood reservoir, and produces antibodies?

  • Kidney
  • Liver
  • Spleen
  • Pancreas
    Spleen

Injury to a hollow abdominal organ would MOST likely result in:

  • leakage of contents into the abdominal cavity.
  • impairment in the blood’s clotting abilities.
  • profound shock due to severe internal bleeding.
  • pain secondary to blood in the peritoneum.
    leakage of contents into the abdominal cavity.

Diabetes is a metabolic disorder in which the body’s ability to metabolize what is impaired?

  • Fats
  • Protein
  • Glucose
  • Electrolytes
    Glucose

What is the function of insulin in the body?

  • It regulates oxygen flow to the cells.
  • It regulates chemical production in the endocrine glands.
  • It enables glucose in the blood to enter cells.
  • It prevents glucose from oversaturating cells.
    It enables glucose in the blood to enter cells.

What is released when the concentration of glucose drops in the blood?

  • Epinephrine
  • Glucagon
  • Dopamine
  • Insulin
    Glucagon

Which of the following is the most common metabolic disease of childhood?

  • Diabetes mellitus type 2
  • Ketoacidosis
  • Hyperosmolar hyperglycemic nonketotic syndrome
  • Diabetes mellitus type 1
    Diabetes mellitus type 1

Which of the following organs can rapidly sustain permanent damage when the body’s glucose level is too low?

  • Brain
  • Liver
  • Heart
  • Kidney
    Brain

Which of the following is a normal glucose level in an adult?

  • 200 mg/dL
  • 80 mg/dL
  • 140 mg/dL
  • 20 mg/dL
    80 mg/dL

You are treating a 36-year-old patient displaying the following signs and symptoms: rapid, shallow respirations; pale, cool, clammy skin; tachycardia; weakness on the left side of the body; and confusion and irritability. What should you suspect?

  • Symptomatic hyperglycemia
  • DKA
  • Symptomatic hypoglycemia
  • Vasoocclusive crisis
    Symptomatic hypoglycemia

The condition in which blood clots as a result of an abnormality of the system of coagulation is called what?

  • Hemophilia
  • Thrombophilia
  • Sickle cell disease
  • Thrombosis
    Thrombophilia

Which of the following is classified as an autoimmune disorder?

  • Sickle cell disease
  • Acidosis
  • Type 1 diabetes
  • type 2 diabetes
    Type 1 diabetes

In which condition are you most likely to encounter a vasoocclusive crisis?

  • Hemophilia
  • Sickle cell disease
  • Thrombophilia
  • Symptomatic hypoglycemia
    Sickle cell disease

Which of the following conditions is the diabetic patient at an increased risk of developing?

  • Hepatitis B
  • Alcoholism
  • Blindness
  • Depression
    Blindness

A patient with hypoglycemia will often present with which of the following signs/symptoms?

  • Pale, cool, and clammy skin
  • Deep, rapid respirations
  • Hypertension
  • Warm, red, and dry skin
    Pale, cool, and clammy skin

Which of the following statements regarding glucose is correct?

  • Most cells will function normally without glucose.
  • The brain requires insulin to allow glucose to enter the cells.
  • The brain requires glucose as much as it requires oxygen.
  • Blood glucose levels decrease in the absence of insulin.
    The brain requires glucose as much as it requires oxygen.

A 66-year-old woman experienced a sudden onset of difficulty breathing. She has a history of type 2 diabetes and deep vein thrombosis (DVT). On the basis of her medical history, which of the following should the EMT suspect?

  • Congestive heart failure
  • Diabetic ketoacidosis
  • Pulmonary embolism
  • Severe hypoglycemia
    Pulmonary embolism

Proper procedure for administering oral glucose to a patient includes all of the following, EXCEPT:

  • checking the medication’s expiration date.
  • requesting permission from medical control.
  • ensuring the absence of a gag reflex.
  • assessing the patient’s mental status.
    ensuring the absence of a gag reflex.

You are treating a 40-year-old male with a documented blood sugar reading of 480 mg/dL. The patient is semiconscious and breathing shallowly, and is receiving assisted ventilation from your partner. You should recognize that definitive treatment for this patient includes:

  • dextrose.
  • oxygen.
  • insulin.
  • glucagon.
    insulin.

The main function of the endocrine system is to _.

  • maintain homeostasis
  • regulate blood flow
  • produce glandular secretions
  • regulate glucose and calcium
    maintain homeostasis

A patient with an altered mental status; high blood glucose levels; and deep, rapid breathing may have a condition known as __.

  • hyperosmolar hyperglycemic nonketotic coma
  • hyperglycemic crisis
  • hypoglycemic crisis
  • diabetic ketoacidosis
    diabetic ketoacidosis

Symptomatic hypoglycemia will MOST likely develop if a patient:

  • takes too much of his or her prescribed insulin.
  • eats a regular meal followed by mild exertion.
  • misses one or two prescribed insulin injections.
  • markedly overeats and misses an insulin dose.
    takes too much of his or her prescribed insulin.

A 37-year-old female with a history of diabetes presents with excessive urination and weakness of 2 days’ duration. Her blood glucose level reads 320 mg/dL. If this patient’s condition is not promptly treated, she will MOST likely develop:

  • acidosis and dehydration.
  • irreversible renal failure.
  • severe insulin shock.
  • hypoxia and overhydration.
    irreversible renal failure.

The normal blood glucose level is between:

  • 30 and 150 mg/dL.
  • 60 and 80 mg/dL.
  • 160 and 200 mg/dL.
  • 80 and 120 mg/dL.
    80 and 120 mg/dL.

During your assessment of a 19-year-old male, you are told that he is being treated with factor VIII. This indicates that:

  • he has hemophilia A.
  • he has a thrombosis.
  • he has thrombophilia.
  • his blood clots too quickly.
    he has hemophilia A.

Excessive eating caused by cellular “hunger” is called:

  • polyphagia.
  • polydipsia.
  • dysphasia.
  • dyspepsia.
    polyphagia.

A 75-year-old male with type 1 diabetes presents with chest pain and a general feeling of weakness. He tells you that he took his insulin today and ate a regular meal approximately 2 hours ago. You should treat this patient as though he is experiencing:

  • hyperglycemia.
  • hypoglycemia.
  • an acute stroke.
  • a heart attack.
    a heart attack.

Which of the following statements regarding sickle cell disease is correct?

  • In sickle cell disease, the red blood cells are abnormally shaped and are less able to carry oxygen.
  • Sickle cell disease is an inherited blood disorder that causes the blood to clot too quickly.
  • Because of their abnormal shape, red blood cells in patients with sickle cell disease are less apt to lodge in a blood vessel.
  • The red blood cells of patients with sickle cell disease are round and contain hemoglobin.
    In sickle cell disease, the red blood cells are abnormally shaped and are less able to carry oxygen.

A man finds his 59-year-old wife unconscious on the couch. He states that she takes medications for type 2 diabetes. He further tells you that his wife has been ill recently and has not eaten for the past 24 hours. Your assessment reveals that the patient is unresponsive. You should:

  • open and maintain her airway and assess breathing.
  • administer 100% oxygen via a nonrebreathing mask.
  • assess for the presence of a medical identification tag.
  • administer oral glucose between her cheek and gum.
    open and maintain her airway and assess breathing.

In contrast to type 1 diabetes, type 2 diabetes:

  • occurs when antibodies attack insulin-producing cells.
  • is caused by resistance to insulin at the cellular level.
  • is caused by a complete lack of insulin in the body.
  • is commonly diagnosed in children and young adults.
    is caused by resistance to insulin at the cellular level.

The EMT should assess for hypoglycemia in small children with a severe illness or injury because:

  • illness or injury causes the pancreas to produce less insulin.
  • a child’s cells do not uptake glucose as rapidly as adults’ do.
  • children overproduce insulin during severe illness or injury.
  • children cannot store excess glucose as effectively as adults.
    children cannot store excess glucose as effectively as adults.

The two main types of cells contained in blood are called _.

  • platelets and plasma
  • transport and clotting
  • erythrocytes and leukocytes
  • hemoglobin A and S
    erythrocytes and leukocytes

Classic signs and symptoms of hypoglycemia include:

  • cool, clammy skin; weakness; tachycardia; and rapid respirations.
  • warm, dry skin; irritability; bradycardia; and rapid respirations.
  • warm, dry skin; hunger; abdominal pain; and deep, slow respirations.
  • cold, clammy skin; bradycardia; hunger; and deep, rapid respirations.
    cool, clammy skin; weakness; tachycardia; and rapid respirations.

You are responding to a call where an 8-year-old has been stung by a wasp. His skin is pale with patches of raised red spots on his hands, arms, and face. These spots are most likely what?

  • A fungal infection
  • Acne
  • Angioedema
  • Urticaria
    Urticaria

Why is epinephrine used for anaphylaxis?

  • It stops the allergic reaction from progressing.
  • It reverses bronchoconstriction and vasodilation.
  • It slows tachycardia.
  • It decreases blood flow.
    It reverses bronchoconstriction and vasodilation.

What is an allergen?

  • A substance that causes an allergic reaction
  • A chemical substance released by the immune system in allergic reactions that is responsible for many of the symptoms of anaphylaxis, such as vasodilation
  • A poison or harmful substance
  • A raised, swollen, well-defined area on the skin resulting from an insect bite or allergic reaction
    A substance that causes an allergic reaction

An extreme, life-threatening, systemic reaction that may include shock and respiratory failure is known as what?

  • Urticaria
  • Allergic reaction
  • Anaphylaxis
  • Angioedema
    Anaphylaxis

You are treating a child with a bee sting. The stinger is still embedded in the child’s skin. What should you do?

  • Scrape the skin with the edge of a stiff object.
  • Leave the stinger in place.
  • Remove the stinger with tweezers.
  • Squeeze the area around the stinger.
    Scrape the skin with the edge of a stiff object.

You have responded to a scene where the 33-year-old male patient is in obvious respiratory distress, breathing rapidly with audible wheezing. The patient’s skin is flushed, red, and covered in hives. When you attempt to question the patient, you find he can speak in only two- to three-word sentences. Respirations are 28 breaths/min and labored. Pulse is 120 beats/min. Skin is pale and cool with widespread urticaria and angioedema of the lips. Blood pressure is 88/60 mm Hg, and SpO2 is 88%. Which of the following interventions should you administer first?

  • Give the patient a nonsteroidal anti-inflammatory drug (NSAID).
  • Administer epinephrine.
  • Give the patient an antihistamine.
  • Provide high-concentration oxygen.
    Provide high-concentration oxygen.

When assessing a patient’s breathing, you hear wheezing in all lung fields. What are you hearing?

  • Crackling, moist breath sounds
  • A coarse, low-pitched breath sound heard in patients with chronic mucus in the upper airways
  • A high-pitched whistling sound caused by bronchoconstriction
  • A high-pitched noise heard primarily on inspiration
    A high-pitched whistling sound caused by bronchoconstriction

What is a wheal?

  • Swelling of the tongue
  • A poison or harmful substance
  • A raised, swollen, well-defined area on the skin
  • Widespread areas of generalized itching and/or burning that appear as multiple raised areas on the skin
    A raised, swollen, well-defined area on the skin

Which of the following patients has the most significant chance of having a contraindication to epinephrine?

  • A teenager with no history of allergies
  • A 6-year-old with significant peanut allergies
  • An 80-year-old male with a history of acute coronary syndrome
  • A 47-year-old female with bronchospasms
    An 80-year-old male with a history of acute coronary syndrome

What is the common dose of epinephrine contained in an adult EpiPen?

  • 3 mg
  • 0.3 mg
  • 0.15 mg
  • 1 mg
    0.3 mg

The stinger from a honeybee should be:

  • squeezed with tweezers and removed.
  • scraped away from the skin.
  • left in place and covered.
  • irrigated with copious amounts of water.
    scraped away from the skin.

A 37-year-old male is having a severe allergic reaction to penicillin. He does not have an epinephrine auto-injector and your protocols do not allow you to carry epinephrine on the ambulance. How should you proceed with the treatment of this patient?

  • Administer oxygen, transport at once, and request a paramedic intercept.
  • Remain at the scene with the patient and request a paramedic ambulance.
  • Ask the patient if he has any diphenhydramine tablets that you can administer.
  • Quickly determine if there are any bystanders who may carry epinephrine.
    Administer oxygen, transport at once, and request a paramedic intercept.

When a foreign substance invades the body, the body will __.

  • protect itself
  • go on alert
  • attempt to inactivate the foreign substance
  • All of these answers are correct.
    All of these answers are correct.

Epinephrine stimulates the __ response, increasing blood pressure and relieving bronchospasm.

  • parasympathetic
  • respiratory
  • sympathetic
  • cardiac
    sympathetic

The effects of epinephrine are typically observed within _ following administration.

  • 30 minutes
  • 1 minute
  • 1 hour
  • 30 seconds
    1 minute

The adult epinephrine auto-injector delivers _ mg of epinephrine, and the infant-child auto-injector delivers _ mg.

  • 0.1; 0.01
  • 0.03; 0.3
  • 0.01; 0.1
  • 0.3; 0.15
    0.3; 0.15

Urticaria is the medical term for:

  • burning.
  • hives.
  • swelling.
  • a wheal.
    hives.

Which of the following would MOST likely provide clues regarding the source of a patient’s allergic reaction?

  • The patient’s general physical appearance
  • The time of year in which the exposure occurred
  • The patient’s family history
  • The environment in which the patient is found
    The environment in which the patient is found

Anaphylaxis caused by stinging insects is typically an allergic reaction to __ rather than the bite or sting itself.

  • injected poison
  • irritating toxin
  • deadly venom
  • All of these answers are correct.
    All of these answers are correct.

Which of the following signs/symptoms are indicative of respiratory involvement of an allergic reaction?

  • Flushed, itching, or burning skin
  • A sense of impending doom
  • Tightness in the chest or throat
  • All of these answers are correct.
    Tightness in the chest or throat

Which of the following medications blocks the release of histamines?

  • Diphenhydramine (Benadryl)
  • Epinephrine (Adrenalin)
  • Albuterol (Ventolin)
  • Acetaminophen (Tylenol)
    Diphenhydramine (Benadryl)

After administering 0.3 mg of epinephrine via auto-injector to a 22-year-old woman with an allergic reaction, you note improvement in her breathing and dissipation of her hives. However, she is still anxious and tachycardic. You should:

  • consider administering 0.15 mg of epinephrine to completely resolve her allergic reaction.
  • contact medical control and obtain authorization to administer another 0.3 mg of epinephrine.
  • transport her rapidly, as it is obvious that she is having a severe reaction to the epinephrine.
  • monitor her closely but recall that anxiety and tachycardia are side effects of epinephrine.
    monitor her closely but recall that anxiety and tachycardia are side effects of epinephrine.

Anaphylaxis is MOST accurately defined as a(n):

  • allergic reaction that causes bronchodilation and vasoconstriction.
  • moderate allergic reaction that primarily affects the vasculature.
  • severe allergic reaction that typically resolves without treatment.
  • extreme allergic reaction that may affect multiple body systems.
    extreme allergic reaction that may affect multiple body systems.

When an allergic reaction proceeds to life-threatening anaphylaxis, it will usually do so __.

  • quickly, within 30 minutes.
  • slowly, over more than 30 minutes.
  • slowly, over several hours.
  • quickly, within 30 seconds.
    quickly, within 30 minutes.

A 19-year-old female was stung multiple times on the legs by fire ants. She states that she is allergic to fire ants, but does not carry her own epinephrine. The patient is conscious and alert and complains of pain to the area of the bites. Her blood pressure is 122/70 mm Hg, her pulse is 100 beats/min and strong, and her respirations are 18 breaths/min and unlabored. You should:

  • administer oxygen and transport her to the hospital.
  • advise her to see her physician as soon as possible.
  • request a paramedic unit to administer epinephrine.
  • position her legs well above the level of her heart.
    administer oxygen and transport her to the hospital.

At the site of the injury, signs and symptoms of an animal or insect bite or sting may include pain, localized heat, and a wheal, which appears as __.

  • a series of small, raised pustules
  • an uncomfortable depression in the skin
  • a raised, well-defined area of the skin
  • a large area of red, blotchy skin
    a raised, well-defined area of the skin

The goal of invaders such as bacteria and viruses is to __.

  • spread disease and destruction
  • cause an immune response
  • kill their human host
  • use a human body as a home
    use a human body as a home

Patients may experience allergic reactions to which of the following substances?

  • Plant material
  • Food and medications
  • All of these answers are correct.
  • Insect or animal bites and stings
    All of these answers are correct.

Care for a victim of an immunologic emergency who is severely hypotensive should include which of the following?

  • Routine spinal immobilization due to the potential for traumatic injury
  • Position the patient’s airway and initiate positive-pressure ventilations.
  • Initiate basic life support measures, including the use of an automated external defibrillator, if necessary.
  • Apply high-flow oxygen therapy, place the patient in a shock position, and help maintain the patient’s body temperature.
    Apply high-flow oxygen therapy, place the patient in a shock position, and help maintain the patient’s body temperature.

Which of the following sounds indicates swelling of the upper airway?

  • Wheezing
  • Rales
  • Stridor
  • Rhonchi
    Stridor

You are assessing a 28-year-old patient and you notice he has pinpoint pupils. You suspect a drug overdose. What drug is the likely cause of this phenomenon?

  • Methamphetamine
  • Alcohol
  • Cocaine
  • Heroin
    Heroin

Which of the following might make it difficult for you to establish an IV access in a patient with a drug abuse problem?

  • Hypotension
  • Tachycardia
  • Nausea and/or vomiting
  • Venous scarring
    Venous scarring

You are assessing a patient who presents with slurred speech, hypoventilation, and hypotension. Which of the following might the patient have taken?

  • Diazepam
  • Pilocarpine
  • Diphenhydramine
  • Albuterol
    Diazepam

What is the most common route of poisoning?

  • Injection
  • Absorption
  • Ingestion
  • Inhalation
    Ingestion

What should you give a patient displaying respiratory depression and hypotension?

  • Phenobarbital
  • Codeine
  • Activated charcoal
  • Naloxone
    Naloxone

Which of the following is a CNS stimulant?

  • Sedative-hypnotics
  • Alcohol
  • Sympathomimetics
  • Opiates
    Sympathomimetics

Antihistamines are an example of which of the following?

  • Sympathomimetic
  • Cholinergic agent
  • Anticholinergic agent
  • Synthetic cathinone
    Anticholinergic agent

You can use the mnemonic DUMBELS to remember the signs and symptoms of which of the following?

  • Acetaminophen poisoning
  • Cholinergic agents
  • Food poisoning
  • Anticholinergic agents
    Cholinergic agents

Which of the following types of food poisoning results from an organism?

  • Staphylococcus
  • Botulism
  • Salmonella
  • Datura
    Salmonella

For which of the following would you use a DuoDote Auto-Injector?

  • Carbon monoxide poisoning
  • Acetaminophen overdose
  • Sarin gas attack
  • Morphine overdose
    Sarin gas attack

An overdose of acetaminophen, the active ingredient in Tylenol, will MOST likely cause:

  • CNS depression.
  • kidney failure.
  • gastric ulcers.
  • liver failure.
    liver failure.

Hypotension, hypoventilation, and pinpoint pupils would be expected following an overdose of:

  • crack cocaine.
  • oxycodone (Percocet).
  • ecstasy.
  • amphetamine sulfate (Benzedrine).
    oxycodone (Percocet).

A person who routinely misuses a substance and requires increasing amounts to achieve the same effect is experiencing:

  • addiction.
  • tolerance.
  • dependence.
  • withdrawal.
    tolerance.

Heroin is an example of a(n):

  • hypnotic.
  • cholinergic.
  • sympathomimetic.
  • opioid.
    opioid.

If the victim of a toxicologic emergency vomits, an EMT should _.

  • not approach the vomitus.
  • have the patient enclose the vomitus in a container and take it to the emergency department.
  • safely dispose of the vomitus as soon as possible and decontaminate the vehicle as necessary.
  • use appropriate personal protective equipment and examine the vomitus for pill fragments or other clues for patient care.
    use appropriate personal protective equipment and examine the vomitus for pill fragments or other clues for patient care.

When caring for a known alcoholic patient with severe trauma to the chest and abdomen, you should be concerned that:

  • signs and symptoms of shock may be masked by the stimulant effects produced by alcohol.
  • long bone fractures are likely because chronic alcohol consumption weakens the structure of the bones.
  • internal bleeding may be profuse because prolonged alcohol use may impair the blood’s ability to clot.
  • delirium tremens (DTs) are commonly induced by physical trauma and can lead to life-threatening seizures.
    internal bleeding may be profuse because prolonged alcohol use may impair the blood’s ability to clot.

You and your paramedic partner are caring for a patient who ingested codeine, acetaminophen (Tylenol), and hydrocodone (Vicodin). The patient is unresponsive, his breathing is slow and shallow, and his pulse is slow and weak. Treatment for this patient should include:

  • oxygen via a nasal cannula and atropine sulfate.
  • assisted ventilation and naloxone (Narcan).
  • assisted ventilation and flumazenil (Romazicon).
  • high-flow oxygen via a nonrebreathing mask.
    assisted ventilation and naloxone (Narcan).

As you enter the residence of a patient who has possibly overdosed, you should:

  • be alert for personal hazards.
  • observe the scene for drug bottles.
  • quickly gain access to the patient.
  • look for drug paraphernalia.
    be alert for personal hazards.

How do poisons typically act to harm the body?

  • By changing the normal metabolism of cells or by destroying them
  • By causing a slowing of nearly all bodily functions
  • By interfering with normal neurologic function
  • By causing burns and damage to either external or internal organs
    By changing the normal metabolism of cells or by destroying them

Activated charcoal is given to patients who have ingested certain substances because it:

  • binds to the substance and prevents absorption.
  • induces vomiting and empties the stomach.
  • is a direct antidote for many toxic substances.
  • decreases absorption of poisons into the lungs.
    binds to the substance and prevents absorption.

Most poisonings occur via the __ route.

  • absorption
  • ingestion
  • inhalation
  • injection
    ingestion

Which of the following statements regarding the Salmonella bacterium is correct?

  • Refrigeration of food will prevent salmonellosis.
  • The Salmonella bacterium itself causes food poisoning.
  • The Salmonella bacterium produces toxins that cause food poisoning.
  • Symptoms of salmonellosis appear within 12 hours.
    The Salmonella bacterium itself causes food poisoning.

Which of the following questions would be LEAST pertinent during the initial questioning of a patient who ingested a substance?

  • What type of substance was taken?
  • How much of the substance was taken?
  • How long ago was the substance taken?
  • Why was the substance ingested?
    Why was the substance ingested?

Which of the following sets of vital signs would the EMT MOST likely encounter in a patient with acute cocaine overdose?

  • BP, 180/100 mm Hg; pulse, 50 beats/min
  • BP, 200/100 mm Hg; pulse, 150 beats/min
  • BP, 60/40 mm Hg; pulse, 140 beats/min
  • BP, 190/90 mm Hg; pulse, 40 beats/min
    BP, 200/100 mm Hg; pulse, 150 beats/min

Victims of inhaled poisoning will require which of the following?

  • Transport to an emergency department for evaluation
  • An oropharyngeal or nasopharyngeal device to maintain an airway
  • Suctioning to clear the airway
  • Ventilation with a bag-valve mask
    Transport to an emergency department for evaluation

The recommended treatment for absorbed or contact poisons includes _.

  • collecting and transporting the poisonous substance to the emergency department for identification
  • neutralizing the poisonous substance with a chemical
  • transporting the patient to the emergency
    department with the poisonous substance in place
  • safely removing or diluting the poisonous substance
    safely removing or diluting the poisonous substance

A 49-year-old male presents with confusion, sweating, and visual hallucinations. The patient’s wife tells you that he is a heavy drinker and she thinks he had a seizure shortly before your arrival. This patient is MOST likely experiencing:

  • delirium tremens.
  • alcohol intoxication.
  • acute schizophrenia.
  • acute hypovolemia.
    delirium tremens.

In an apparent suicide attempt, a 19-year-old female ingested a full bottle of amitriptyline (Elavil). At present, she is conscious and alert and states that she swallowed the pills approximately 30 minutes earlier. Her blood pressure is 90/50 mm Hg, her pulse is 140 beats/min and irregular, and her respirations are 22 breaths/min with adequate depth. When transporting this patient, you should be MOST alert for:

  • seizures and cardiac arrhythmias.
  • an increase in her blood pressure.
  • a sudden outburst of violence.
  • acute respiratory depression.
    seizures and cardiac arrhythmias.

You receive a call to a residence where a man found his wife unresponsive on the couch. The patient’s respiratory rate is 8 breaths/min, her breathing is shallow, her heart rate is 40 beats/min, and her pulse is weak. The husband hands you an empty bottle of hydrocodone (Vicodin), which was refilled the day before. You should:

  • perform a rapid head-to-toe exam.
  • contact the poison control center.
  • ventilate her with a BVM.
  • apply oxygen via a nonrebreathing mask.
    ventilate her with a BVM.

Of the four avenues of poisoning, generally _ is the most worrisome in terms of treatment to the EMS provider.

  • absorption
  • ingestion
  • injection
  • inhalation
    injection

What should the EMT do immediately after physically restraining a violent patient?

  • Advise the patient why restraint was needed.
  • Inform medical control of the situation
  • Document the time the restraints were applied
  • Reassess the patient’s airway and breathing.
    Reassess the patient’s airway and breathing.

Where should you position yourself when beginning the primary assessment of a patient displaying a psychiatric issue?

  • At a distance
  • Directly in front of the patient
  • At the patient’s dominant side
  • At the patient’s nondominant side
    At a distance

When assessing a patient with a behavioral issue, you need to ask questions about three contributors: proper CNS functioning, whether the patient took hallucinogenic or other drugs or alcohol, and what else?

  • Significant life changes
  • Personal history of physical trauma
  • Family history of psychotic disorders
  • Coping mechanisms
    Significant life changes

Why should a patient not be transported in a prone position with his/her hands tied behind the back?

  • It is embarrassing for the patient and therefore contraindicated.
  • Positional asphyxia could occur.
  • It is easier for the patient to bite you from this position.
  • The restraints can cause circulatory problems in the wrists and ankles.
    Positional asphyxia could occur.

Which of the following is a sign or symptom of PTSD?

  • Bradycardia
  • Cyanosis
  • Lack of emotion
  • Hypervigilance
    Hypervigilance

How is a behavioral crisis defined?

  • A condition in which a person exhibits agitated behavior combined with disorientation, hallucinations, or delusions
  • A violent act intended to inflict harm
  • A mental disorder characterized by the loss of contact with reality
  • Any reaction to events that interferes with the activities of daily living
    Any reaction to events that interferes with the activities of daily living

Which of the following behavioral crises is most likely to lead to suicide?

  • Depression
  • PTSD
  • Excited delirium
  • Psychosis
    Depression

Which of the following statements is true regarding the returning combat veteran?

  • Guilt, shame, paranoia, and hostility are rare in combat veterans.
  • Physical restraints are an important tool to calm the veteran with PTSD.
  • Suicide is rare in this population.
  • Diesel fumes can trigger PTSD in combat veterans.
    Diesel fumes can trigger PTSD in combat veterans.

As an EMT, what is your main goal in a psychiatric emergency?

  • Defusing/controlling the situation and providing transport
  • Showing empathy
  • Referring the patient to appropriate mental health resources
  • Identifying the underlying cause of the behavioral emergency
    Defusing/controlling the situation and providing transport

A patient with which of the following conditions is likely to have a flashback?

  • PTSD
  • Schizophrenia
  • Excited delirium
  • Depression
    PTSD

__ is what you can see of a person’s response to the environment.

  • Neurosis
  • Psychosis
  • Affect
  • Behavior
    Behavior

A 22-year-old male with a history of clinical depression called 9-1-1 and stated that he has attempted to kill himself. Your unit and law enforcement officers arrive at the scene simultaneously. You find the patient lying supine on the living room floor. He is unresponsive and cyanotic. An empty bottle of hydromorphone (Dilaudid) is found on an adjacent table. You should:

  • provide care after determining what Dilaudid is used for.
  • open the patient’s airway and assess his respirations.
  • wait for the police to examine him before providing care.
  • ask the police to handcuff the patient for safety purposes.
    open the patient’s airway and assess his respirations.

A 66-year-old male presents with bizarre behavior. His daughter states that he did not seem to recognize her and was very rude to her. The patient is conscious, and has a patent airway and adequate breathing. You should:

  • ask the daughter how her father normally behaves.
  • conclude that the patient has Alzheimer disease.
  • advise the patient that his behavior is unacceptable.
  • carefully restrain the patient and transport at once.
    ask the daughter how her father normally behaves.

A functional disorder is a:

  • physiological disorder caused by a structural abnormality.
  • physiological disorder in the absence of a structural abnormality.
  • psychological disorder caused by a structural abnormality.
  • psychological disorder in the absence of a structural abnormality.
    physiological disorder in the absence of a structural abnormality.

A patient with a history of schizophrenia called EMS because he was experiencing abdominal pain. When law enforcement arrived, the patient became violent, necessitating the placement of handcuffs. When you assess the patient, he tells you that killing someone will make his abdominal pain go away. His vital signs are stable. How should you manage this situation?

  • Transport the patient in a prone position on the stretcher
  • Request a police officer to accompany you in the ambulance
  • Request that a police officer transport him to the hospital
  • Refuse to transport the patient because of his homicidal threat
    Request a police officer to accompany you in the ambulance

A technique used to gain insight into a patient’s thinking, which involves repeating in question form what the patient has said, is called:

  • intuitive listening.
  • passive listening.
  • reflective listening.
  • active listening.
    reflective listening.

General guidelines for managing a patient with a behavioral emergency include:

  • being prepared to spend extra time with the patient.
  • allowing the patient to be alone if he or she wishes.
  • placing the patient between yourself and an exit.
  • firmly identifying yourself as an EMS provider.
    being prepared to spend extra time with the patient.

Immediately after physically restraining a violent patient, the EMT should:

  • reassess the patient’s airway and breathing.
  • document the time the restraints were applied.
  • advise the patient why restraint was needed.
  • inform medical control of the situation.
    reassess the patient’s airway and breathing.

In addition to ensuring his or her own safety, the EMT’s responsibility when caring for a patient with a behavioral emergency is to:

  • determine the underlying cause of the problem and offer advice.
  • diagnose the patient’s problem and provide definitive treatment.
  • diffuse and control the situation and safely transport the patient.
  • transport the patient directly to a specialized psychiatric facility.
    diffuse and control the situation and safely transport the patient.

In contrast to a behavioral crisis, a psychiatric emergency occurs when a person:

  • experiences a sudden attack of panic secondary to a stressful situation.
  • becomes agitated or violent and is a threat to him- or herself or others.
  • exhibits impaired functioning due to a chemical or genetic disturbance.
  • experiences feelings of sadness and despair for longer than a month.
    becomes agitated or violent and is a threat to him- or herself or others.

Organic brain syndrome is defined as:

  • a dysfunction of the brain caused by abnormal physical or physiological function.
  • a change in behavior or mental status secondary to decreased cerebral blood flow.
  • a disorder that cannot be traced to the abnormal structure or function of an organ.
  • bizarre behavior secondary to a chemical imbalance or disturbance in the brain.
    a dysfunction of the brain caused by abnormal physical or physiological function.

People at a higher risk for suicide include all of the following, EXCEPT:

  • children with parents addicted to alcohol.
  • those with a recent diagnosis of a serious illness.
  • substance abusers.
  • married males older than 30 years.
    married males older than 30 years.

The first step in assessing a patient with a behavioral emergency is to:

  • restrain the patient.
  • obtain proper consent.
  • ensure your safety.
  • take vital signs.
    ensure your safety.

The single most significant factor that contributes to suicide is:

  • depression.
  • advanced age.
  • a chronic illness.
  • drug abuse.
    depression.

When assessing a patient with a behavioral crisis, you should:

  • frisk the patient for the presence of weapons.
  • spend as little time with the patient as possible.
  • ask the police to handcuff the patient for safety.
  • be direct and clearly state your intentions.
    be direct and clearly state your intentions.

Which of the following conditions or factors would be the LEAST likely to result in a change in behavior?

  • Antihypertensive medications
  • Exposure to excess heat or cold
  • Low blood glucose levels
  • Inadequate blood flow to the brain
    Antihypertensive medications

Which of the following conditions or situations would MOST likely make excited delirium worse?

  • Dimly lit environment
  • Limited physical contact
  • Alcohol withdrawal
  • Elevated blood glucose
    Alcohol withdrawal

Which of the following statements regarding the physical examination of a patient with a behavioral problem is correct?

  • Performing a physical examination on a patient with a behavioral problem will likely cause him or her to become violent and should be avoided in the field.
  • The physical examination of a patient with a behavioral problem often yields crucial information and should always be performed, even if the patient is violent.
  • Although the physical exam can be difficult, the patient’s emotional state may be determined by noting facial expressions, pulse rate, and respirations.
  • It is only appropriate to perform a physical examination on a patient with a behavioral problem if he or she is unconscious or is being physically restrained.
    Although the physical exam can be difficult, the patient’s emotional state may be determined by noting facial expressions, pulse rate, and respirations.

You are assessing a conscious 55-year-old male with a sudden change in behavior. Which of the following clinical findings would be MOST suggestive of dysfunction of this patient’s central nervous system?

  • Excessive tearing or crying
  • Consistent eye contact
  • An irregular pulse
  • Rapid eye movement
    Rapid eye movement

What is menarche?

  • The cycle in which the ovum is released
  • The area of tissue between the vagina and the anus
  • The onset of menstruation, which usually occurs between ages 11 and 16 years
  • The end of menstrual activity, which usually occurs in a woman’s 50s
    The onset of menstruation, which usually occurs between ages 11 and 16 years

Which of the following organs or structures controls the process of ovulation and menstruation?

  • Fallopian tubes
  • Vagina
  • Cervix
  • Ovaries
    Ovaries

You are assessing a female patient who tells you she feels blood coming from her vaginal area. She gives you permission to look and you observe moderate bleeding from the vagina. What should you do to manage the bleeding?

  • Pack the vagina with trauma dressings.
  • Ask the patient to insert a tampon.
  • Place a sanitary pad over the vaginal opening.
  • Apply direct pressure using trauma dressings.
    Place a sanitary pad over the vaginal opening.

If PID is not treated, what can occur?

  • Ectopic pregnancy
  • Uterine collapse
  • Low-birth-weight babies
  • Arthritis
    Ectopic pregnancy

You are assessing a 38-year-old woman who is reporting achy abdominal pain, burning on urination, and a slight fever. As you assist her to the ambulance, you notice that she has a shuffling gait. What condition does this indicate?

  • Ectopic pregnancy
  • Chlamydia
  • Bacterial vaginosis
  • PID
    PID

In the case of a sexual assault, what is your most important treatment option?

  • Transport to the hospital
  • Psychologic support
  • Supine positioning
  • Low-flow oxygen
    Psychologic support

In addition to providing medical care to a rape victim, which of the following is the most critical thing you should do?

  • Refer the patient to a rape crisis center.
  • Do a full assessment of the victim’s mental status to determine whether an assault has actually occurred.
  • Ask very specific questions to help identify the assailant.
  • Move the patient to a private location.
    Move the patient to a private location.

What should you do when a female patient has vaginal bleeding of unknown cause?

  • Treat for shock.
  • Determine the cause of the bleeding.
  • Tell the patient to see her gynecologist as soon as possible.
  • Pack the vagina with sterile gauze to stop blood loss.
    Treat for shock.

What is an EMT’s responsibility regarding evidence in the case of a sexual assault?

  • Analyze
  • Catalogue
  • Collect
  • Preserve
    Preserve

Why is it important to transport a sexual assault victim to the hospital even if she does not have any obvious injuries?

  • There may be injuries that were not detected in the field, such as internal bleeding, that have a delayed onset of symptoms.
  • It is not safe to allow the victim to return home until the police have had a chance to collect evidence.
  • Full documentation needs to be completed before the case can go to court.
  • The police need confirmation that a sexual assault actually occurred.
    There may be injuries that were not detected in the field, such as internal bleeding, that have a delayed onset of symptoms.

A 26-year-old female presents with heavy vaginal bleeding. She is conscious, but restless. Her blood pressure is 84/54 mm Hg, her pulse is 120 beats/min and weak, and her respirations are 22 breaths/min with adequate depth. She tells you that she inserted a tampon about 2 hours ago. You should:

  • administer high-flow oxygen, perform a detailed assessment of her vaginal area for signs of trauma, place her on her side, and transport.
  • administer high-flow oxygen, ask her to remove the tampon, perform a detailed secondary assessment, and transport promptly.
  • assist her ventilations with a bag-valve mask, place one sterile dressing into her vagina, perform a rapid secondary assessment, and transport.
  • administer high-flow oxygen, place a sterile pad over her vagina, keep her warm, elevate her lower extremities, and transport without delay.
    administer high-flow oxygen, place a sterile pad over her vagina, keep her warm, elevate her lower extremities, and transport without delay.

A patient with pelvic inflammatory disease will typically complain of _.

  • abdominal pain associated with menstruation
  • nausea and vomiting associated with intercourse
  • bleeding associated with stress
  • aches and fever associated with urination
    abdominal pain associated with menstruation

Causes of vaginal bleeding include which of the following?

  • Ectopic pregnancy
  • Vaginal trauma
  • Spontaneous abortion
  • All of these answers are correct.
    All of these answers are correct.

During gynecologic emergencies, in addition to the standard SAMPLE questions, the EMT should attempt to determine which of the following?

  • The amount of time since of the patent’s last menstrual cycle
  • The possibility that the patient may be pregnant
  • If the patient is sexually active
  • All of these answers are correct.
    All of these answers are correct.

In anticipation of receiving a fertilized ovum, the lining of the uterine wall:

  • diverts blood flow to the vagina.
  • sheds and is expelled externally.
  • thins and begins to separate.
  • becomes engorged with blood.
    becomes engorged with blood.

In cases of sexual assault, it is important to ensure that you do which of the following?

  • Immediately write down your observations of the scene before assessing the patient.
  • Call for a female EMT or ALS care provider if you yourself are not a female provider.
  • Discuss with law enforcement any suspicions that you may have about the perpetrator.
  • All of these answers are correct.
    Call for a female EMT or ALS care provider if you yourself are not a female provider.

In contrast to bleeding caused by external trauma to the vagina, bleeding caused by conditions such as polyps or cancer:

  • can be controlled in the field.
  • may be relatively painless.
  • often presents with acute pain.
  • is typically not as severe.
    may be relatively painless.

It is common for young females who experience their first menstrual period to:

  • become so emotionally distraught that they contemplate suicide.
  • experience abdominal cramping, which may be misinterpreted.
  • have a false positive home pregnancy test result.
  • lose up to 500 mL of blood within the first 24 hours.
    experience abdominal cramping, which may be misinterpreted.

Law enforcement personnel request your assistance to assess a 31-year-old female who was sexually assaulted. When you arrive at the scene, you find the patient sitting on a curb outside her apartment. She is conscious, alert, and crying. When you ask her what happened, she tells you that she does not want to be treated or transported to the hospital. She further tells you that all she wants to do is clean up. You should:

  • provide emotional support and visually assess her for obvious trauma.
  • advise her that she cannot clean herself up because this will destroy evidence.
  • perform a limited hands-on assessment to detect life-threatening injuries.
  • ask her if there is anyone you can contact, such as a friend or relative.
    provide emotional support and visually assess her for obvious trauma.

Patient complaints of pain during intercourse, lower abdominal discomfort, and nausea may be indicative of which of the following?

  • Chlamydia
  • Pelvic inflammatory disease
  • Gonorrhea
  • Bacterial vaginosis
    Pelvic inflammatory disease

Some women experience _ during ovulation.

  • hypotension
  • severe abdominal pain
  • slight cramping
  • severe bleeding
    slight cramping

The _ and are folds of tissue that surround the urethral and vaginal openings.

  • fallopian tubes; uterus
  • vagina; cervix
  • labia majora; labia minora
  • perineum; anus
    labia majora; labia minora

The _ connect(s) each ovary with the uterus.

  • cervix
  • egg
  • fallopian tubes
  • abdomen
    fallopian tubes

The EMT should ensure that vaginal bleeding is _.

  • controlled directly by the patient
  • taken seriously and the patient is transported for gynecologic evaluation
  • not caused by traumatic injury
  • severe enough to warrant transport
    taken seriously and the patient is transported for gynecologic evaluation

The physical examination of a sexual assault victim should be:

  • performed in the presence of at least two police officers.
  • deferred until the patient can be evaluated by a physician.
  • as detailed as possible so all injuries can be documented.
  • limited to a brief survey for life-threatening injuries.
    limited to a brief survey for life-threatening injuries.

limited to a brief survey for life-threatening injuries.

  • allow law enforcement to take her statement before you begin treatment.
  • advise her that she will not be allowed to shower or change her clothes.
  • ask the patient for a concise, detailed report of what happened to her.
  • place any bloodstained clothing or other articles in separate paper bags.
    place any bloodstained clothing or other articles in separate paper bags.

Which of the following conditions would MOST likely lead to pelvic inflammatory disease if left untreated?

  • Chlamydia
  • Genital herpes
  • Ectopic pregnancy
  • Ovarian cysts
    Chlamydia

Which of the following drugs is commonly referred to as “roofies”?

  • GHB
  • MDMA
  • Rohypnol
  • Ketamine
    Rohypnol

Which of the following infections, when left untreated, may lead to infection of the blood and brain?

  • Pelvic inflammatory disease
  • Bacterial vaginosis
  • Chlamydia
  • Gonorrhea
    Gonorrhea

While the ages can vary, women typically experience menstruation from approximately to ___ years of age.

  • 10; 70
  • 20; 60
  • 10; 40
  • 11; 50
    11; 50

A 50-year-old man with diabetes has an altered mental status and is unable to tell you when he last ate or took his insulin. Your glucometer keeps malfunctioning and you are unable to determine his blood glucose level. Which of the following clinical signs would MOST likely lead you to the correct diagnosis?

  • Hypotension and tachycardia
  • Rapid and weak pulse
  • Deep and rapid breathing
  • Restlessness and irritability
    Deep and rapid breathing

A 58-year-old man complains of chest discomfort and nausea. He is conscious and alert; his blood pressure is 140/90 mm Hg, his pulse is 104 beats/min, and his respirations are 16 breaths/min. Your partner has applied supplemental oxygen. Prior to assisting the patient with one of his prescribed nitroglycerin tablets, you ask him if he takes medication to treat erectile dysfunction (ED) and he tells you that he does. You should:

  • ask him what he takes, how much, and when he last took it.
  • administer his nitroglycerin and then reassess his blood pressure.
  • recall that erectile ED drugs can cause significant hypertension.
  • avoid giving him nitroglycerin and transport him at once.
    ask him what he takes, how much, and when he last took it.

A 60-year-old man complains of chest pain. He is conscious and alert and denies shortness of breath. Which of the following questions would be the MOST appropriate to ask him?

  • “Does the pain in your chest move to either of your arms?”
  • “Were you exerting yourself when the chest pain began?”
  • “Do you have any heart problems or take any medications?”
  • “Does the pain in your chest feel like a stabbing sensation?”
    “Do you have any heart problems or take any medications?”

A 70-year-old female was recently discharged from the hospital following a total hip replacement. Today, she presents with restlessness, tachycardia, and a blood pressure of 100/64 mm Hg. Her skin is hot and moist. You should be MOST suspicious that she is experiencing:

  • septic shock.
  • a local infection.
  • decompensated shock.
  • pump failure.
    septic shock.

A 71-year-old male is semiconscious following a sudden, severe headache. There is vomitus on his face and his respirations are slow and shallow. The EMT must immediately:

  • perform oropharyngeal suctioning.
  • insert a nasopharyngeal airway.
  • apply oxygen via a nonrebreathing mask.
  • begin assisting the patient’s ventilations.
    perform oropharyngeal suctioning.

A 73-year-old man presents with a generalized rash, which he thinks may have been caused by an antibiotic that he recently began taking. He has a history of coronary artery disease, hypertension, and emphysema. He is conscious and alert, his blood pressure is 144/94 mm Hg, and his pulse is 64 beats/min and regular. You auscultate his breath sounds and hear scattered wheezing, although he is not experiencing respiratory distress. You should:

  • begin transport and request to administer epinephrine if his systolic blood pressure falls below 110 mm Hg.
  • administer oxygen if needed, transport the patient, and monitor him for signs of deterioration.
  • avoid the use of epinephrine because of his cardiac history, even if his symptoms become severe.
  • ask him if he has epinephrine and request approval from medical control to administer it to the patient.
    administer oxygen if needed, transport the patient, and monitor him for signs of deterioration.

Abdominal thrusts in a conscious child or adult with a severe upper airway obstruction are performed:

  • until he or she experiences cardiac arrest.
  • in sets of five followed by reassessment.
  • until he or she loses consciousness.
  • about 1 inch below the xiphoid process.
    until he or she loses consciousness.

An absence seizure is also referred to as a:

  • generalized motor seizure.
  • grand mal seizure.
  • total body seizure.
  • petit mal seizure.
    petit mal seizure.

The left ventricle has the thickest walls because it:

  • pumps blood into the aorta and systemic circulation.
  • receives blood directly from the systemic circulation.
  • uses less oxygen than other chambers of the heart.
  • pumps blood to the lungs to be reoxygenated.
    pumps blood into the aorta and systemic circulation.

What is the alveolar minute volume of a patient with a tidal volume of 500 mL, a dead space volume of 150 mL, and a respiratory rate of 16 breaths/min?

  • 8,000 mL
  • 5,600 mL
  • 6,000 mL
  • 7,400 mL
    5,600 mL

When assessing an 80-year-old patient in shock, it is important to remember that:

  • the patient’s cardiac output is able to increase by nearly 200% in response to the decrease in perfusion.
  • in older adults, it is especially common to observe a significant decrease in heart rate in response to shock.
  • it is common to see a more significant increase in heart rate than what would be expected in younger adults.
  • age-related changes in the cardiovascular system may make the patient less able to compensate for decreased perfusion.
    age-related changes in the cardiovascular system may make the patient less able to compensate for decreased perfusion.

When assessing for fluid collection in the lungs during auscultation of lung sounds, you should:

  • start at the lower lung fields and determine at which level you start hearing clear breath sounds.
  • pay special attention to the exhalation phase because this is when you will likely hear rales or rhonchi.
  • note the presence of a high-pitched whistling sound, which is an indicator of fluid in the lungs.
  • auscultate the posterior chest first and compare the apex of one lung to the base of the opposite lung.
    start at the lower lung fields and determine at which level you start hearing clear breath sounds.

When gathering a patient’s medications, you find the following: Isordil, Lasix, Nexium, and digoxin. Which of these medications can be obtained over-the-counter (OTC)?

  • Nexium
  • Lasix
  • Digoxin
  • Isordil
    Nexium

Which of the following clinical signs is unique to anaphylactic shock?

  • Wheezing
  • Dizziness
  • Hypotension
  • Pallor
    Wheezing

Which of the following medication routes has the slowest rate of absorption?

  • Inhalation
  • Rectal
  • Sublingual
  • Oral
    Oral

Which of the following statements regarding gastrointestinal bleeding is correct?

  • In the majority of cases, bleeding within the gastrointestinal tract occurs acutely and is severe.
  • Chronic bleeding within the gastrointestinal tract is usually more severe than bleeding that occurs acutely.
  • Bleeding within the gastrointestinal tract is a symptom of another disease, not a disease itself.
  • Lower gastrointestinal bleeding results from conditions such as Mallory-Weiss syndrome.
    Bleeding within the gastrointestinal tract is a symptom of another disease, not a disease itself.

Which of the following statements regarding hepatitis A is correct?

  • Although there is no vaccine against hepatitis A, treatment is usually successful.
  • Infection with hepatitis A causes chronic illness with a high mortality rate.
  • Hepatitis A can only be transmitted by a patient who has an acute infection.
  • Hepatitis A is primarily transmitted via contact with blood or other body fluids.
    Hepatitis A can only be transmitted by a patient who has an acute infection.

Which of the following statements regarding smooth muscle is correct?

  • The biceps and quadriceps are examples of smooth muscle.
  • Smooth muscle is found exclusively within blood vessels.
  • Smooth muscle is under control of the voluntary nervous system.
  • A person has no voluntary control over smooth muscle.
    A person has no voluntary control over smooth muscle.

Which of the following statements regarding ventricular fibrillation (V-fib) is correct?

  • Most patients in V-fib have a weak carotid pulse.
  • Defibrillation is only indicated for witnessed V-fib.
  • V-fib is a state of rapid ventricular contraction.
  • V-fib results in an absence of forward blood flow.
    V-fib results in an absence of forward blood flow.

You are assessing a 49-year-old man who, according to his wife, experienced a sudden, severe headache and then passed out. He is unresponsive and has slow, irregular breathing. His blood pressure is 190/94 mm Hg and his pulse rate is 50 beats/min. His wife tells you that he has hypertension and diabetes. He has MOST likely experienced:

  • an occluded cerebral artery.
  • a ruptured cerebral artery.
  • a complex partial seizure.
  • acute hypoglycemia.
    a ruptured cerebral artery.

You are dispatched to a residence for a 67-year-old female who was awakened by shortness of breath and sharp chest pain. Her husband tells you that she was recently discharged from the hospital after having hip surgery. Your assessment reveals dried blood around her mouth, facial cyanosis, and an oxygen saturation of 88%. You should suspect:

  • spontaneous pneumothorax.
  • acute pulmonary embolism.
  • acute pulmonary edema.
  • right-sided heart failure.
    acute pulmonary embolism.

You are dispatched to a residence where a middle-aged man was found unconscious in his front yard. There are no witnesses who can tell you what happened. You find him in a prone position; his eyes are closed and he is not moving. Your FIRST action should be to:

  • log roll him as a unit to a supine position.
  • open his airway with a jaw-thrust maneuver.
  • assess the rate and quality of his breathing.
  • palpate for the presence of a carotid pulse.
    log roll him as a unit to a supine position.

The process of exchanging oxygen and carbon dioxide between the alveoli and the blood of the capillaries is called:

  • pulmonary ventilation.
  • external respiration.
  • cellular metabolism.
  • alveolar ventilation.
    external respiration.

To select the proper size oropharyngeal airway, you should measure from the:

  • corner of the mouth to the superior ear.
  • angle of the jaw to the center of the mouth.
  • center of the mouth to the posterior ear.
  • corner of the mouth to the earlobe.
    corner of the mouth to the earlobe.

While providing CPAP to a patient in severe respiratory distress, you note that his heart rate has increased by 20 beats/min. He is conscious, but is no longer following verbal commands. You should:

  • remove the CPAP device and apply oxygen by nonrebreathing mask.
  • remove the CPAP device and ventilate him with a bag-valve mask.
  • increase the amount of pressure that the CPAP device is delivering.
  • decrease the amount of pressure that the CPAP device is delivering.
    remove the CPAP device and ventilate him with a bag-valve mask.

You are ventilating a 40-year-old uninjured man who is apneic but has a pulse. When your partner reassesses his blood pressure, she notes that it has decreased significantly from previous readings. You should:

  • increase the volume of your ventilations and reassess his blood pressure.
  • perform a head-to-toe assessment to look for signs of bleeding.
  • reduce the rate or volume of the ventilations you are delivering.
  • increase the rate at which you are ventilating and reassess his blood pressure.
    reduce the rate or volume of the ventilations you are delivering.

You are ventilating an apneic woman with a bag-valve mask. She has dentures, which are tight fitting. Adequate chest rise is present with each ventilation, and the patient’s oxygen saturation reads 96%. When you reassess the patency of her airway, you note that her dentures are now loose, although your ventilations are still producing adequate chest rise. You should:

  • leave her dentures in place, but carefully monitor her for an airway obstruction.
  • attempt to replace her dentures so that they fit tightly and resume ventilations.
  • remove her dentures, resume ventilations, and assess for adequate chest rise.
  • remove her dentures at once and increase the rate and volume of your ventilations.
    remove her dentures, resume ventilations, and assess for adequate chest rise.

Neurogenic shock occurs when:

  • the spinal cord is severed and causes massive hemorrhaging.
  • massive vasoconstriction occurs distal to a spinal cord injury.
  • failure of the nervous system causes widespread vasodilation.
  • there is too much blood to fill a smaller vascular container.
    failure of the nervous system causes widespread vasodilation.

You are transporting a 33-year-old male who was involved in a major motor vehicle crash. You have addressed all immediate and potentially life-threatening conditions and have stabilized his condition with the appropriate treatment. With an estimated time of arrival at the hospital of 20 minutes, you should:

  • repeat your secondary assessment.
  • reassess his condition in 5 minutes.
  • take his vital signs in 15 minutes.
  • arrange for an ALS rendezvous.
    reassess his condition in 5 minutes.

A 22-year-old female patient is complaining of dyspnea and numbness and tingling in her hands and feet after an argument with her fiancé. Her respirations are 40 breaths/min. You should:

  • request a paramedic to give her a sedative.
  • provide reassurance and give oxygen as needed.
  • have her breathe into a paper or plastic bag.
  • position her on her left side and transport at once.
    provide reassurance and give oxygen as needed.

A 62-year-old man with a history of congestive heart failure presents with severe respiratory distress and with an oxygen saturation of 82%. When you auscultate his lungs, you hear widespread rales. He is conscious and alert, is able to follow simple commands, and can only speak in two- to three-word sentences at a time. You should:

  • place him in a supine position and assist his ventilations with a bag-valve mask and high-flow oxygen.
  • place him in a position of comfort, deliver oxygen via nasal cannula, and closely monitor his breathing.
  • apply a CPAP device, monitor his blood pressure, and observe him for signs of improvement or deterioration.
  • force fluid from his alveoli by hyperventilating him with a bag-valve mask at a rate of at least 20 breaths/min.
    apply a CPAP device, monitor his blood pressure, and observe him for signs of improvement or deterioration.

Dyspnea is MOST accurately defined as:

  • shortness of breath or difficulty breathing.
  • labored breathing with reduced tidal volume.
  • a complete cessation of respiratory effort.
  • a marked increase in the exhalation phase.
    shortness of breath or difficulty breathing.

You are dispatched to an apartment complex where a 21-year-old female has apparently overdosed on several narcotic medications. She is semiconscious and has slow, shallow respirations. You should:

  • insert a nasopharyngeal airway and begin assisted ventilation.
  • place her in the recovery position and monitor for vomiting.
  • apply oxygen via a nonrebreathing mask and transport at once.
  • insert an oropharyngeal airway and perform oral suctioning.
    insert a nasopharyngeal airway and begin assisted ventilation.

A 40-year-old man is in cardiac arrest. Your partner is performing CPR. You are attaching the AED when the patient’s wife tells you that he has an automatic implanted cardiac defibrillator (AICD). The AED advises that a shock is indicated. What should you do?

  • Continue CPR and transport the patient to the closest appropriate hospital.
  • Deliver the shock followed by immediate resumption of CPR.
  • Contact medical control and request permission to defibrillate.
  • Avoid defibrillation as this will damage the patient’s AICD.
    Deliver the shock followed by immediate resumption of CPR.

Acute coronary syndrome (ACS) is a term used to describe:

  • the warning signs that occur shortly before a heart attack.
  • the exact moment that a coronary artery is completely occluded.
  • a group of symptoms that are caused by myocardial ischemia.
  • a severe decrease in perfusion caused by changes in heart rate.
    a group of symptoms that are caused by myocardial ischemia.

Cardiogenic shock following AMI is caused by:

  • hypovolemia secondary to severe vomiting.

B. widespread dilation of the systemic vasculature.

  • a profound increase in the patient’s heart rate.
  • decreased pumping force of the heart muscle.
    decreased pumping force of the heart muscle.

Common side effects of nitroglycerin include all of the following, EXCEPT:

  • hypotension.
  • severe headache.
  • hypertension.
  • bradycardia.
    hypertension.

Common signs and symptoms of AMI include all of the following, EXCEPT:

  • sudden unexplained sweating.
  • shortness of breath or dyspnea.
  • pain exacerbated by breathing.
  • irregular heartbeat.
    pain exacerbated by breathing.

You are assessing a 49-year-old man who complains of chest pressure that began the night before. He is conscious, but anxious, and tells you he has a history of angina and hypertension. After applying high-flow oxygen, you expose his chest to auscultate his lungs and note that he has a nitroglycerin patch on his right upper chest. His skin is cool and pale, his blood pressure is 78/50 mm Hg, and his pulse is 110 beats/min and irregular. You should:

  • move the nitroglycerin patch to the other side of his chest in case you need to apply the AED, keep him warm, and transport without delay.
  • ask him if the nitroglycerin patch he is wearing has improved his chest pressure, complete your secondary assessment, and transport promptly.
  • immediately remove the nitroglycerin patch, apply the AED in case he develops cardiac arrest, and transport to the closest hospital.
  • remove the nitroglycerin patch, place him in a supine position and elevate his lower extremities, and prepare for immediate transport.
    remove the nitroglycerin patch, place him in a supine position and elevate his lower extremities, and prepare for immediate transport.

You are dispatched to a convenience store for a patient who passed out. Upon arriving at the scene, you find two off-duty EMTs performing CPR on the patient, a 58-year-old male. Your initial action should be to:

  • have the EMTs stop CPR and assess for a pulse.
  • request a paramedic unit and quickly attach the AED.
  • feel for a pulse while compressions are ongoing.
  • quickly attach the AED and push the analyze button.
    feel for a pulse while compressions are ongoing.

You are dispatched to a residence for a 56-year-old male with an altered mental status. Upon arrival at the scene, the patient’s wife tells you that he complained of chest pain the day before, but would not allow her to call EMS. The patient is semiconscious; has rapid, shallow respirations; and has a thready pulse. You should:

  • apply a nonrebreathing mask.
  • obtain baseline vital signs.
  • attach the AED immediately.
  • begin ventilatory assistance.
    begin ventilatory assistance.

Kussmaul respirations are an indication that the body is:

  • attempting to eliminate acids from the blood.
  • severely hypoxic and is eliminating excess CO2.
  • compensating for decreased blood glucose levels.
  • trying to generate energy by breathing deeply.
    attempting to eliminate acids from the blood.

To which of the following diabetic patients should you administer oral glucose?

  • A semiconscious 40-year-old female without a gag reflex
  • A confused 55-year-old male with tachycardia and pallor
  • A conscious 37-year-old female with nausea and vomiting
  • An unconscious 33-year-old male with cool, clammy skin
    A confused 55-year-old male with tachycardia and pallor

When assessing an unresponsive diabetic patient, the primary visible difference between hyperglycemia and hypoglycemia is the:

  • patient’s mental status.
  • rate of the patient’s pulse.
  • rate and depth of breathing.
  • presence of a medical identification tag.
    rate and depth of breathing.

If a woman with vaginal bleeding reports syncope, the EMT should assume that she:

  • has an ectopic pregnancy.
  • is pregnant.
  • is in shock.
  • has an infection.
    is in shock.

When a woman presents with abdominal pain or other vague symptoms, the EMT is often unable to determine the nature of the problem until he or she:

  • has gathered patient history information.
  • ascertains if the patient was ever pregnant.
  • has formed a general impression of the patient.
  • has obtained a complete set of vital signs.
    has gathered patient history information.

Whenever possible, a female sexual assault victim should be:

  • asked to provide a brief description of the perpetrator.
  • given the option of being treated by a female EMT.
  • thoroughly assessed, even if no signs of injury exist.
  • encouraged to take a shower and change her clothes.
    given the option of being treated by a female EMT.

A young male is unresponsive after overdosing on an opioid. He is not breathing and his pulse is weak. The EMT should immediately:

  • request an ALS ambulance.
  • administer naloxone.
  • ventilate with a BVM.
  • begin chest compressions.
    ventilate with a BVM.

If gastric distention begins to make positive-pressure ventilation difficult, you should:

  • reposition the patient’s airway.
  • suction the patient’s oropharynx.
  • increase the rate of ventilation.
  • insert an oropharyngeal airway.
    reposition the patient’s airway.

The MOST appropriate treatment for a patient with a mild upper airway obstruction includes:

  • visualizing the airway and removing the obstruction.
  • administering oxygen and transporting immediately.
  • advising the patient not to make any attempts to cough.
  • performing five back blows and five abdominal thrusts.
    administering oxygen and transporting immediately.

Which of the following has the greatest impact on producing injury?

  • Gravity
  • Distance
  • Mass
  • Speed
    Speed

How many collisions are there in a typical vehicle impact?

  • 1
  • 3
  • 4
  • 2
    3

In which type of vehicle crash are you most likely to find a patient with whiplash injuries?

  • Rotational
  • Frontal
  • Rear-end
  • Rollover
    Rear-end

The phenomenon that results from the rapid changes in tissue and fluid pressure that occur with the passage of a projectile, resulting in serious injury to internal organs distant to the actual path of the bullet, is known as what?

  • Tumble
  • Drag
  • Cavitation
  • Fragmentation
    Cavitation

What type of blast injury occurs when objects are propelled by the force of the blast wave and strike the victim, causing injury?

  • Tertiary
  • Primary
  • Quaternary
  • Secondary
    Secondary

What level of trauma center provides prompt assessment, resuscitation, and stabilization of injured patients and emergency operations, but not definitive care for all patients?

  • Level III
  • Level II
  • Level I
  • Level IV
    Level III

You are assessing a patient who opens her eyes when you speak to her, who can respond to you but seems confused as to time and place, and localizes pain. What is her Glasgow Coma Scale score?

  • 12
  • 15
  • 10
  • 7
    12

You are assessing a patient who was in a front-end car crash. His GCS is 12, systolic blood pressure is 81 mm Hg, and respiratory rate is 22 breaths/min. What is this patient’s Revised Trauma Score?

  • 12
  • 15
  • 10
  • 7
    10

You would use the Revised Trauma Score scoring system for a patient if there is potential for what type of trauma?

  • Head
  • Extremity
  • Chest
  • Abdominal
    Head

At what step in the ACS-COT/CDC updated field triage decision scheme would you assess mechanism of injury and evidence of high-energy impact?

  • Step 2
  • Step 1
  • Step 4
  • Step 3
    Step 3

Which of the following interventions is the MOST critical to the outcome of a patient with multisystem trauma?

  • Early administration of oxygen
  • Elevation of the lower extremities
  • Intravenous fluid administration
  • Rapid transport to a trauma center
    Rapid transport to a trauma center

A 12-year-old male jumped approximately 12 feet from a tree and landed on his feet. He complains of pain to his lower back. What injury mechanism is MOST likely responsible for his back pain?

  • Lateral impact to the spine
  • Energy transmission to the spine
  • Secondary fall after the initial impact
  • Direct trauma to the spinal column
    Energy transmission to the spine

When assessing an elderly patient who fell, it is important to remember that:

  • bilateral hip fractures usually occur when an elderly person falls.
  • elderly patients who fall usually have a secondary head injury.
  • osteoporosis can cause a fracture as a result of a fall from a standing position.
  • any fall in the elderly is considered to be high-energy trauma.
    osteoporosis can cause a fracture as a result of a fall from a standing position.

While en route to a major motor vehicle crash, an on-scene police officer advises you that a 6-year-old male who was riding in the front seat is involved. He further states that the child was only wearing a lap belt and that the air bag deployed. On the basis of this information, you should be MOST suspicious that the child has experienced:

  • open abdominal trauma.
  • blunt trauma to the head.
  • neck and facial injuries.
  • lower extremity fractures.
    neck and facial injuries.

Force acting over a distance is the definition of:

  • kinetic energy.
  • latent energy.
  • work.
  • potential energy.
    work.

According to the American College of Surgeons Committee on Trauma (ACS-COT), an adult trauma patient should be transported to the highest level of trauma center if he or she:

  • has a GCS score of less than or equal to 13 with a mechanism attributed to trauma.
  • was involved in a motor vehicle crash in which another patient in the same vehicle was killed.
  • has a bleeding disorder or takes anticoagulant medications and has any blunt or penetrating injury.
  • has a systolic blood pressure of less than 110 mm Hg or a heart rate greater than 110 beats/min.
    has a GCS score of less than or equal to 13 with a mechanism attributed to trauma.

Which types of motor vehicle collisions present the greatest potential for multiple impacts?

  • Frontal and rotational
  • Lateral and rollover
  • Rear-end and rotational
  • Rotational and rollover
    Rotational and rollover

A 15-year-old female was struck by a small car while riding her bicycle. She was wearing a helmet and was thrown to the ground, striking her head. In addition to managing problems associated with airway, breathing, and circulation, it is MOST important for you to:

  • leave her bicycle helmet on.
  • obtain baseline vital signs.
  • inspect the helmet for cracks.
  • stabilize her entire spine.
    stabilize her entire spine.

Which of the following injuries would MOST likely occur as a direct result of the third collision in a motor vehicle crash?

  • Forehead lacerations
  • Aortic rupture
  • Flail chest
  • Extremity fractures
    Aortic rupture

Following a blunt injury to the head, a 22-year-old female is confused and complains of a severe headache and nausea. On the basis of these signs and symptoms, you should be MOST concerned with the possibility of:

  • a fracture of the skull.
  • airway compromise.
  • intracranial bleeding.
  • spinal cord injury.
    intracranial bleeding.

Which of the following destinations is most appropriate for a 41-year-old male patient who was involved in a rollover motor vehicle collision and is unconscious and unresponsive, assuming that travel times to each is equal?

  • Only a Level I trauma center
  • Any designated trauma center is acceptable.
  • A Level I or Level II trauma center
  • A Level IV or Level III trauma center
    A Level I or Level II trauma center

When assessing a patient who experienced a blast injury, it is important to remember that:

  • primary blast injuries are the most easily overlooked.
  • solid organs usually rupture from the pressure wave.
  • primary blast injuries are typically the most obvious.
  • secondary blast injuries are usually the least obvious.
    primary blast injuries are the most easily overlooked.

Approximately 25% of severe injuries to the aorta occur during:

  • lateral collisions.
  • rollover collisions.
  • frontal collisions.
  • rear-end collisions.
    lateral collisions.

When the speed of a motor vehicle doubles, the amount of kinetic energy:

  • triples.
  • quadruples.
  • doubles.
  • is not affected.
    quadruples.

According to the Association of Air Medical Services, you should consider air medical transport of a trauma patient if:

  • he or she was involved in a motor vehicle crash in which another occupant in the same vehicle was killed, even if your patient’s injuries are minor.
  • the patient requires advanced life support care and stabilization, and the nearest ALS-ground ambulance is more than 5 to 10 minutes away.
  • traffic conditions hamper the ability to get the patient to a trauma center by ground within the ideal time frame for the best clinical outcome.
  • ground transport will take your ambulance out of service for an extended period of time, regardless of the severity of the patient’s injuries.
    traffic conditions hamper the ability to get the patient to a trauma center by ground within the ideal time frame for the best clinical outcome.

Which of the following findings would be LEAST suggestive of the presence of high-energy trauma?

  • Deployment of the air bag
  • Steering wheel deformity
  • Intrusion into the vehicle
  • Dismounted seats
    Deployment of the air bag

The index of suspicion is MOST accurately defined as:

  • the way in which traumatic injuries occur.
  • the detection of less obvious life-threatening injuries.
  • a predictable pattern that leads to serious injuries.
  • your awareness and concern for potentially serious underlying injuries.
    your awareness and concern for potentially serious underlying injuries.

Your patient has a Glasgow Coma Scale (GCS) score of 13, a systolic blood pressure of 80 mm Hg, and a respiratory rate of 8 breaths/min. His Revised Trauma Score (RTS) is:

  • 11.
  • 8.
  • 9.
  • 10.
    9.

Which of the following statements regarding gunshot wounds is correct?

  • Low-velocity bullets will cause the greatest amount of trauma.
  • The size of a bullet has the greatest impact on the injury produced.
  • The speed of a bullet has the greatest impact on the injury produced.
  • High-velocity bullets will cause less severe internal injuries.
    The speed of a bullet has the greatest impact on the injury produced.

A young male sustained a gunshot wound to the abdomen during an altercation. As your partner is assessing and managing his airway, you should control the obvious bleeding and then:

  • apply a cervical collar.
  • auscultate bowel sounds.
  • assess for an exit wound.
  • obtain baseline vital signs.
    assess for an exit wound.

Which of the following are the key to the formation of blood clots?

  • Plasma
  • Platelets
  • White blood cells
  • Red blood cellsateletesl
    Platelets

How soon do cells in the brain and spinal cord start to die without constant perfusion?

  • After 2-3 hours
  • Within 45 minutes
  • Within 15-20 minutes
  • Within 4-6 minutes
    Within 4-6 minutes

In general, what is the maximum amount of acute blood loss the adult body can tolerate?

  • 2 liters
  • 20% of total blood volume
  • 10% of total blood volume
  • 12 pints
    20% of total blood volume

The condition in which low blood volume results in inadequate perfusion is called what?

  • Cardiogenic shock
  • Neurogenic shock
  • Hypovolemic shock
  • Compensated shock
    Hypovolemic shock

Which type of bleeding is characterized by a bright red color and spurting?

  • Arterial
  • Hemoptysis
  • Venous
  • Capillary
    Arterial

Which of the following would be the first sign of hypovolemic shock?

  • Anxiety
  • Nausea and vomiting
  • Pale skin color
  • Decreasing blood pressure
    Anxiety

You determine that your patient is experiencing internal bleeding. What should you do first?

  • Apply cold packs.
  • Administer oxygen.
  • Immobilize the injury.
  • Apply pressure dressings.
    Administer oxygen.

Which condition is likely when signs of hypotension; tachycardia; and cool, clammy skin are found?

  • Internal bleeding
  • Intracranial bleeding
  • CNS depression
  • Shock
    Shock

What should you do first to control external bleeding in an extremity?

  • Apply a pressure dressing.
  • Apply a tourniquet.
  • Elevate the extremity.
  • Apply direct pressure.
    Apply direct pressure.

You are assessing a patient who is bleeding from the ear, which gives you a high index of suspicion that he has a skull fracture. How should you address this?

  • Pack a pressure dressing in the ear canal.
  • Loosely cover the site.
  • Do not do anything. Trying to stop the blood flow will force the blood back into the head.
  • Apply direct pressure.
    Loosely cover the site.

Which of the following statements regarding hemophilia is correct?

  • Approximately 25% of the population has hemophilia.
  • Hemophilia is defined as a total lack of platelets.
  • Hemophiliacs take aspirin to enhance blood clotting.
  • Patients with hemophilia may bleed spontaneously.
    Patients with hemophilia may bleed spontaneously.

Significant vital sign changes will occur if the typical adult acutely loses more than __ of his or her total blood volume.

  • 15%
  • 5%
  • 20%
  • 10%
    20%

A 43-year-old man is experiencing a severe nosebleed. His blood pressure is 190/110 mm Hg and his heart rate is 90 beats/min and bounding. Preferred treatment for this patient includes:

  • having the patient pinch his own nostrils and then lie supine.
  • packing both nostrils with gauze pads until the bleeding stops.
  • pinching the patient’s nostrils and having him lean forward.
  • placing a rolled 4² × 4² dressing between his lower lip and gum.
    pinching the patient’s nostrils and having him lean forward.

Following blunt trauma to the abdomen, a 21-year-old female complains of diffuse abdominal pain and pain to the left shoulder. Your assessment reveals that her abdomen is distended and tender to palpation. On the basis of these findings, you should be MOST suspicious of injury to the:

  • gallbladder.
  • liver.
  • pancreas.
  • spleen.
    spleen.

Internal bleeding into a fractured extremity is MOST often controlled by:

  • applying a tourniquet.
  • keeping the patient warm.
  • applying chemical ice pack.
  • splinting the extremity.
    splinting the extremity.

Which portion of the blood carries oxygen to and wastes away from body tissues?

  • Platelets
  • Red blood cells
  • White blood cells
  • Plasma
    Red blood cells

Which organ or organ system has the greatest tolerance for lack of perfusion (shock)?

  • Kidneys
  • Brain
  • Gastrointestinal system
  • Skeletal muscle
    Gastrointestinal system

Circulation of blood within an organ or tissue in adequate amounts to meet the cells’ oxygen, nutritional, and waste-removal needs is termed _.

  • hypoperfusion
  • coagulation
  • perfusion
  • hemorrhage
    perfusion

If applying a dressing to control the bleeding of a patient’s arm, the EMT should __.

  • apply direct pressure first
  • use large or small gauze pads or dressings depending upon the size of the wound
  • cover the entire wound, above and below, with the dressing
  • All of these answers are correct.
    All of these answers are correct.

Which section of the heart receives deoxygenated blood?

  • Atria
  • Right
  • Left
  • Ventricles
    Right

A 67-year-old male presents with weakness, dizziness, and melena that began approximately 2 days ago. He denies a history of trauma. His blood pressure is 90/50 mm Hg and his pulse is 120 beats/min and thready. You should be MOST suspicious that this patient is experiencing:

  • intrathoracic hemorrhaging.
  • an aortic aneurysm.
  • acute appendicitis.
  • gastrointestinal bleeding.
    gastrointestinal bleeding.

The severity of bleeding should be based on all of the following findings, EXCEPT:

  • clinical signs and symptoms.
  • poor general appearance.
  • systolic blood pressure.
  • the mechanism of injury.
    systolic blood pressure.

Hypovolemic shock occurs when:

  • the clotting ability of the blood is enhanced.
  • at least 10% of the patient’s blood volume is lost.
  • the patient’s systolic blood pressure is less than 100 mm Hg.
  • the body cannot compensate for rapid blood loss.
    the body cannot compensate for rapid blood loss.

A young male was shot in the abdomen by an unknown type of gun. He is semiconscious, has shallow breathing, and is bleeding externally from the wound. As you control the external bleeding, your partner should:

  • obtain baseline vital signs.
  • assist the patient’s ventilations.
  • perform a secondary assessment.
  • apply a nonrebreathing mask.
    assist the patient’s ventilations.

Whether you are using a commercial device or a stick and triangular bandage as a tourniquet, it is important to remember that:

  • the tourniquet should be applied directly over a joint if possible because this provides better bleeding control.
  • you should try to control the bleeding by applying pressure to a proximal arterial pressure point first.
  • bulky dressings should be securely applied over the tourniquet to further assist in controlling the bleeding.
  • the tourniquet should only be removed at the hospital because bleeding may return if the tourniquet is released.
    the tourniquet should only be removed at the hospital because bleeding may return if the tourniquet is released.

Which of the following findings would be the MOST significant when assessing a patient with possible internal bleeding?

  • The patient has not eaten in 24 hours.
  • The patient has a history of hypertension.
  • The patient had a stroke 5 years prior.
  • The patient takes rivaroxaban (Xeralto).
    The patient takes rivaroxaban (Xeralto).

Which of the following splinting devices would be MOST appropriate to use for a patient who has an open fracture of the forearm with external bleeding?

  • Sling and swathe
  • Air splint
  • Cardboard splint
  • Vacuum splint
    Air splint

If direct pressure fails to immediately stop severe bleeding from an extremity, you should apply:

  • digital pressure to a proximal artery.
  • a tourniquet proximal to the injury.
  • additional sterile dressings.
  • a splint and elevate the extremity.
    a tourniquet proximal to the injury.

What mechanism(s) does the body use to control bleeding?

  • Clotting
  • Coagulation
  • Vasoconstriction
  • All of these answers are correct.
    All of these answers are correct.

In older patients, the first indicator of nontraumatic internal bleeding may be:

  • diaphoresis and pale skin.
  • a low blood pressure.
  • weakness or dizziness.
  • a heart rate over 120 beats/min.
    weakness or dizziness.

Which of the following is the final stage of wound healing?

  • Formation of new capillaries
  • Inflammation
  • Cessation of blood loss
  • Collagen production
    Collagen production

You are assessing a 28-year-old with a rigid cast extending the length of his left leg. The patient is complaining of pain on passive movement and he is showing signs of impaired circulation. What do you suspect?

  • Ecchymosis
  • Crush syndrome
  • Avulsion
    Compartment syndrome

You are assessing a patient who has been in a high-impact T-bone collision. She is bleeding from the abdomen and you can see part of her large intestine outside her body. What is this injury called?

  • Impalement
  • Evisceration
  • Avulsion
  • Ecchymosis
    Evisceration

You are treating an 8-year-old who fell while riding his bike on a gravel road. The wound on his arm is seeping blood, is about 7 inches long, and there is a lot of dirt and gravel in it. How should you treat this injury?

  • Apply occlusive dressing.
  • Flush the area with sterile saline.
  • Manually brush/pick the foreign material out.
  • Apply sterile dressing.
    Apply sterile dressing.

The severity of a thermal injury directly correlates with temperature, concentration, and what other factor?

  • Duration of exposure
  • Current
  • Chemical reaction
  • Source of thermal injury
    Duration of exposure

You are responding to a fight at a restaurant and find a man, conscious and alert. His face is mottled with blisters and abrasions, and he has blood on his shirt. He tells you he was trying to stop the fight when he was hit in the face with scalding hot coffee and fell backwards over a chair. According to the rule of nines, what percentage of the patient’s skin surface is burned?

  • 4.5%
  • 18%
  • 9%
  • 12%
    9%

What is your first responsibility when treating a burn patient?

  • Administering high-flow oxygen
  • Stopping the burning
  • Preventing loss of body heat
  • Identifying life-threatening injuries
    Stopping the burning

Which of the following is true about electrical burns?

  • They always have an entrance wound but only occasionally have an exit wound.
  • Seriousness of electrical burns depends on the type of current, amperage, and conductivity.
  • They are always more severe than the external signs indicate.
  • The patient can go into cardiac arrest at any time within 24 hours of contact with electricity.
    They are always more severe than the external signs indicate.

Which type of soft-tissue injury is LEAST likely to result in infection?

  • Contusions on the left lateral chest
  • Abrasions to the face
  • Burns to the face
  • Abdominal laceration
    Contusions on the left lateral chest

When using the rule of nines, which of the following do you need to include in your calculations?

  • Superficial, full-, and partial-thickness burns
  • Only full-thickness burns
  • Full- and partial-thickness burns
  • Superficial and full-thickness burns
    Full- and partial-thickness burns

A 21-year-old male was working in an auto repair shop and sustained radiator burns to the anterior aspect of both arms and to his anterior chest. According to the rule of nines, this patient has burns that cover _ of his BSA.

  • 18%
  • 36%
  • 45%
  • 27%
    18%

A 33-year-old male sustained an abdominal evisceration to the left lower quadrant of his abdomen after he was cut with a large knife. After appropriately managing his ABCs and assessing him for other life-threatening injuries, how you should care for his wound?

  • Irrigate it with sterile water and cover it with a dry dressing.
  • Cover the exposed bowel and keep his legs in a straight position.
  • Carefully replace the exposed bowel into the abdomen and transport.
  • Cover it with moist, sterile gauze and secure with an occlusive dressing.
    Cover it with moist, sterile gauze and secure with an occlusive dressing.

A 39-year-old male was struck in the head by a baseball during a game. He is confused and has slurred speech. He has a large hematoma in the center of his forehead and cannot remember the events preceding the injury. After manually stabilizing his head and assessing his airway, you should:

  • palpate his radial pulses.
  • administer high-flow oxygen.
  • apply ice to the hematoma.
  • perform a neurologic exam.
    administer high-flow oxygen.

A 56-year-old male has an incomplete avulsion to his right forearm. After controlling any bleeding from the wound, you should:

  • carefully probe the wound to determine if the bleeding is venous or arterial.
  • thoroughly irrigate the wound with sterile water and cover it with a sterile dressing.
  • replace the avulsed flap to its original position and cover it with a sterile dressing.
  • carefully remove the avulsed flap and wrap it in a moist, sterile trauma dressing.
    replace the avulsed flap to its original position and cover it with a sterile dressing.

A burn that is characterized by redness and pain is classified as a:

  • partial-thickness burn.
  • second-degree burn.
  • superficial burn.
  • full-thickness burn.
    superficial burn.

A closed soft-tissue injury characterized by swelling and ecchymosis is called a(n):

  • crush injury.
  • abrasion.
  • contusion.
  • hematoma.
    contusion.

An 8-year-old male was bitten by a stray dog. He has a large laceration to the back of his left hand, which your partner covered with a sterile dressing and bandage. In addition to transporting the child to the hospital, you should:

  • administer oxygen via a nonrebreathing mask.
  • advise the child that he will need rabies shots.
  • ask the child’s father to try to locate the dog.
  • report the incident to the appropriate authorities.
    report the incident to the appropriate authorities.

As you approach a young male who was involved in an industrial accident, you note that his eyes are closed and that he is not moving. You can see several large contusions to his arms, a laceration to his forehead with minimal bleeding, and a closed deformity to his right leg. You should:

  • open his airway and assess his breathing status.
  • perform an immediate head-to-toe assessment.
  • assess his pulse for rate, regularity, and quality.
  • apply high-flow oxygen and assess his injuries.
    open his airway and assess his breathing status.

Burns to pediatric patients are generally considered more serious than burns to adults because:

  • pediatric patients are more prone to hyperthermia.
  • pediatric patients have a proportionately larger volume of blood.
  • pediatric patients have more surface area relative to total body mass.
  • most burns in children are the result of child abuse.
    pediatric patients have more surface area relative to total body mass.

During your assessment of a patient who was shot in the abdomen, you notice a large entrance wound with multiple small puncture wounds surrounding it. This wound pattern is MOST consistent with a:

  • shotgun.
  • .22-caliber pistol.
  • handgun.
  • .357 magnum.
    shotgun.

In addition to severe bleeding, the MOST life-threatening complication associated with an open neck injury is:

  • an air embolism.
  • nerve fiber damage.
  • a spinal fracture.
  • an ischemic stroke.
    an air embolism.

In which of the following patients should you remove an impaled object?

  • A conscious and alert patient with a fishhook impaled in the eye
  • A pulseless and apneic patient with a knife impaled in the back
  • A semiconscious patient with an ice pick impaled in the chest
  • An apneic patient with a shard of glass impaled in the abdomen
    A pulseless and apneic patient with a knife impaled in the back

The sebaceous glands produce sebum, a material that:

  • discharges sweat onto the skin’s surface.
  • waterproofs the skin and keeps it supple.
  • pulls the hair erect when you are cold.
  • facilitates shedding of the epidermis.
    waterproofs the skin and keeps it supple.

When caring for a patient whose arm is covered with a dry chemical, you should:

  • quickly irrigate the arm with large amounts of water.
  • deactivate the chemical with a 5% vinegar solution.
  • brush away the chemical before flushing with water.
  • use forceful streams of water to remove the chemical.
    brush away the chemical before flushing with water.

Which of the following areas of the body has the thinnest skin?

  • Back
  • Soles of the feet
  • Ears
  • Scalp
    Ears

Which of the following is a severe burn in a 65-year-old patient?

  • Partial-thickness burn to 20% of the BSA
  • Superficial burn to 30% of the BSA
  • Full-thickness burn to 1% of the BSA
  • Second-degree burn covering 10% of the BSA
    Partial-thickness burn to 20% of the BSA

Which of the following open soft-tissue injuries is limited to the superficial layer of the skin and results in the least amount of blood loss?

  • Laceration
  • Incision
  • Abrasion
  • Avulsion
    Abrasion

Which of the following processes occurs during the inflammation phase of the healing process?

  • The blood vessels in and around the injury site constrict, which forces bacteria and other microorganisms away, thus preventing significant infection.
  • White blood cells are forced away from the injury site, thus allowing an increase in the flow of red blood cells, resulting in increased blood flow.
  • The immune system releases histamines, which cause vasodilation and increased capillary permeability, resulting in local redness and swelling.
  • The veins and arteries at the injury site constrict and platelets aggregate, which stops bleeding and causes a temporary increase in the size of the wound.
    The immune system releases histamines, which cause vasodilation and increased capillary permeability, resulting in local redness and swelling.

Which of the following statements regarding crush syndrome is correct?

  • Provided that a patient with a crush injury is freed from entrapment within 6 hours, the amount of tissue damaged is generally minimal.
  • With crush syndrome, massive blood vessel damage occurs following severe soft-tissue injuries, such as amputation of an extremity.
  • Compromised arterial blood flow leads to crush syndrome and can occur when an area of the body is trapped for longer than 4 hours.
  • Tissue damage that occurs in crush syndrome is severe, but kidney injury is unlikely because toxins are quickly eliminated from the body.
    Compromised arterial blood flow leads to crush syndrome and can occur when an area of the body is trapped for longer than 4 hours.

You have applied a dressing and roller-gauze bandage to a laceration on the arm of a young female. During transport, she begins to complain of numbness and tingling in her hand. You should:

  • assess distal circulation and readjust the bandage as needed.
  • remove the gauze bandage and replace it with an elastic one.
  • carefully manipulate her arm until the symptoms subside.
  • remove the bandage and dressing and apply another one.
    assess distal circulation and readjust the bandage as needed.

What is a hematoma?

  • A rupture of the eye
  • A characteristic crackling sensation felt on palpation of the skin, caused by the presence of air in soft tissue
  • A collection of blood within the tissues
  • A flap of skin that is separated from the underlying muscle and fascia
  • A collection of blood within the tissues

Which of the following fractures is associated with bruising around the ears and blood coming from the nose?

  • Maxilla fracture
  • Basilar skull fracture
  • Orbit fracture
  • Mandibular fracture
    Basilar skull fracture

What is hyphema?

  • Nosebleed
  • Bleeding into the anterior chamber of the eye
  • Pain or difficulty swallowing
  • When air sucked into the heart
    Bleeding into the anterior chamber of the eye

You are assessing a patient who was hit in the face by a baseball bat. There are multiple contusions on his face, and he reports double vision. His left eye does not track with his right eye. What should you suspect?

  • Corneal burn
  • Blow-out fracture
  • Foreign object entrapment
  • Retinal detachment
    Blow-out fracture

Which of the following is or are found in the middle ear?

  • Tympanic membrane
  • Pinna
  • Anvil, hammer, and stirrup
  • Eustachian tube
    Anvil, hammer, and stirrup

In which instance would you consider removing an impaled object?

  • When the object is impaled in the ear and creates basilar trauma
  • When the object is impaled in the neck into or near the carotid artery
  • When the object is impaled in the eye and threatens vision
  • When the object is impaled in the cheek and compromises the airway
    When the object is impaled in the cheek and compromises the airway

What is an air embolism?

  • A clinical situation in which a vein is punctured and air is sucked into the heart
  • A swelling or enlargement of the wall of a blood vessel that results from weakening of the vessel wall
  • A lack of oxygen that deprives tissues of necessary nutrients, resulting from partial or complete blockage of blood flow
  • A crackling sensation felt when palpating the neck resulting from air in the soft tissues
  • A clinical situation in which a vein is punctured and air is sucked into the heart

Unequal pupils most likely indicate what type of injury?

  • Skull
  • Brain
  • Spine
  • Chest
    Brain

You are treating a patient with significant bruising to her face and a broken nose. She is conscious, but her LOC is significantly altered and her breathing is labored. What should you do to manage this patient’s airway?

  • Insert a nasopharyngeal airway.
  • Insert an oropharyngeal airway.
  • Apply high-flow oxygen via a nonrebreathing mask.
  • Give oxygen via a nasal cannula at 4 L/min.
    Insert an oropharyngeal airway.

Which area of the body are you palpating if you feel subcutaneous emphysema?

  • Abdomen
  • Spine
  • Chest
  • Neck
    Neck

The upper jawbones are called the:

  • maxillae
  • mandible
  • zygoma
  • mastoid
    maxillae

A 6-year-old female was riding her bicycle and struck a clothesline with her throat. She is breathing, but with obvious difficulty. Your assessment reveals a crackling sensation in the soft tissues of her neck and facial cyanosis. In addition to the appropriate airway management, the intervention that will MOST likely improve her chance of survival is:

  • careful monitoring her vital signs.
  • quickly immobilizing her spinal column.
  • rapidly transporting her to the hospital.
  • requesting a paramedic ambulance.
    rapidly transporting her to the hospital.

When performing a full body scan, you should assess for __.

  • DCAP-BTLS
  • OPQRST
  • AVPU
  • SAMPLE
    DCAP-BTLS

You are assessing a 59-year-old male and note that his pupils are unequal. He is conscious and alert. When obtaining his medical history, it is MOST pertinent to ask him if he:

  • is allergic to any medications.
  • regularly sees a family physician.
  • has a history of eye surgeries.
  • noticed the change during a meal.
    has a history of eye surgeries.

A 4-year-old female has a peanut lodged in the external auditory canal of her right ear. You should:

  • use tweezers to try to remove the object.
  • thoroughly flush her ear with sterile saline.
  • remove the peanut with a cotton-tipped swab.
  • transport her to the emergency department.
    transport her to the emergency department.

Which of the following statements regarding anterior nosebleeds is correct?

  • They are usually severe and require aggressive treatment to control.
  • They usually originate from the septum area and bleed slowly.
  • They are usually caused by a fracture of the basilar skull.
  • They cause blood to drain into the posterior pharynx.
    They usually originate from the septum area and bleed slowly.

When caring for a patient with an open facial injury, the EMT’s immediate priority should be to:

  • consider the mechanism of injury.
  • manually stabilize the patient’s head.
  • wear gloves and facial protection.
  • closely assess the patient’s airway.
    wear gloves and facial protection.

When transporting a patient with a facial injury, it is MOST important to be as descriptive as possible with the hospital regarding the patient’s injuries because:

  • it saves time on repeat assessments at the hospital.
  • they may need to call a specialist to see the patient.
  • they must make arrangements for an ICU bed.
  • most patients with facial trauma will need surgery.
    they may need to call a specialist to see the patient.

Which of the following statements regarding the vitreous humor is correct?

  • It is a clear, watery fluid that is located in front of the lens and can be replaced if it is lost.
  • It is a clear fluid that is produced by the lacrimal glands and cannot be replaced if it is lost.
  • It is a clear, watery fluid surrounding the eye and can be replaced if it is lost.
  • It is a clear, jellylike fluid near the back of the eye that cannot be replaced if it is lost.
    It is a clear, jellylike fluid near the back of the eye that cannot be replaced if it is lost.

A 52-year-old unrestrained female struck the steering wheel with her face when her truck collided with another vehicle. She has obvious swelling to her face and several dislodged teeth. A visual exam of her mouth reveals minimal bleeding. She is conscious and alert with a blood pressure of 130/80 mm Hg, a pulse of 110 beats/min, and respirations of 22 breaths/min with adequate tidal volume. You should:

  • assist ventilations with a BVM device, immobilize her spine, suction her oropharynx for 30 seconds, and transport.
  • fully immobilize her spine, attempt to locate the dislodged teeth, suction as needed, and transport.
  • fully immobilize her spine, irrigate her empty tooth sockets, attempt to locate the dislodged teeth, and transport.
  • apply oxygen via a nonrebreathing mask, suction her airway as needed, disregard the dislodged teeth, and transport.
    fully immobilize her spine, attempt to locate the dislodged teeth, suction as needed, and transport.

Significant trauma to the face should increase the EMT’s index of suspicion for a(n):

  • spinal column injury.
  • displaced mandible.
  • airway obstruction.
  • basilar skull fracture.
    spinal column injury.

Following blunt trauma to the face, a 21-year-old male complains of a severe headache and decreased ability to move his eyes. This patient’s clinical presentation is MOST consistent with:

  • a blowout fracture.
  • a lacerated globe.
  • optic vessel compression.
  • a ruptured eyeball.
    a blowout fracture.

A factory worker was splashed in the eyes with a strong acid chemical. He complains of intense pain and blurred vision. Your ambulance does not carry bottles of sterile saline or water. You should:

  • flush both eyes with an alcohol-based solution and transport.
  • mix baking soda with water and irrigate his eyes with the solution.
  • irrigate both eyes continuously for 20 minutes with plain water.
  • neutralize the acid chemical in his eye with an alkaline chemical.
    irrigate both eyes continuously for 20 minutes with plain water.

The MOST significant complication associated with facial injuries is:

  • damage to the eyes.
  • mandibular immobility.
  • airway compromise.
  • cervical spine injury.
    airway compromise.

The Adam’s apple is:

  • below the thyroid cartilage and forms the upper part of the trachea.
  • the small indentation in between the thyroid and cricoid cartilages.
  • the upper part of the larynx that is formed by the thyroid cartilage.
  • the lower part of the larynx that is formed by the cricoid cartilage.
    the upper part of the larynx that is formed by the thyroid cartilage.

When a light is shone into the pupil:

  • it should become smaller in size.
  • it should become larger in size.
  • the opposite pupil should dilate.
  • both pupils should dilate together.
    it should become smaller in size.

The superficial temporal artery can be palpated:

  • at the angle of the jaw.
  • just anterior to the tragus.
  • slightly above the ear.
  • over the mastoid process.
    just anterior to the tragus.

If your patient swallows blood following facial trauma, there is an increased risk of __.

  • vomiting
  • hypotension
  • altered LOC
  • GI trauma
    vomiting

Bleeding from soft-tissue injuries to the face is MOST effectively controlled with:

  • ice packs and elevation of the patient’s head.
  • pressure dressings and chemical ice packs.
  • digital pressure to an adjacent pulse point.
  • direct pressure using dry, sterile dressings.
    direct pressure using dry, sterile dressings.

A 30-year-old female presents with redness, inflammation, and pain to her left eye. During your assessment, you note that she is having difficulty keeping her eyes open. You should suspect that she is experiencing:

  • acute retinitis.
  • a corneal abrasion.
  • conjunctivitis.
  • a detached retina.
    conjunctivitis.

Which of the following acts as a shock absorber for the CNS?

  • Pia mater
  • Cerebrospinal fluid
  • Fascia
  • Dura meter
    Cerebrospinal fluid

Accounting for approximately 80% of all skull fractures, which of the following often present with no physical signs?

  • Basilar skull fractures
  • Depressed skull fracture
  • Linear skull fractures
  • Open skull fractures
    Linear skull fractures

Place a clean dressing over the bloody one.
You are treating a patient who might have a skull fracture. What should you do if a dressing you have applied to a head wound becomes soaked?

  • Replace the dressing with a fresh one.
  • Remove the dressing and apply an occlusive dressing.
  • Place a clean dressing over the bloody one.
  • Apply direct pressure and add a compression dressing.

Cushing triad in a patient is a sign of which of the following?

  • Concussion
  • Spinal cord damage
  • Intracranial pressure
  • Blood clot in the brain
    Intracranial pressure

While performing a secondary assessment of a patient who was hit with a tire iron on the side of the head, you find a depressed area above the patient’s left ear. This indicates that the patient could have which of the following?

  • A subarachnoid hemorrhage
  • An intracerebral hematoma
  • A subdural hematoma
  • An epidural hematoma
    An epidural hematoma

When securing a patient to a backboard, what area of the body should you secure last?

  • Head
  • Upper legs
  • Pelvis
  • Upper torso
    Head

The time between an initial period of unconsciousness and a subsequent loss of consciousness is referred to as what?

  • Lucid interval
  • Danger zone
  • Recognition period
  • Coherent stage
    Lucid interval

You are reassessing a patient and you find that her left pupil is dilated and fixed. What does this indicate?

  • Increased intracranial pressure
  • Decreased blood pressure
  • Drug abuse
  • Increased blood pressure
    Increased intracranial pressure

Battle sign is an indication of which of the following?

  • Concussion
  • Contusion
  • Skull fracture
  • Secondary injury
    Skull fracture

You are treating a patient who went face-first through a windshield. She has extensive head injuries and is displaying hypertension, bradycardia, and Cheyne-Stokes respirations. Which of the following should you suspect?

  • C6
  • Cushing triad
  • Cerebral edema
  • Concussion
    Cushing triad

An indicator of an expanding intracranial hematoma or rapidly progressing brain swelling is:

  • an acute increase in the patient’s pulse rate.
  • acute unilateral paralysis following the injury.
  • a rapid deterioration of neurologic signs.
  • a progressively lowering blood pressure.
    a rapid deterioration of neurologic signs.

The five sections of the spinal column, in descending order, are the:

  • coccygeal, sacral, lumbar, thoracic, and cervical.
  • cervical, coccygeal, thoracic, sacral, and lumbar.
  • thoracic, cervical, lumbar, coccygeal, and sacral.
  • cervical, thoracic, lumbar, sacral, and coccygeal.
    cervical, thoracic, lumbar, sacral, and coccygeal.

A patient with a head injury presents with abnormal flexion of his extremities. What numeric value should you assign to him for motor response?

  • 2
  • 5
  • 4
  • 3
    3

The ideal procedure for moving an injured patient from the ground to a backboard is:

  • the use of a scoop stretcher.
  • the direct patient carry.
  • the four-person log roll.
  • the clothes drag.
    the four-person log roll.

Which of the following breathing patterns is MOST indicative of increased intracranial pressure?

  • Increased rate with a normal inspiratory time and a prolonged expiratory time
  • Irregular rate, pattern, and volume of breathing with intermittent periods of apnea
  • Increased rate and depth with the distinct odor of acetone on the patient’s breath
  • Slow, shallow, occasional gasps that progress to prolonged periods of apnea
    Irregular rate, pattern, and volume of breathing with intermittent periods of apnea

What part of the nervous system controls the body’s voluntary activities?

  • Autonomic
  • Sensory
  • Central
  • Somatic
    Somatic

You should be MOST suspicious that a patient has experienced a significant head injury if his or her pulse is:

  • irregular.
  • weak.
  • slow.
  • rapid.
    slow.

When immobilizing a trauma patient’s spine, the EMT manually stabilizing the head should not let go until:

  • the patient has been secured to the ambulance stretcher.
  • an appropriately sized cervical collar has been applied.
  • the patient has been completely secured to the backboard.
  • the head has been stabilized with lateral immobilization.
    the patient has been completely secured to the backboard.

During your primary assessment of a 19-year-old unconscious male who experienced severe head trauma, you note that his respirations are rapid, irregular, and shallow. He has bloody secretions draining from his mouth and nose. You should:

  • assist his ventilations with a BVM.
  • suction his oropharynx for up to 15 seconds.
  • immobilize his spine and transport immediately.
  • pack his nostrils to stop the drainage of blood.
    suction his oropharynx for up to 15 seconds.

Which of the following statements regarding secondary brain injury is correct?

  • Signs are often present immediately after an impact to the head.
  • It results from direct brain trauma following an impact to the head.
  • Hypoxia and hypotension are the two most common causes of secondary brain injury.
  • Because cerebral edema develops quickly, it is considered to be a primary brain injury.
    Hypoxia and hypotension are the two most common causes of secondary brain injury.

The central nervous system (CNS) is composed of the:

  • cerebrum and meninges.
  • cerebellum and brain.
  • meninges and spinal cord.
  • brain and spinal cord.
    brain and spinal cord.

In contrast to a cerebral concussion, a cerebral contusion:

  • results from a laceration to the brain tissue.
  • does not cause pressure within the skull.
  • involves physical injury to the brain tissue.
  • usually does not cause a loss of consciousness.
    involves physical injury to the brain tissue.

A tight-fitting motorcycle helmet should be left in place unless:

  • the patient complains of severe neck or back pain.
  • the patient must be placed onto a long backboard.
  • the helmet is equipped with a full face shield or visor.
  • it interferes with your assessment of the airway.
    it interferes with your assessment of the airway.

Hyperextension injuries of the spine are MOST commonly the result of:

  • diving.
  • falls.
  • compression.
  • hangings.
    hangings.

When assessing a patient with a head injury, you note the presence of thin, bloody fluid draining from his right ear. This indicates:

  • rupture of the tympanic membrane following diffuse impact to the head.
  • fractures to the internal structures of the ear following direct trauma.
  • a linear skull fracture and a significant increase in intracranial pressure.
  • significant pressure and bleeding in between the skull and dura mater.
    rupture of the tympanic membrane following diffuse impact to the head.

A man jumped from the roof of his house and landed on his feet. He complains of pain to his heels, knees, and lower back. This mechanism of injury is an example of:

  • axial loading.
  • hyperflexion.
  • distraction.
  • hyperextension.
    axial loading.

Common signs and symptoms of a serious head injury include all of the following, EXCEPT:

  • decerebrate posturing.
  • a rapid, thready pulse.
  • CSF leakage from the ears.
  • widening pulse pressure.
    a rapid, thready pulse.

The Glasgow Coma Scale (GCS) is used to assess:

  • eye opening, verbal response, and motor response.
  • mental status, eye opening, and respiratory rate.
  • sensory response, pupil reaction, and heart rate.
  • verbal response, eye opening, and mental status.
    eye opening, verbal response, and motor response.

Once a cervical collar has been applied to a patient with a possible spinal injury, it should not be removed unless:

  • lateral immobilization has been applied.
  • it causes a problem managing the airway.
  • sensory and motor functions remain intact.
  • the patient adamantly denies neck pain.
    it causes a problem managing the airway.

When activated, the sympathetic nervous system produces all of the following effects, EXCEPT:

  • shunting of blood to vital organs.
  • pupillary constriction.
  • dilation of the bronchiole smooth muscle.
  • increase in heart rate.
    pupillary constriction.

What is ventilation?

  • The process of removing carbon dioxide and waste products from the body
  • The exchange of gases across the alveoli of the lungs
  • The process of delivering oxygen to the cells
  • The body’s ability to move air in and out of the chest and lung tissue
    The body’s ability to move air in and out of the chest and lung tissue

When a patient sustains a spinal cord injury above the C3 region, which of the following is likely to happen?

  • Loss of the ability to breathe due to loss of ability of phrenic nerves to function
  • Cardiogenic shock
  • Severe pain radiating from C3 down to L1
  • Belly breathing due to the loss of power to move the intercoastal muscles
    Loss of the ability to breathe due to loss of ability of phrenic nerves to function

You are assessing a patient who sustained a blunt force chest injury during a motorcycle accident. There are no obvious signs of external bleeding, but the patient is hypovolemic and there are decreased breath sounds on the left side. Which of the following should you suspect?

  • Hemopneumothorax
  • Traumatic asphyxia
  • Hemothorax
  • Pneumothorax
    Hemothorax

Which of the following are you likely to see in flail chest?

  • Paradoxical motion
  • Ventricular fibrillation
  • Myocardial contustion
  • Belly breathing
    Paradoxical motion

What are the two most common injuries caused by penetrating chest trauma?

  • Commotio cordis and hemopneumothorax
  • Pulmonary and myocardial contusion
  • Open pneumothorax and cardiac tamponade
  • Flail chest and simple pneumothorax
    Open pneumothorax and cardiac tamponade

What is the best position in which to place a patient when you want to assess for jugular vein distention?

  • Modified fowler
  • Prone
  • Sitting at a 45-degree angle
  • Supine
    Sitting at a 45-degree angle

You are assessing a patient and notice he has a collapsed jugular vein. What does this indicate?

  • Tension pneumothorax
  • Hemothorax
  • Commotio cordis
  • Cardiac tamponade
    Hemothorax

You are treating a patient who fell hard on her right side when she fell off a bike. Exposure of the patient’s chest reveals a large bruise on the lateral aspect of the right side of the chest. When you palpate the area, the patient yells out in extreme pain and states that she cannot take a deep breath. What condition should you suspect?

  • Rib fractures
  • Open pneumothorax
  • Spontaneous pneumothorax
  • Cardiac tamponade
    Rib fractures

Which of the following blood vessels can be lacerated by a fractured rib?

  • Jugular vein
  • Iliac veins
  • Aorta
  • Brachial artery
    Aorta

In which condition is it recommended you use positive-pressure ventilation?

  • Pneumothorax
  • Tension pneumothorax
  • Traumatic asphyxia
  • Flail chest
    Flail chest

Closed chest injuries are typically caused by _.

  • high-velocity weapons
  • penetrating trauma
  • blunt trauma
  • flying debris
    blunt trauma

A man called EMS 12 hours after injuring his chest. Your assessment reveals a flail segment to the right side of the chest. The patient is experiencing respiratory distress and his oxygen saturation is 78%. His breath sounds are equal bilaterally and his jugular veins are normal. You should suspect:

  • traumatic asphyxia.
  • tension pneumothorax.
  • pulmonary contusion.
  • massive hemothorax.
    pulmonary contusion.

Patients with chest injuries will often present with _.

  • Cheyne-Stokes respirations
  • tachypnea
  • Kussmaul respirations
  • agonal respirations
    tachypnea

You arrive at the scene of a major motor vehicle crash. The patient, a 50-year-old female, was removed from her vehicle prior to your arrival. Bystanders who removed her state that she was not wearing a seatbelt. The patient is unresponsive, tachycardic, and diaphoretic. Your assessment reveals bilaterally clear and equal breath sounds, a midline trachea, and collapsed jugular veins. You should be MOST suspicious that this patient has experienced a:

  • massive hemothorax.
  • pericardial tamponade.
  • laceration of the aorta.
  • tension pneumothorax.
    laceration of the aorta.

Immediately life-threatening chest injuries must be found and managed during the _.

  • scene size-up
  • secondary assessment
  • primary assessment
  • patient history
    primary assessment

Which of the following is most likely to cause immediate death?

  • Myocardial contusion
  • Aortic rupture
  • Pulmonary contusion
  • Aortic dissection
    Aortic rupture

When a person is lying supine at the end of exhalation, the diaphragm:

  • contracts and flattens inferiorly.
  • may rise as high as the nipple line.
  • is less prone to penetrating trauma.
  • descends below the level of the navel.
    may rise as high as the nipple line.

An open pneumothorax occurs when:

  • a fractured rib perforates the tissue of the lung.
  • air enters the pleural space from outside the body.
  • extreme pleural pressure causes the lung to rupture.
  • air enters the pleural space from a perforated lung.
    air enters the pleural space from outside the body.

Following a stab wound to the left anterior chest, a 25-year-old male presents with a decreased level of consciousness and signs of shock. Which of the following additional assessment findings should increase your index of suspicion for a cardiac tamponade?

  • Engorged jugular veins
  • A rapid, irregular pulse
  • Diminished breath sounds
  • Widening pulse pressure
    Engorged jugular veins

If a patient with a chest injury is only able to inhale small amounts of air per breath, he or she:

  • will maintain adequate minute volume if his or her respiratory rate stays the same.
  • often breathes at a slower rate because of lung damage caused by the injury.
  • must increase his or her respiratory rate to maintain adequate minute volume.
  • will eliminate more carbon dioxide than if he or she were breathing deeply.
    must increase his or her respiratory rate to maintain adequate minute volume.

In order to avoid exacerbating a patient’s injury, it is especially important to use extreme caution when providing positive-pressure ventilation to patients with a:

  • pneumothorax.
  • cardiac tamponade.
  • myocardial contusion.
  • flail chest.
    pneumothorax.

A simple pneumothorax:

  • is commonly caused by blunt chest trauma.
  • heals on its own without any treatment.
  • often has a nontraumatic cause.
  • is caused by penetrating chest trauma.
    is commonly caused by blunt chest trauma.

A 28-year-old male was struck in the chest with a baseball bat during an altercation. He is conscious and alert and complains of severe chest pain. Your assessment reveals a large area of ecchymosis over the sternum and a rapid, irregular pulse. In addition to providing supplemental oxygen, you should:

  • apply bulky dressings to the sternum.
  • prepare for immediate transport.
  • apply an AED and assess his BP.
  • determine if he has cardiac problems.
    prepare for immediate transport.

Children are often “belly breathers” because _.

  • they are routinely hypoxic
  • they are consciously controlling ventilations
  • their intercostal muscles are not developed
  • their diaphragm is not functional
    their intercostal muscles are not developed

While jogging, a 19-year-old male experienced an acute onset of shortness of breath and pleuritic chest pain. He is conscious and alert with stable vital signs. Your assessment reveals that he has diminished breath sounds over the left side of the chest. You should:

  • circumferentially tape a dressing around his chest.
  • recognize that he needs a needle decompression.
  • immediately perform a rapid head-to-toe exam.
  • administer oxygen and transport to the hospital.
    administer oxygen and transport to the hospital.

You have sealed the open chest wound of a 40-year-old male who was stabbed in the anterior chest. Your reassessment reveals that he is experiencing increasing respiratory distress and tachycardia, and is developing cyanosis. You should:

  • begin ventilatory assistance.
  • partially remove the dressing.
  • call for a paramedic ambulance.
  • begin rapid transport at once.
    partially remove the dressing.

A spinal cord injury at the level of C7 would MOST likely result in:

  • paralysis of all the respiratory muscles.
  • paralysis of the intercostal muscles.
  • paralysis of the diaphragm.
  • immediate cardiac arrest.
    paralysis of the intercostal muscles.

A flail chest occurs when:

  • a segment of fractured ribs bulges during the inhalation phase.
  • a segment of the chest wall is detached from the thoracic cage.
  • more than three ribs are fractured on the same side of the chest.
  • multiple ribs are fractured on both sides of the thoracic cage.
    a segment of the chest wall is detached from the thoracic cage.

You respond to a residence for a 40-year-old female who was assaulted by her husband; the scene has been secured by law enforcement. Upon your arrival, you find the patient lying supine on the floor in the kitchen. She is semiconscious with severely labored breathing. Further assessment reveals a large bruise to the left anterior chest, jugular venous distention, and unilaterally absent breath sounds. As your partner is supporting her ventilations, you should:

  • obtain a set of baseline vital signs.
  • perform a focused secondary exam.
  • immediately request ALS support.
  • insert an oropharyngeal airway.
    immediately request ALS support.

The thoracic cavity is separated from the abdominal cavity by the:

  • intercostal margin.
  • diaphragm.
  • anterior rib cage.
  • costovertebral angle.
    diaphragm.

Bruising over the right upper quadrant could indicate injury to which of the following?

  • Liver
  • Appendix
  • Kidney
  • Colon
    Liver

You should have an index of suspicion that a patient with pain referred to the right shoulder has an injury to which organ?

  • Intestine
  • Bladder
  • Liver
  • Kidney
    Liver

Which of the following are considered solid organs?

  • Pancreas and spleen
  • Liver and intestines
  • Kidneys and bladder
  • Spleen and stomach
    Pancreas and spleen

Which of the following signs would indicate that an injury to the kidney has occurred?

  • Hemoptysis
  • Hematemesis
  • Hematuria
  • Hematoma
    Hematuria

You are assessing a patient who reports pain that is tearing and describes it as going from the abdomen posteriorly. What should you suspect?

  • Gallbladder inflammation
  • Kidney damage
  • Dissecting abdominal aneurysm
  • Ruptured appendix
    Dissecting abdominal aneurysm

What is the major cause of death following injury to a hollow organ in the abdomen?

  • Sepsis
  • Evisceration
  • Rigidity
  • Internal hemorrhage
    Sepsis

What is the purpose of wrapping a sexual assault victim in a burn sheet?

  • To keep the patient calm (swaddling)
  • To protect the patient’s privacy
  • To maintain body temperature as a result of shock
  • To preserve potential evidence
    To preserve potential evidence

Which of the following is an indication of a spleen injury?

  • Radiating pain from left upper quadrant down through the flank
  • Hematuria
  • Referred left shoulder pain
  • Referred right shoulder pain
    Referred left shoulder pain

Which of the following statements is true?

  • There are four levels of velocity.
  • Low-velocity injuries produce extensive temporary would channels.
  • The velocity of an object is not a critical determination in your assessment.
  • The higher the velocity of a projectile, the larger the cavity it produces.
    The higher the velocity of a projectile, the larger the cavity it produces.

Where is abdominal injury most likely to occur in young children?

  • Appendix
  • Liver
  • Gallbladder
  • Fractured ribs
    Liver

All of the following are hollow abdominal organs, EXCEPT the:

  • ureters.
  • bladder.
  • spleen.
  • stomach.
    spleen.

Which of the following organs would MOST likely bleed profusely when injured?

  • Intestine
  • Stomach
  • Liver
  • Bladder
    Liver

You are transporting a patient with blunt abdominal trauma. The patient is unstable and is experiencing obvious signs and symptoms of shock. Your estimated time of arrival at the hospital is less than 10 minutes. After treating the patient appropriately, you should:

  • perform a comprehensive secondary assessment.
  • begin documenting the call on the patient care form.
  • closely monitor him and reassess him frequently.
  • forgo the hospital radio report because of his condition.
    closely monitor him and reassess him frequently.

Difficulty breathing and a sunken appearance of the anterior abdominal wall is MOST indicative of a ruptured:

  • aorta.
  • diaphragm.
  • stomach.
  • spleen.
    diaphragm.

You are transporting a 42-year-old male who experienced blunt abdominal trauma. He is receiving oxygen at 12 L/min via a nonrebreathing mask and full spinal precautions have been applied. During your reassessment, you note his level of consciousness has decreased and his respirations have become shallow. You should:

  • suction his oropharynx to ensure it is clear of secretions and then increase the oxygen flow rate to 15 L/min.
  • reassess his vital signs and then notify the receiving hospital of the change in his clinical status.
  • perform a comprehensive secondary assessment to determine why his clinical status has changed.
  • insert an airway adjunct if he will tolerate it and begin assisting his ventilations with a BVM.
    insert an airway adjunct if he will tolerate it and begin assisting his ventilations with a BVM.

Which of the following statements regarding abdominal eviscerations is correct?

  • The protruding organs should be kept warm and moist.
  • Most eviscerations occur to the left upper quadrant.
  • The organs should be replaced carefully to avoid heat loss.
  • Adherent material is preferred when covering an evisceration.
    The protruding organs should be kept warm and moist.

In pediatric patients, the liver and spleen are _.

  • more protected by the thorax compared to adults
  • larger in proportion to the abdomen
  • less likely to bleed when injured
  • smaller in proportion to the abdomen
    larger in proportion to the abdomen

Compression injury is most likely due to which of the following?

  • Hollow-organ rupture
  • Improperly placed lab belt
  • Ejection of unrestrained driver
  • Stabbing
    Improperly placed lab belt

Because the depth of an open abdominal wound is often difficult to determine:

  • the abdomen must be vigorously palpated.
  • prompt transport to the hospital is essential.
  • vital signs should be monitored frequently.
  • the EMT must perform a thorough exam.
    prompt transport to the hospital is essential.

A 54-year-old male experienced an avulsion to his penis when his foreskin got caught in the zipper of his pants. He was able to unzip his pants and remove the foreskin prior to your arrival. Your assessment reveals that he is in severe pain and that the avulsion is bleeding moderately. The MOST appropriate treatment for this patient includes:

  • applying direct pressure with a dry, sterile dressing.
  • administering 100% oxygen via a nonrebreathing mask.
  • requesting a paramedic to administer pain medication.
  • covering the avulsion with moist, sterile dressings.
    applying direct pressure with a dry, sterile dressing.

Your documentation on a sexual assault victim should _.

  • be objective and factual
  • describe the status of the suspect(s)
  • include your opinion of the nature of the incident
  • be subjective and summarize the crime
    be objective and factual

Placing a pregnant patient in a supine position during the third trimester of pregnancy:

  • is recommended if the patient has severe abdominal discomfort.
  • results in spontaneous urinary incontinence if the bladder is full.
  • may decrease the amount of blood that returns to the heart.
  • often causes hypotension secondary to cardiac compression.
    may decrease the amount of blood that returns to the heart.

Which of the following is true regarding injury to the kidneys?

  • Kidney injuries are rarely caused by blunt trauma.
  • Injury to the kidneys usually indicates injury to other organs.
  • The kidneys are not well protected.
  • Only minimal force is needed to damage the kidneys.
    Injury to the kidneys usually indicates injury to other organs.

Abdominal pain, vomiting, and fever are most likely due to _.

  • evisceration
  • infection
  • hypovolemia
  • hemorrhage
    infection

You should suspect a kidney injury anytime the patient presents with _.

  • dyspnea
  • abdominal distention
  • a hematoma in the flank region
  • nausea
    a hematoma in the flank region

Injuries to the external male genitalia _.

  • frequently lead to hypovolemic shock
  • are often life-threatening
  • are rarely life-threatening
  • usually result in permanent damage
    are rarely life-threatening

When a patient stiffens the muscles of the abdomen, it is known as _.

  • distention.
  • instability.
  • guarding.
  • crepitus.
    guarding.

When a hollow organ is punctured during a penetrating injury to the abdomen:

  • it commonly protrudes through the injury site.
  • it will bleed profusely and rapidly cause shock.
  • peritonitis may not develop for several hours.
  • the abdomen will become instantly distended.
    peritonitis may not develop for several hours.

When should you visually inspect the external genitalia on your patient?

  • Only when there is a complaint of severe pain or other injury
  • Only when ordered by medical direction
  • Always during the secondary assessment
  • Anytime the patient agrees to treatment and transport
    Only when there is a complaint of severe pain or other injury

Your presence is requested by law enforcement to assess a 33-year-old female who was sexually assaulted. The patient is conscious and obviously upset. As you are talking to her, you note an impressive amount of blood on her clothes in the groin area. Her blood pressure is 98/58 mm Hg, her pulse is 130 beats/min, and her respirations are 24 breaths/min. You should:

  • arrange for a rape crisis center representative to speak with the patient.
  • allow her to change her clothes and take a shower before you transport.
  • control any external bleeding, administer oxygen, and transport at once.
  • visualize the vaginal area and pack the vagina with sterile dressings.
    control any external bleeding, administer oxygen, and transport at once.

When assessing distal circulation in a patient’s lower extremities, which pulse should you palpate?

  • Popliteal
  • Iliac
  • Dosalis pedis
  • Femoral
    Popliteal

Which MOI causes a fracture or dislocation at a distant point?

  • Indirect force
  • High-impact injury
  • Direct blow
  • Twisting force
    Indirect force

What is the most reliable indicator of an underlying fracture?

  • Deformity
  • Crepitus
  • Guarding
  • Point tenderness
    Point tenderness

A disruption of a joint in which the bone ends are no longer in contact is known as what?

  • Strain
  • Fracture
  • Dislocation
  • Sprain
    Dislocation

In the musculoskeletal injury grading system, under which category would you place a laceration of a major nerve or blood vessel?

  • Moderate
  • Serious
  • Critical
  • Severe
    Serious

Which of the following would you use to stabilize an AC separation?

  • Sling and swathe
  • Traction splint
  • Zippered air splint
  • Rigid splint
    Sling and swathe

You are attending to a patient with a nondisplaced elbow fracture. She has a strong pulse and good capillary refill. How should you address this type of injury?

  • Apply ice to reduce swelling.
  • Apply a sling and swathe to reduce pain.
  • Splint from the shoulder joint to the wrist joint to stabilize the entire arm.
  • Apply a Sager traction splint to stabilize the joint.
    Splint from the shoulder joint to the wrist joint to stabilize the entire arm.

What is the primary goal of in-line traction?

  • Reduce swelling
  • Minimize pain
  • Avoid further neurovascular compromise
  • Prevent permanent disability
    Avoid further neurovascular compromise

Which sign/symptom would give you a high index of suspicion that a patient may have compartment syndrome?

  • Referred pain
  • Limb deformity
  • Hypersensation
  • Disproportionate pain
    Disproportionate pain

Which of the following is a drawback of an air splint?

  • Temperature changes affect air pressure in the splint.
  • It restricts distal blood flow.
  • It is complicated to use and requires multiple EMTs.
  • It does not provide uniform contact, so bone movement is possible.
    Temperature changes affect air pressure in the splint.

During your assessment of a 29-year-old female with significant deformity to her left elbow, you are unable to palpate a radial pulse. Your transport time to the hospital is approximately 40 minutes. You should:

  • splint the elbow in the position of deformity and transport immediately.
  • carefully straighten the injured arm and secure it with padded board splints.
  • apply gentle manual traction in line with the limb and reassess for a pulse.
  • make two or three attempts to restore distal circulation by manipulating the elbow.
    apply gentle manual traction in line with the limb and reassess for a pulse.

Which of the following musculoskeletal injuries would pose the greatest threat to a patient’s life?

  • Bilateral femur fractures
  • Nondisplaced long bone fractures
  • Pelvic fracture with hypotension
  • An amputated extremity
    Pelvic fracture with hypotension

A “hip” fracture is actually a fracture of the:

  • proximal femur.
  • femoral shaft.
  • pubic symphysis.
  • pelvic girdle.
    proximal femur.

The musculoskeletal system refers to the:

  • connective tissue that supports the skeleton.
  • bones and voluntary muscles of the body.
  • nervous system’s control over the muscles.
  • involuntary muscles of the nervous system.
    bones and voluntary muscles of the body.

With regard to musculoskeletal injuries, the zone of injury is defined as the:

  • part of the body that sustained secondary injury.
  • area of soft-tissue damage surrounding the injury.
  • exact part of the bone or joint that was disrupted.
  • area of obvious deformity over the site of impact.
    area of soft-tissue damage surrounding the injury.

A supracondylar or intercondylar fracture is also known as a fracture of the:

  • olecranon process.
  • radial head.
  • distal humerus.
  • proximal radius.
    distal humerus.

In general, musculoskeletal injuries should be splinted before moving the patient unless:

  • deformity and swelling are present.
  • the patient is in severe pain.
  • the patient is clinically unstable.
  • transport time is less than 15 minutes.
    the patient is clinically unstable.

Which of the following statements regarding compartment syndrome is correct?

  • In most cases, compartment syndrome develops within a few minutes after an injury.
  • Most cases of compartment syndrome occur following a severe fracture of the pelvis.
  • Compartment syndrome typically develops within 6 to 12 hours after an injury.
  • Compartment syndrome occurs because of increased pressure within the bone cavity.
    Compartment syndrome typically develops within 6 to 12 hours after an injury.

Skeletal muscle is attached to the bone by tough, ropelike, fibrous structures called:

  • cartilage.
  • fascia.
  • tendons.
  • ligaments.
    tendons.

The MOST significant hazard associated with splinting is:

  • reduction in circulation distal to the injury site.
  • aggravation of the injury or worsened pain.
  • delaying transport of a critically injured patient.
  • compression of nerves, tissues, and vasculature.
    delaying transport of a critically injured patient.

When caring for a patient with a possible fracture of the scapula, the EMT should:

  • carefully assess the patient for breathing problems.
  • assume that minimal force was applied to the back.
  • apply rigid board splints across the chest and back.
  • recognize that scapular fractures are life threatening.
    carefully assess the patient for breathing problems.

An open fracture is MOST accurately defined as a fracture in which:

  • a bullet shatters the underlying bone.
  • a large laceration overlies the fracture.
  • bone ends protrude through the skin.
  • the overlying skin is no longer intact.
    the overlying skin is no longer intact.

A 17-year-old female dislocated her patella while playing soccer. Her knee is flexed and she complains of severe pain. You should:

  • keep her knee flexed and secure it with padded board splints.
  • flex her knee slightly more and assess for distal circulation.
  • make one attempt to return the patella to its normal position.
  • gently straighten her knee and apply a padded board splint.
    keep her knee flexed and secure it with padded board splints.

In which of the following situations should the EMT splint an injured limb in the position of deformity?

  • When distal circulation and neurological functions are absent
  • If transport time to the hospital is greater than 20 to 30 minutes
  • If a traction splint will be used to immobilize the injured extremity
  • If resistance is encountered or the patient experiences severe pain
    If resistance is encountered or the patient experiences severe pain

A 22-year-old female was ejected from her car after striking a tree head-on. As you approach her, you note obvious closed deformities to both of her femurs. She is not moving and does not appear to be conscious. You should:

  • apply manual stabilization to both of her femurs.
  • administer oxygen and perform a rapid assessment.
  • stabilize her head and perform a primary assessment.
  • assess for a carotid pulse and assist her ventilations.
    stabilize her head and perform a primary assessment.

A 54-year-old male accidentally shot himself in the leg while cleaning his gun. Your assessment reveals a small entrance wound to the medial aspect of his right leg. The exit wound is on the opposite side of the leg and is actively bleeding. The patient complains of numbness and tingling in his right foot. You should:

  • control the bleeding and cover the wound with a sterile dressing.
  • assess distal pulses as well as sensory and motor functions.
  • gently manipulate the injured leg until the numbness dissipates.
  • manually stabilize the leg above and below the site of injury.
    control the bleeding and cover the wound with a sterile dressing.

A fracture is MOST accurately defined as a(n):

  • disruption in the midshaft of a bone.
  • total loss of function in a bone.
  • break in the continuity of the bone.
  • abnormality in the structure of a bone.
    break in the continuity of the bone.

Crepitus and false motion are:

  • positive indicators of a fracture.
  • most common with dislocations.
  • only seen with open fractures.
  • indicators of a severe sprain.
    positive indicators of a fracture.

During your secondary assessment of a 30-year-old male who fell 25 feet, you note crepitus when palpating his pelvis. Your partner advises you that the patient’s blood pressure is 80/50 mm Hg and his heart rate is 120 beats/min and weak. After completing your assessment, you should:

  • log roll the patient onto a long backboard and transport at once.
  • perform a focused physical exam with emphasis on the pelvis.
  • defer spinal immobilization and transport to a trauma center.
  • stabilize the pelvis with a pelvic binder and protect the spine.
    stabilize the pelvis with a pelvic binder and protect the spine.

Which of the following musculoskeletal injuries would MOST likely result in deformity?

  • Hairline fracture
  • Displaced fracture
  • Severe strain
  • Moderate sprain
    Displaced fracture

Hypothermia is diagnosed when the core body temperature falls below what temperature?

  • 92°F (33.3°C)
  • 95°F (35°C)
  • 90°F (32.2°C)
  • 98°F (36.7°C)
  • 95°F (35°C)

Shivering stops and muscle activity ceases once the body’s core temperature reaches what?

  • 95°F (35°C)
  • 85°F (29.4°C)
  • 90°F (32.2°C)
  • 80°F (26.7°C)
    90°F (32.2°C)

Rough handling of a patient with severe hypothermia may cause which of the following dysrhythmias?

  • Sinus tachycardia
  • Sinus bradycardia
  • Asystole
  • Ventricular fibrillation
    Ventricular fibrillation

In the context of an environment that is 90°F with 85% humidity, what does moist, pale, cool skin indicate?

  • The body can no longer regulate core temperature.
  • Core body temperature and ambient temperature are equal.
  • The patient is hypothermic.
  • There has been excessive fluid and salt loss.
    There has been excessive fluid and salt loss.

What is often the first sign of heat stroke?

  • Low pulse rate and blood pressure
  • Change in behavior
  • Muscle cramps
  • Profuse sweating
    Change in behavior

In which of the following environmental emergencies are you most likely to encounter laryngospasm?

  • After being bitten by a coral snake
  • Drowning
  • Hyperthermia
  • A tick bite
    Drowning

What is the most dangerous, and most common, emergency in scuba diving?

  • Diving reflex
  • Air embolism
  • Decompression sickness
  • Drowning
    Air embolism

You are treating a patient who was climbing a 10,000-foot mountain when he began to experience a severe, constant, throbbing headache; ataxia; and extreme fatigue. He is nauseated and, as you assess him, he loses consciousness. Which of the following should you suspect?

  • Mountain sickness
  • HACE
  • Rocky Mountain spotted fever
  • HAPE
    HACE

You suspect that the patient you are treating was bitten by a rattlesnake. What should you do first?

-Catch the snake so you can take it to the ED.

  • Apply ice to the wound.
  • Place the patient is a supine position.
  • Apply suction to the wound to suck out the venom.
    Place the patient is a supine position.

A person who stands outside in windy, wintry weather and wears only lightweight clothing is experiencing heat loss mainly by what method?

  • Conduction
  • Convection
  • Evaporation
  • Respiration
    Convection

You receive a call to a residence for a sick patient. Upon your arrival, you find the patient, a 53-year-old diabetic male, lying down on his front porch. His wife tells you that he had been mowing the lawn in the heat for the past 3 hours. The patient is confused and has hot, moist skin. His pulse is weak and thready, and his blood pressure is 90/50 mm Hg. You should:

  • perform a head-to-toe assessment and look for signs of trauma.
  • load him into the ambulance and begin rapid cooling interventions.
  • place him in a sitting position and have him drink 1 L of water.
  • administer one tube of oral glucose and reassess his mental status.
    load him into the ambulance and begin rapid cooling interventions.

You respond to a local lake where a diver complains of difficulty breathing that occurred immediately after rapidly ascending from a depth of approximately 30 feet. On assessment, you note that he has cyanosis around his lips and has pink froth coming from his nose and mouth. You should:

  • suction his mouth and nose, apply high-flow oxygen, monitor the patient’s breath sounds for a pneumothorax, and contact medical control regarding transport to a recompression facility.
  • place him in a semi-sitting position, suction his mouth and nose, apply a continuous positive airway pressure (CPAP) device, and transport to the closest emergency department.
  • position him supine with his head elevated 30°, suction his mouth and nose, hyperventilate him with a bag-valve mask, and contact medical control for further guidance.
  • suction his mouth and nose, keep him supine and elevate his legs to prevent air bubbles from entering his brain, administer high-flow oxygen, and transport to a hyperbaric chamber.
    suction his mouth and nose, apply high-flow oxygen, monitor the patient’s breath sounds for a pneumothorax, and contact medical control regarding transport to a recompression facility.

You are transporting a 28-year-old man with a frostbitten foot. The patient’s vital signs are stable and he denies any other injuries or symptoms. The weather is treacherous and your transport time to the hospital is approximately 45 minutes. During transport, you should:

  • protect the affected part from further injury.
  • cover his foot with chemical heat compresses.
  • rewarm his foot in 102°F to 104°F (38.9°C to 40°C) water.
  • administer oxygen via a nonrebreathing mask.
    protect the affected part from further injury.

Common signs and symptoms of heat exhaustion include all of the following, EXCEPT:

  • nausea.
  • headache.
  • tachycardia.
  • hot, dry skin.
    hot, dry skin.

The venom of a black widow spider is toxic to the:

  • renal system.
  • respiratory system.
  • nervous system.
  • cardiovascular system.
    nervous system.

A 30-year-old male was rescued after being lost in the woods for approximately 18 hours. The outside temperature is 30°F (-1°C). He is immediately placed in the warmed ambulance, where you perform a primary assessment. He is unresponsive, pale, and apneic. You should:

  • apply chemical heat packs to his groin and axillae.
  • apply an AED and assess his cardiac rhythm.
  • assess for a carotid pulse for up to 60 seconds.
  • open his airway and give two rescue breaths.
    assess for a carotid pulse for up to 60 seconds.

Which of the following MOST accurately describes hyperthermia?

  • The body eliminates more heat than it can generate.
  • Heat evaporates a significant amount of body water.
  • The core body temperature exceeds 99.5°F (37°C).
  • The body is exposed to more heat than it can lose.
    The body is exposed to more heat than it can lose.

Hypothermia occurs when the core body temperature falls below:

  • 88°F (31°C).
  • 98°F (37°C).
  • 90°F (32°C).
  • 95°F (35°C).
    95°F (35°C).

The body’s natural cooling mechanism, in which sweat is converted to a gas, is called:

  • radiation.
  • evaporation.
  • conduction.
  • convection.
    evaporation.

In contrast to Lyme disease, Rocky Mountain spotted fever:

  • presents with flu-like symptoms and a bull’s-eye rash.
  • may be confused with rheumatoid arthritis.
  • causes painful joint swelling after a few days or weeks.
  • can cause paralysis and cardiorespiratory collapse.
    can cause paralysis and cardiorespiratory collapse.

Drowning is MOST accurately defined as:

  • water in the lungs following submersion in water.
  • death from suffocation after submersion in water.
  • temporary survival after submersion in water.
  • death beyond 24 hours after submersion in water.
    death from suffocation after submersion in water.

Burns associated with lightning strikes are typically:

  • superficial.
  • full-thickness.
  • third-degree.
  • partial-thickness.
    superficial.

When a warm hand is immersed in water that is 70°F (21°C), heat is transferred from the hand to the water through a process called:

  • convection.
  • evaporation.
  • conduction.
  • radiation.
    conduction.

Most of the serious injuries associated with scuba diving are caused by:

  • cold water temperature.
  • alcohol consumption.
  • too rapid of an ascent.
  • too rapid of a descent.
    too rapid of an ascent.

The EMT must assume that any unwitnessed water-related incident is accompanied by:

  • an air embolism.
  • possible spinal injury.
  • alcohol intoxication.
  • cold-water immersion.
    possible spinal injury.

The MOST prominent symptom of decompression sickness is:

  • tightness in the chest.
  • dizziness and nausea.
  • difficulty with vision.
  • abdominal or joint pain.
    abdominal or joint pain.

You and your partner respond to a park where several people were reportedly struck by lightning. When you arrive, you find three patients. The first patient is lying supine on the ground; he is unresponsive and does not appear to be breathing. The second patient is ambulatory, appears confused, and is holding his arm against his chest. The third patient is sitting on the ground holding the sides of his head. After calling for backup, you should:

  • immediately begin CPR on the unresponsive patient, but cease resuscitation efforts if there is no response after 5 minutes of treatment.
  • focus your initial treatment efforts on the patients who are conscious because the unresponsive patient is likely in irreversible cardiac arrest.
  • assess the unresponsive patient’s pulse, begin CPR starting with chest compressions if he is pulseless, and attach the AED as soon as possible.
  • recognize that the patients who are conscious are at high risk for developing cardiac arrest and quickly assess them for potentially life-threatening injuries.
    assess the unresponsive patient’s pulse, begin CPR starting with chest compressions if he is pulseless, and attach the AED as soon as possible.

A 31-year-old male was bitten on the leg by an unidentified snake. The patient is conscious and alert and in no apparent distress. Your assessment of his leg reveals two small puncture marks with minimal pain and swelling. In addition to administering oxygen and providing reassurance, further care for this patient should include:

  • supine positioning, splinting the leg, and transporting.
  • transporting only with close, continuous monitoring.
  • elevating the lower extremities and giving antivenin.
  • applying ice to the wound and transporting quickly.
    supine positioning, splinting the leg, and transporting.

Which of the following is an early sign of pit viper envenomation?

  • Signs and symptoms of hypoperfusion
  • General weakness and diaphoresis
  • Local swelling and ecchymosis
  • Syncope and bleeding at distal sites
    Local swelling and ecchymosis

All of the following terms refer to a body part that is cold but not frozen, EXCEPT:

  • frostbite.
  • trench foot.
  • immersion foot.
  • frostnip.
    frostbite.

Which of the following is a normal change in pregnancy?

  • Decreases in blood volume
  • Tightening of joints
  • Blood clotting decreases
  • Increased respiratory rates and decreased minute volumes
    Increased respiratory rates and decreased minute volumes

When transporting a pregnant patient in her third trimester, how should you position her?

  • On her right side
  • Supine
  • Fowler position
  • On her left side
    On her left side

When does the onset of labor begin?

  • When the woman has the urge to push
  • When the amniotic sac ruptures
  • When the fetus enters the birth canal
  • When uterine contractions begin
    When uterine contractions begin

If amniotic fluid is green, what does this signify?

  • Maternal infection
  • Premature rupture of the placenta
  • The presence of meconium
  • Prolonged gestation
    The presence of meconium

As you are delivering the head during birth, you observe the umbilical cord is wrapped once around the neck. What should you do?

  • Ask the patient not to push, and prepare for immediate transport.
  • Try to slip the cord gently over the head.
  • Clamp and cut the cord, and remove the cord from around the neck.
  • Pull on the cord to speed the delivery.
    Try to slip the cord gently over the head.

What would the Apgar score be for a newborn whose body is pink but whose hands and feet are cyanotic, has a pulse rate of 98 bpm, cries but does not recoil from stimulus, resists attempts to straighten hips and knees, and has slow respirations?

  • 4
  • 8
  • 6
  • 10
    6

If a newborn’s heart rate is 75 bpm, what should you do?

  • Keep the newborn warm and assess him/her continuously.
  • Begin assisted ventilation with a BVM and room air.
  • Begin chest compressions.
  • Begin assisted ventilation with a BVM and 100% oxygen.
    Begin assisted ventilation with a BVM and room air.

What is a primary characteristic of eclampsia?

  • Severe, persistant headache
  • Hypotension
  • Bleeding
  • Seizures
    Seizures

Which of the following is a sign that delivery is imminent?

  • Crowning
  • Bloody show
  • Contractions
  • Lightening
    Crowning

The answer to which of the following questions will help you determine whether delivery for this patient is imminent?

  • Is this your first baby?
  • Do you use drugs or drink alcohol?
  • Do you know if there is a chance of multiple deliveries?
  • Have you had a previous complicated pregnancy?
    Is this your first baby?

When preparing a pregnant patient for delivery, you should position her:

  • in a supine position with her legs spread.
  • in a sitting position with her hips elevated 12 inches.
  • on her left side with the right leg elevated.
  • on a firm surface with her hips elevated 2 to 4 inches.
    on a firm surface with her hips elevated 2 to 4 inches.

A 26-year-old female who is 34 weeks pregnant is experiencing a seizure. Her husband tells you that her blood pressure has been high and that she had been complaining of a headache for the past few days. You should:

  • insert an oral airway and ventilate her with a bag-valve mask.
  • place her on her side, administer high-flow oxygen, and transport.
  • elevate her legs to improve brain perfusion and keep her warm.
  • administer oral glucose for presumed hypoglycemia and transport.
    place her on her side, administer high-flow oxygen, and transport.

By the 20th week of pregnancy, the uterus is typically at or above the level of the mother’s:

  • superior diaphragm.
  • xiphoid process.
  • pubic bone.
  • belly button.
    belly button.

A pregnant trauma patient may lose a significant amount of blood before showing signs of shock because:

  • blood is shunted to the uterus and fetus during major trauma.
  • pregnancy causes vasodilation and a lower blood pressure.
  • pregnant patients can dramatically increase their heart rate.
  • pregnant patients have an overall increase in blood volume.
    pregnant patients have an overall increase in blood volume.

Following delivery of a full-term baby, you have properly cared for the baby and have clamped and cut the umbilical cord. During transport, you note that the mother is experiencing moderate vaginal bleeding. You should:

  • elevate her legs 6 to 8 inches and cover her with a blanket.
  • carefully insert a sterile trauma dressing into her vagina.
  • place her legs together and position her on her left side.
  • firmly massage the uterine fundus with a circular motion.
    firmly massage the uterine fundus with a circular motion.

A mother who is pregnant with her first baby is typically in the first stage of labor for approximately:

  • 16 hours.
  • 4 hours.
  • 10 hours.
  • 8 hours.
    16 hours.

You are attending to a 23-year-old female patient who is 16 weeks pregnant with her second child. The patient has apparently fallen and sustained an injury to her upper right arm. When you ask about the incident history, the patient is reluctant to explain what happened and becomes very quiet. Based on this information, you should:

  • immediately remove the patient from the environment and transport.
  • ask more detailed questions and press the issue until you have a more detailed understanding of the incident.
  • contact police and remain at the scene until they arrive and escort you to the hospital.
  • continue your care in a private area, document any details regarding the scene and the incident, and reassure her as you provide care.
    continue your care in a private area, document any details regarding the scene and the incident, and reassure her as you provide care.

You and your partner are both male and are attending to a 28-year-old female patient complaining of diffuse abdominal pain. The patient is 34 weeks pregnant with her first child. The patient refuses to allow you to examine her, and her husband informs you that their culture does not allow males to examine or care for pregnant women. You should:

  • respect the patient’s wishes, ensure that the appropriate documentation is completed, and transport the patient.
  • insist that the patient requires proper care and that requires an adequate physical assessment and that you cannot be responsible for the outcome.
  • inform the patient that by calling for an ambulance, she is agreeing to the care provided and continue with your assessment and management.
  • call for the police to ensure that patient assessment is carried out.
    respect the patient’s wishes, ensure that the appropriate documentation is completed, and transport the patient.

The presence of meconium in the amniotic fluid indicates:

  • that full newborn resuscitation will be needed.
  • that the baby’s airway may be obstructed.
  • that the fetus is at least 4 weeks premature.
  • an expected finding in full-term infants.
    that the baby’s airway may be obstructed.

A nuchal cord is defined as an umbilical cord that:

  • has separated from the placenta.
  • is lacerated due to a traumatic delivery.
  • is wrapped around the baby’s neck.
  • has abnormally developed blood vessels.
    is wrapped around the baby’s neck.

The amniotic fluid serves to:

  • insulate and protect the fetus.
  • remove viruses from the fetus.
  • assist in fetal development.
  • transfer oxygen to the fetus.
    insulate and protect the fetus.

Common interventions used to stimulate spontaneous respirations in the newborn include all of the following, EXCEPT:

  • suctioning of the upper airway.
  • positive-pressure ventilations.
  • some form of tactile stimulation.
  • thorough drying with a towel.
    positive-pressure ventilations.

A history of pelvic inflammatory disease or tubal ligations increases a woman’s risk for:

  • gestational diabetes.
  • preeclampsia.
  • an ectopic pregnancy.
  • placenta previa.
    an ectopic pregnancy.

From what internal female organ is the fetus expelled during delivery?

  • Cervix
  • Vagina
  • Uterus
  • Perineum
    Uterus

Which of the following questions is of LEAST pertinence when determining whether a mother will deliver her baby within the next few minutes?

  • “Do you feel the need to push?”
  • “When are you due?”
  • “Have you had a sonogram?”
  • “Is this your first baby?”
    “Have you had a sonogram?”

When determining the frequency of contractions, you should time the contractions from the:

  • start of one to the end of the next.
  • end of one to the end of the next.
  • start of one to the start of the next.
  • end of one to the start of the next.
    start of one to the start of the next.

If a baby is born at 7:52, the second Apgar score should be calculated at:

  • 7:53.
  • 8:00.
  • 7:59.
  • 7:57.
    7:57.

During your visual inspection of a 25-year-old woman in labor, you see the baby’s head crowning at the vaginal opening. What should you do?

  • Place your fingers in the vagina to assess for a nuchal cord.
  • Maintain firm pressure to the head until it completely delivers.
  • Apply gentle pressure to the baby’s head as it delivers.
  • Tell the mother not to push and transport her immediately.
    Apply gentle pressure to the baby’s head as it delivers.

Three days after delivering her baby, a 30-year-old woman complains of a sudden onset of difficulty breathing. Her level of consciousness is decreased and she is tachycardic. The EMT should suspect:

  • acute pulmonary edema.
  • spontaneous pneumothorax.
  • pulmonary embolism.
  • intrauterine bleeding.
    pulmonary embolism.

Your 22-year-old patient is in active labor. Upon visual inspection, you note that the infant’s leg is protruding from the vagina. Appropriate management of this situation includes:

  • carefully attempting to push the infant’s leg off of the umbilical cord.
  • placing the mother in a recumbent position and rapidly transporting.
  • placing the mother supine with her head down and pelvis elevated.
  • gently pulling on the infant’s leg in an attempt to facilitate delivery.
    placing the mother supine with her head down and pelvis elevated.

With which age group should you talk to the child, not just the parent, while taking the medical history?

  • Preschooler
  • Toddler
  • Adolescent
  • School-age
    School-age

Which of the following is true about a child’s breathing?

  • A child’s oxygen demand is about the same as that of an adult.
  • The trachea is proportionally larger in an infant, leading to a greater potential for choking.
  • Older children experience intercostal muscle fatigue more quickly than younger children.
  • Because less air is exchanged with each breath, detection of poor air movement or complete absence of breath sounds can be difficult.
    Because less air is exchanged with each breath, detection of poor air movement or complete absence of breath sounds can be difficult.

What does the PAT assess?

  • Appearance, work of breathing, circulation
  • Tone, interactiveness, consolability, look, speech/cry
  • Alert, responsive, unresponsive
  • Airway, breathing, circulation
    Appearance, work of breathing, circulation

What should you use the Wong-Baker FACES scale to determine?

  • Motor response
  • LOC
  • Work of breathing
  • Pain level
    Pain level

What does respiratory syncytial virus (RSV) cause?

  • Bronchiolitis
  • Pneumonia
  • Pertussis
  • Epiglottitis
    Bronchiolitis

You are assessing a 6-year-old boy who has a fever of 102 and is showing signs of altered LOC. He is complaining of pain when he tries to turn his head. What condition should you suspect?

  • Meningitis
  • Spinal cord injury
  • Anaphylactic
  • Febrile seizure
    Meningitis

In a normal adult, greater than 30% to 40% of blood volume loss significantly increases the risk of shock. What is the threshold of blood volume loss in children?

  • 30%
  • 25%
  • 50%
  • 45%
    25%

You are using JumpSTART in a triage situation involving several children. How would you rate a patient who is breathing spontaneously, has a peripheral pulse, and is appropriately responsive to painful stimuli?

  • Red tag
  • Green tag
  • Yellow tag
  • Black tag
    Yellow tag

You are responding to a 9-1-1 call where the mother reported finding her 3-month-old daughter cyanotic and unresponsive in her crib. When you stimulate the child, she responds. This is an example of what condition?

  • Shaken baby syndrome
  • Apparent life-threatening event
  • SIDS
  • Febrile seizure
    Apparent life-threatening event

If a pediatric patient begins seizing again during your care, which of the following would be your treatment priority?

  • Providing rapid transport
  • Stopping the seizure
  • Securing and clearing the airway
  • Maintaining the patient’s body temperature
    Securing and clearing the airway

You are dispatched to a residence for a child with respiratory distress. The patient, an 18-month-old female, is tachypneic, has sternal retractions, and is clinging to her mother. Her skin is pink and dry, and her heart rate is 120 beats/min. The MOST appropriate treatment for this child includes:

  • administering blow-by oxygen and transporting the child with her mother.
  • requesting a paramedic ambulance to insert an advanced airway device.
  • separating the child from her mother and providing ventilatory assistance.
  • allowing the child to remain with her mother and applying a nasal cannula.
    administering blow-by oxygen and transporting the child with her mother.

Signs and symptoms of meningitis in the infant or child include all of the following, EXCEPT:

  • altered mental status.
  • sunken fontanelles.
  • headache and fever.
  • a stiff or painful neck.
    sunken fontanelles.

A 2-year-old female has experienced a seizure. When you arrive at the scene, the child is conscious, crying, and clinging to her mother. Her skin is hot and moist. The mother tells you that the seizure lasted approximately 5 minutes. She further tells you that her daughter has no history of seizures, but has had a recent ear infection. You should:

  • suspect that the child has meningitis and transport at once.
  • allow the mother to drive her daughter to the hospital.
  • place the child in cold water to attempt to reduce her fever.
  • cool the child with tepid water and transport to the hospital.
    cool the child with tepid water and transport to the hospital.

When a child is struck by a car, the area of greatest injury depends MOSTLY on the:

  • speed at which the car was traveling when impact occurred.
  • age of the child and the size of the car that struck him or her.
  • size of the child and the height of the bumper upon impact.
  • height of the child and the speed at which the car was traveling.
    size of the child and the height of the bumper upon impact.

When questioning the parent of a child who ingested a poisonous substance, which of the following questions would be of LEAST pertinence initially?

  • Why did your child ingest the poison?
  • What time did the ingestion occur?
  • Do you know what substance was ingested?
  • Have you noticed any signs or symptoms?
    Why did your child ingest the poison?

Burns in children are commonly caused by all of the following, EXCEPT:

  • exposure to caustic chemicals.
  • scalding water in a bathtub.
  • entrapment in a structural fire.
  • hot items on a stovetop.
    entrapment in a structural fire.

Pale skin in a child indicates that the:

  • oxygen content in the blood is decreased.
  • blood vessels near the skin are constricted.
  • child is in severe decompensated shock.
  • child’s core body temperature is elevated.
    blood vessels near the skin are constricted.

Common causes of seizures in children include all of the following, EXCEPT:

  • poisonings or ingestion.
  • electrolyte imbalances.
  • hyperglycemia.
  • infection.
    hyperglycemia.

When assessing an 8-year-old child, you should:

  • talk to the child, not just the caregiver.
  • rely solely on the parent for information.
  • refrain from taking a blood pressure.
  • use a toe-to-head assessment approach.
    talk to the child, not just the caregiver.

While triaging patients at the scene of a motor-vehicle crash, you encounter a 5-year-old child who is unresponsive and apneic. After positioning his airway, you should:

  • deliver 5 rescue breaths.
  • palpate for a carotid pulse.
  • categorize him as deceased.
  • categorize him as immediate.
    palpate for a carotid pulse.

An infant with severe dehydration would be expected to present with:

  • excessive tearing.
  • moist oral mucosa.
  • absent urine output.
  • bulging fontanelles.
    absent urine output.

A viral infection that may cause obstruction of the upper airway in a child is called:

  • croup.
  • asthma.
  • bronchitis.
  • epiglottitis.
    croup.

Infection should be considered a possible cause of an airway obstruction in an infant or child, especially if he or she presents with:

  • drooling or congestion.
  • acute respiratory distress.
  • skin that is cool and dry.
  • extreme restlessness.
    drooling or congestion.

When caring for a female child who has possibly been sexually abused, you should:

  • immediately report your suspicions to the parents.
  • have a female EMT remain with her if possible.
  • encourage the child to urinate and take a shower.
  • carefully examine the genitalia for signs of injury.
    have a female EMT remain with her if possible.

The purpose of the pediatric assessment triangle (PAT) is to:

  • determine if the child’s vital signs are within the age-appropriate limits.
  • gather critical data by performing a rapid hands-on assessment of the child.
  • facilitate a rapid head-to-toe assessment of the child by visualization only.
  • allow you to rapidly and visually form a general impression of the child.
    allow you to rapidly and visually form a general impression of the child.

The first month of life after birth is referred to as the:

  • neonatal period.
  • start of infancy.
  • premature phase.
  • toddler period.
    neonatal period.

A 5-year-old child has had severe vomiting and diarrhea for 4 days. Which of the following assessment findings would be the MOST indicative of decompensated shock?

  • Respiratory rate of 30 breaths/min
  • Blood pressure of 70/40 mm Hg
  • Capillary refill time of 4 seconds
  • Pulse rate greater than 120/min
    Blood pressure of 70/40 mm Hg

Which of the following statements regarding sudden infant death syndrome (SIDS) is correct?

  • SIDS is most commonly the result of an overwhelming infection.
  • The cause of death following SIDS can be established by autopsy.
  • Certain cases of SIDS are predictable and therefore preventable.
  • Death as a result of SIDS can occur at any time of the day or night.
    Death as a result of SIDS can occur at any time of the day or night.

Which of the following findings is LEAST suggestive of child abuse?

  • Consistency in the method of injury reported by the caregiver
  • Evidence of alcohol consumption or drug use at the scene
  • An unexplained delay in seeking medical care after the injury
  • Burns to the hands or feet that involve a glove distribution
    Consistency in the method of injury reported by the caregiver

Unless he or she is critically ill or injured, you should generally begin your assessment of a toddler:

  • at the feet.
  • at the head.
  • en route to the hospital.
  • in the ambulance.
    at the feet.

Which interview technique should you use when addressing a geriatric patient?

  • Have both you and your partner ask question.
  • Speak loudly and slowly.
  • Refer to the patient by his first name.
  • Listen carefully to the answers the patient provides.
    Listen carefully to the answers the patient provides.

You are assessing an 80-year-old patient who explains that he was awakened by a sudden feeling of suffocation and respiratory distress. What are his symptoms a characteristic of?

  • Orthopnea
  • Paroxysmal nocturnal dyspnea
  • Exertional dyspnea
  • Intermittent sleep apnea
    Paroxysmal nocturnal dyspnea

Which of the following is a nonpreventable factor in stroke?

  • Age
  • Hypertension
  • Obesity
  • Atrial fibrillation
    Age

In arteriosclerosis, overproduction of abnormal collagen and decreased quantities of elastin lead to what?

  • Decreased coronary artery perfusion
  • Increased coronary artery perfusion
  • Narrowing pulse pressure
  • Hypotension
    Decreased coronary artery perfusion

Which of the following is true regarding delirium?

  • It has a slow onset of progressive disorientation.
  • It is a result of an irreversible metabolic disorder.
  • It is rare in older hospitalized patients.
  • Memory remains intact.
    Memory remains intact.

You are working on a geriatric patient who reports severe abdominal pain radiating through her back and flank. She also reports discomfort in her right leg and your assessment reveals a diminished pulse in the extremity. What should you suspect?

  • Peptic ulcer disease
  • Venous stasis
  • Pulmonary embolism
  • Abdominal aortic aneurysm
    Abdominal aortic aneurysm

Which of the following groups are most likely to commit suicide?

  • Teenage girls (any ethnicity)
  • White males older than 85 years
  • African American women older than 65 years
  • College-educated males aged 45-65 years
    White males older than 85 years

Which of the following would be part of the “S” component of the GEMS diamond?

  • Hazardous living conditions
  • Polypharmacy
  • Activities of daily living
  • Normal changes of aging
    Activities of daily living

On auscultation of a patient’s lungs, you hear crackles. These lung sounds are caused by air passing through what?

  • Thick secretions in the airways
  • Fluid in the alveoli
  • Inflamed airways
  • Constricted airways
    Fluid in the alveoli

Which of the following questions should you ask if you suspect a patient may have congestive heart failure?

  • “In what position do you normally sleep?”
  • “How many hours of sleep do you get each night?”
  • “How many pillows do you sleep on?”
  • “Do you take any medication to help you sleep at night?”
    “How many pillows do you sleep on?”

During the natural process of aging, the number of functional cilia in the respiratory system decreases, resulting in:

  • air-trapping within the alveoli.
  • a decreased ability to cough.
  • an increased risk of COPD.
  • baseline respiratory distress.
    a decreased ability to cough.

An 84-year-old male fell a week ago and has been bedridden since then. Today, he presents with an altered mental status. His skin is pale and cold and his respirations are rapid and shallow. The EMT should suspect:

  • acute hyperglycemia.
  • a subdural hematoma.
  • a systemic infection.
  • hypovolemic shock.
    a systemic infection.

Because of the complexity of the older patient and the vagueness of his or her complaint, you should:

  • rely exclusively on family members for the medical history.
  • perform a rapid assessment on all geriatric patients you treat.
  • limit your physical examination to the area of pain or injury.
  • attempt to differentiate between chronic and acute problems.
    attempt to differentiate between chronic and acute problems.

Poor maintenance of home, poor personal care, and dietary neglect are all possible indicators of __ elder abuse.

  • psychological
  • emotional
  • physical
  • financial
    physical

The stooped posture of some older people, which gives them a humpback appearance, is called:

  • miosis.
  • kyphosis.
  • scoliosis.
  • arthritis.
    kyphosis.

Common causes of depression in the elderly include all of the following, EXCEPT:

  • alcohol abuse and dependence.
  • chronic medical conditions.
  • an acute onset of dementia.
  • prescription medication use.
    an acute onset of dementia.

Clouding of the lenses of the eyes is called:

  • conjunctivitis.
  • cataracts.
  • retinitis.
  • glaucoma.
    cataracts.

Talking about an elderly patient in front of him or her to other members of the family:

  • is usually beneficial because the patient’s cognitive skills are typically impaired.
  • may cause the patient to think that he or she has no say in making decisions.
  • will anger the patient and result in his or her refusal to accept care or transport.
  • often causes the patient to become paranoid and untrusting of your help.
    may cause the patient to think that he or she has no say in making decisions.

An abdominal aortic aneurysm:

  • can sometimes be palpated as a mass in the groin area.
  • is often the result of hypertension and atherosclerosis.
  • is usually not repairable, even if discovered early.
  • causes dull pain that often radiates to the shoulders.
    is often the result of hypertension and atherosclerosis.

When immobilizing a patient with a kyphotic spine to a long backboard, the EMT would MOST likely have to:

  • use a scoop stretcher instead of a log roll.
  • place blankets behind the patient’s head.
  • secure the patient’s head before the torso.
  • force the head into a neutral alignment.
    place blankets behind the patient’s head.

Blood levels of medications may rise in the elderly, sometimes to toxic levels. This is MOST likely due to:

  • pancreatic failure.
  • intentional overdose.
  • renal insufficiency.
  • splenic dysfunction.
    renal insufficiency.

Sedentary behavior while healing from a hip fracture would MOST likely predispose the older patient to:

  • pneumonia.
  • heart failure.
  • osteoporosis.
  • ischemic stroke.
    pneumonia.

To minimize distractions and confusion when assessing an older patient, you should:

  • perform a physical exam and then talk to the patient.
  • elevate your voice and speak directly to the patient.
  • have only one EMT speak to the patient at a time.
  • dismiss the family members from the room or area.
    have only one EMT speak to the patient at a time.

You are assessing a 70-year-old female who complains of intense thirst, frequent urination, and dizziness. She has a history of type 2 diabetes, heart disease, rheumatoid arthritis, and gout. Her blood glucose reads “high.” She is conscious, but confused. Her blood pressure is 92/52 mm Hg, her pulse rate is 130 beats/min and weak, and her respirations are 22 breaths/min and shallow. This patient’s clinical presentation is MOST consistent with:

  • hyperosmolar hyperglycemic nonketotic syndrome.
  • acute renal failure with associated hyperglycemia.
  • hyperglycemia with moderate dehydration.
  • diabetic ketoacidosis.
    hyperosmolar hyperglycemic nonketotic syndrome.

The leading cause of death in the geriatric patient is:

  • hypertension.
  • altered mental status.
  • arthritis.
  • heart disease.
    heart disease.

A 77-year-old female presents with an acute onset of altered mental status. Her son is present and advises that she has a history of hypertension, atrial fibrillation, type 2 diabetes, and glaucoma. He further advises that she takes numerous medications and that she is normally alert. When you assess this patient, it is important to note that:

  • because of her age and medical history, you should suspect Alzheimer disease.
  • dementia typically presents as an acute onset of deterioration of cognitive function.
  • her mental status is likely the result of hypoglycemia and you should give her sugar.
  • the patient is experiencing delirious behavior, which suggests a new health problem.
    the patient is experiencing delirious behavior, which suggests a new health problem.

Which of the following observations or statements represents the “E” in the GEMS diamond?

  • The patient’s residence is cold due to a malfunctioning heater.
  • A patient is assisted with his or her activities of daily living.
  • The patient’s medications have not been filled in 2 months.
  • Elderly patients present atypically and deserve your respect.
    The patient’s residence is cold due to a malfunctioning heater.

In contrast to younger patients, older patients are more prone to a decrease in blood pressure (BP) upon standing because:

  • any change in position causes blood to be shunted to the brain.
  • their red blood cells are destroyed at a faster than normal rate.
  • the body is less able to adapt the BP to rapid postural changes.
  • the aging process results in an overall increase in blood volume.
    the body is less able to adapt the BP to rapid postural changes.

A specific legal document that directs relatives and caregivers regarding the medical treatment that may be given to patients who cannot speak for themselves is called a(n):

  • physician directive.
  • statute of care.
  • advance directive.
  • power of attorney.
    advance directive.

The EMT should suspect left-sided heart failure in the geriatric patient who presents with:

  • fever and a cough that produces green sputum.
  • jugular venous distention and peripheral edema.
  • tachypnea and paroxysmal nocturnal dyspnea.
  • swelling of the lower extremities and weakness.
    tachypnea and paroxysmal nocturnal dyspnea.

Which of the following techniques should you use when you are communicating with a patient who has autism?

  • Speak normally and provide simple, one-step directions.
  • Exaggerate your lip movements.
  • Speak loudly into the patient’s ear.
  • You and your partner should talk to the patient together.
    Speak normally and provide simple, one-step directions.

What does the TEAM approach stand for?

  • Treat, Evaluate, Administer, Monitor
  • Test Extension and Abduction Motion
  • Trust Every Available Member
  • Take Evasive Action Monitoring
    Trust Every Available Member

Nerve damage to the inner ear results in what?

  • Otitis media
  • Sensorineural deafness
  • Conductive hearing loss
  • Tinnitus
    Sensorineural deafness

You are at the home of an elderly man who has a tracheostomy tube and is on mechanical ventilation. His ventilator has malfunctioned. Once you disconnect the ventilator, what should you do?

  • Deliver 100% oxygen through a nonrebreathing mask.
  • Transport the patient to a critical care facility immediately.
  • Use a nasal cannula to deliver oxygen.
  • Deliver oxygen using a face mask over the stoma.
    Deliver oxygen using a face mask over the stoma.

A patient with epilepsy is likely to have which of the following medical devices?

  • LVAD
  • Central venous catheter
  • Shunt
  • Vagus nerve stimulator
    Vagus nerve stimulator

Which of the following is the best source of information when you are dealing with an autistic patient?

  • Medical control
  • The attending doctor
  • The patient him/herself
  • Family members/caregivers
    Family members/caregivers

Which of the following mnemonics is helpful when you are dealing with a tracheostomy tube obstruction?

  • DUMBELS
  • PEARRL
  • DOPE
  • PASTE
    DOPE

What does the Emergency Medical Treatment and Active Labor Act (EMTALA) dictate?

  • All patients are allowed the right to treatment and/or transport.
  • All health care facilities must provide a medical assessment and treatment, regardless of the patient’s ability to pay.
  • All health care facilities must provide a medical assessment and treatment, regardless of the patient’s mental or physical limitations.
  • All EMTs must have a basic level of training in order to administer aid to a patient in the field.
    All health care facilities must provide a medical assessment and treatment, regardless of the patient’s ability to pay.

When treating a patient with spina bifida, which of the following is most likely to produce anaphylaxis?

  • Aspirin
  • Latex
  • Sulfa drugs
  • Iodine
    Latex

What is the best position in which to transport an obese patient?

  • Prone
  • With the feet elevated
  • Supine
  • With the head elevated
    With the head elevated

Common complications associated with central venous catheters include all of the following, EXCEPT:

  • a local infection.
  • bleeding around the line.
  • clotting of the line.
  • rupture of a central vein.
    rupture of a central vein.

A person is said to be obese when he or she is __ over his or her ideal weight.

  • 10% or more
  • 30% or more
  • 20% or more
  • 40% or more
    30% or more

Spina bifida is defined as:

  • congenital inflammation of the spinal cord, usually in the neck.
  • a birth defect caused by incomplete closure of the spinal column.
  • chronic pressure on the brain caused by excess cerebrospinal fluid.
  • a birth defect in which the child is born without spinal vertebrae.
    a birth defect caused by incomplete closure of the spinal column.

When caring for a patient who is visually impaired, it is important to:

  • tell him or her what is happening, identify noises, and describe the situation and surroundings.
  • stand to the side of the patient when speaking if his or her peripheral vision is impaired.
  • leave items such as canes and walkers at the residence if the patient will be carried on a gurney.
  • allow a service dog to remain with the patient at all times, even if the patient is critically ill.
    tell him or her what is happening, identify noises, and describe the situation and surroundings.

A surgical procedure that creates an opening between the intestine and the surface of the body that allows for elimination of waste products is called a(n):

  • colostomy.
  • gastrostomy.
  • intestinal shunt.
  • gastric stoma.
    colostomy.

A 13-year-old child is on a home ventilator. The parents called because the mechanical ventilator is malfunctioning and the child has increasing respiratory distress. You should:

  • reset the ventilator by unplugging it for 30 to 60 seconds.
  • place a call to the home health agency treating this patient.
  • disconnect the ventilator and apply a tracheostomy collar.
  • attempt to troubleshoot the mechanical ventilator problem.
    disconnect the ventilator and apply a tracheostomy collar.

According to the Emergency Medical Treatment and Active Labor Act (EMTALA):

  • all health care facilities must provide a medical assessment and required treatment, regardless of the patient’s ability to pay.
  • all health care facilities are legally obligated to provide assessment and care only if the patient is critically ill or injured.
  • a patient maintains the legal right to recant his or her consent to emergency treatment, even after signing in to the emergency department.
  • a health care facility has the right to refuse assessment and treatment to a patient, but only if his or her condition is not deemed critical.
    all health care facilities must provide a medical assessment and required treatment, regardless of the patient’s ability to pay.

According to the “E” in the DOPE mnemonic, which of the following actions should you perform to troubleshoot inadequate ventilation in a patient with a tracheostomy tube?

  • Check the mechanical ventilator for malfunction.
  • Attempt to pass a suction catheter into the tube.
  • Look for blood or other secretions in the tube.
  • Listen to breath sounds to assess for a pneumothorax.
    Check the mechanical ventilator for malfunction.

Which of the following statements regarding interaction with the caregiver of a child or adult with special health care needs is correct?

  • In most cases, it is more appropriate for the EMT to contact medical control prior to speaking with the patient’s primary caregiver.
  • Before performing an assessment of the patient’s ABCs, the EMT should ask the caregiver about the patient’s medical condition.
  • In general, the EMT should only speak with a certified home health care provider because he or she is the expert on the patient’s illness.
  • Communication with the patient’s caregiver or family members is important because they are the most familiar with the patient’s condition.
    Communication with the patient’s caregiver or family members is important because they are the most familiar with the patient’s condition.

A 70-year-old male complains of shortness of breath. During your assessment, you note that he has bilateral hearing aids. When you ask him questions related to his chief complaint, he does not answer you. You can hear a whistling sound coming from his hearing aids. You should:

  • remove both of his hearing aids and use pencil and paper to communicate.
  • try repositioning the hearing aid or remove it and turn down the volume.
  • remove his hearing aids, turn up the volume, and replace them in his ears.
  • recognize that the batteries in his hearing aids are probably depleted.
    try repositioning the hearing aid or remove it and turn down the volume

Which of the following statements regarding patients with intellectual disabilities is correct?

  • An intellectual disability differs from mental retardation in that it is the result of a congenital abnormality.
  • Most patients with intellectual disabilities have normal cognitive function, but abnormal physical features.
  • Speaking with the patient’s family is the least effective way to determine how much the patient understands.
  • Patients with intellectual disabilities are susceptible to the same disease processes as other patients.
    Patients with intellectual disabilities are susceptible to the same disease processes as other patients.

The purpose of a ventricular peritoneum shunt is to:

  • monitor pressure within the skull in patients with a head injury.
  • prevent excess cerebrospinal fluid from accumulating in the brain.
  • divert excess cerebrospinal fluid to the ventricles of the brain.
  • remove fluid from the abdomen of patients with right-sided heart failure.
    prevent excess cerebrospinal fluid from accumulating in the brain.

You are assessing a 440-lb man who complains of shortness of breath and lower back pain. The patient is conscious and alert, his blood pressure is 148/98 mm Hg, and his heart rate is 120 beats/min. Your MOST immediate action should be to:

  • perform a secondary assessment, focusing on his respiratory system and back.
  • ask a member of your team to locate the best route to move him to the ambulance.
  • avoid placing him in a supine position if possible and administer oxygen.
  • notify the receiving facility and advise them of the patient’s weight and status.
    avoid placing him in a supine position if possible and administer oxygen.

Down syndrome is a genetic defect that occurs as the result of:

  • a separation of chromosome 21.
  • a sperm that contains 24 chromosomes.
  • a triplication of chromosome 21.
  • an extra pair of chromosomes.
    a triplication of chromosome 21.

Which of the following would be the MOST practical method of communicating with a hearing-impaired patient until his or her hearing aids can be located?

  • Using a piece of paper and writing utensil to ask questions
  • Attempting to use body language to determine the problem
  • Contacting dispatch and requesting a sign language interpreter
  • Using a high-pitched voice while speaking directly into the ear
    Using a piece of paper and writing utensil to ask questions

Cerebral palsy is characterized by poorly controlled __ movement.

  • extremity
  • body
  • eye
  • neck
    body

Because a tracheostomy tube bypasses the nose and mouth:

  • secretions can build up in and around the tube.
  • the risk of a local infection is significantly high.
  • severe swelling of the trachea and bronchi can occur.
  • bleeding or air leakage may occur around the tube.
    secretions can build up in and around the tube.

Which of the following statements regarding autism is correct?

  • Impairment of motor activity is a classic sign of autism.
  • The majority of patients with autism do not speak at all.
  • Autism affects females four times greater than males.
  • Most cases of autism are diagnosed by 3 years of age.
    Most cases of autism are diagnosed by 3 years of age.

Which of the following conditions would MOST likely be encountered in a patient with cerebral palsy?

  • Seizure disorder
  • Paralysis
  • Brain tumors
  • Type 2 diabetes
    Seizure disorder

Under what circumstances is a left ventricular assist device used?

  • To permanently replace the function of one or both of the ventricles
  • To ensure that the ventricles contract at an adequate and consistent rate
  • To reduce ventricular pumping force in patients with aortic aneurysms
  • As a bridge to heart transplantation while a donor heart is being located
    As a bridge to heart transplantation while a donor heart is being located

The six-pointed Star of Life emblem identifies vehicles that meet which of the following criteria?

  • Meet federal specifications as licensed or certified ambulances
  • Are staffed by a minimum of one certified advanced EMT (AEMT)
  • Are equipped with supplies to manage mass-casualty situation
  • Are operated federally certified ambulance operators
    Meet federal specifications as licensed or certified ambulances

If you are on an emergency call with your warning lights and siren turned on, you may be allowed to do which of the following?

  • Pass a stopped school bus with the stop sign out.
  • Drive faster than the posted speed limit.
  • Drive against the flow of traffic on an interstate in the left lane.
  • Proceed through a red traffic light or stop sign without stopping.
    Drive faster than the posted speed limit.

Keeping a safe distance between your vehicle and the one in front of you, checking for tailgaters behind your ambulance, and being aware of objects in your mirror’s blind spots are considered maintaining what?

  • A buffer zone
  • An evasion zone
  • A cushion of safety
  • Open space
    A cushion of safety

If you arrive on the scene of a mass-casualty incident, what is the first thing you should do?

  • Ask for additional resources.
  • Begin treating patients.
  • Direct traffic until law enforcement arrives.
  • Declare the area a crime scene.
    Ask for additional resources.

To ensure you have enough reaction time to avoid hitting a motorist who does not move over, you should drive, at minimum, how many seconds behind vehicles traveling at an average speed?

  • 8
  • 2
  • 4
  • 6
    4

When responding to a scene where smoke or possible hazardous materials are present, how should you park?

  • Uphill and downwind
  • Downhill and upwind
  • Uphill and upwind
  • Downhill and downwind
    Uphill and upwind

What are the recommended dimensions for a landing zone for a medivac helicopter?

  • 100 x 100 feet
  • 150 x 150 feet
  • 60 x 60 feet
  • 80 x 80 feet
    100 x 100 feet

If you are signaled by the medivac crew to approach a medivac helicopter that has landed on uneven ground, from which direction should you approach the aircraft?

  • From the front
  • From the downhill side of the aircraft
  • From the rear
  • From the uphill side of the aircraft
    From the downhill side of the aircraft

When driving an ambulance, what is the safest path when cornering?

  • Enter low, apex high, and exit high
  • Enter high and exit low
  • Enter high, apex low, and exit high
  • Enter low and exit low
    Enter high and exit low

A jump kit contains everything you might need in the first 5 minutes with a patient, including which of the following?

  • An AED
  • Long backboard
  • A CPR board
  • A BVM
    A BVM

The process of removing dirt, dust, blood, or other visible contaminants from a surface or equipment is called:

  • cleaning.
  • sterilization.
  • high-level disinfection.
  • disinfection.
    cleaning.

A type _ ambulance features a conventional, truck cab-chassis with a modular ambulance body that can be transferred to a newer chassis as needed.

  • III
  • IV
  • II
  • I
    I

General guidelines for safe ambulance driving include all of the following, EXCEPT:

  • avoiding one-way streets whenever possible.
  • avoiding routes with heavy traffic congestion.
  • assuming that other drivers will not see you.
  • regularly using the siren as much as possible.
    regularly using the siren as much as possible.

While en route to a call for a patient in cardiac arrest, you approach a stopped school bus with its red warning lights flashing. You should:

  • stop and wait until the warning lights stop flashing.
  • pass the bus only after all the children have exited.
  • back up and take an alternate route to the scene.
  • slowly and carefully pass the bus on the left side.
    stop and wait until the warning lights stop flashing.

In which of the following situations would the EMTs MOST likely utilize a police escort?

  • The call is dispatched as an unresponsive patient with CPR in progress.
  • The EMTs are transporting a critical pediatric patient through traffic.
  • The weather is treacherous and there are numerous roads washed out.
  • The EMTs are unfamiliar with the location, but the police officer knows the area.
    The EMTs are unfamiliar with the location, but the police officer knows the area.

A medical transport helicopter is incoming and you are responsible for setting up the landing zone (LZ). You should:

  • ask bystanders to stand at all four corners of the LZ.
  • use weighted cones to mark all four corners of the LZ.
  • use yellow caution tape to mark of the LZ perimeter.
  • place four flares 100 feet apart in an “X” pattern.
    use weighted cones to mark all four corners of the LZ.

The LEAST practical place to store a portable oxygen cylinder is:

  • in the driver’s compartment.
  • near the side or rear door.
  • on the ambulance stretcher.
  • inside the jump kit.
    in the driver’s compartment.

You have just delivered a major trauma patient to the hospital. Shortly after departing the hospital, dispatch advises you of another call. The back of the ambulance is contaminated with bloody dressings and is in disarray, and you are in need of airway equipment and numerous other supplies. You should:

  • advise the dispatcher that you are out of service and to send another unit.
  • have your partner quickly clean the ambulance as you proceed to the call.
  • quickly proceed to the call and clean and restock the ambulance afterwards.
  • proceed to the call, functioning only as an emergency medical responder.
    advise the dispatcher that you are out of service and to send another unit.

Hydroplaning of the ambulance on wet roads would MOST likely occur at speeds of greater than _ mph.

  • 30
  • 15
  • 10
  • 20
    30

When arriving at the scene of an overturned tractor-trailer rig, you note that a green cloud is being emitted from the crashed vehicle. The driver is still in the truck; he is conscious but bleeding profusely from the head. After notifying the hazardous materials team, you should:

  • quickly gain access to the patient.
  • park downhill from the scene.
  • position the ambulance upwind.
  • ask the driver to exit the vehicle.
    position the ambulance upwind.

For every emergency request, the dispatcher should routinely gather and record all of the following information, EXCEPT the:

  • caller’s phone number.
  • nature of the call.
  • location of the patient(s).
  • patient’s medical history.
    patient’s medical history.

The use of lights and siren on an ambulance:

  • allows other drivers to hear and see you from a great distance.
  • legally gives the emergency vehicle operator the right of way.
  • is required any time a patient is being transported to the hospital.
  • signifies a request for other drivers to yield the right of way.
    signifies a request for other drivers to yield the right of way.

Common safety equipment carried on the ambulance includes all of the following, EXCEPT:

  • turnout gear.
  • hazardous materials gear.
  • safety goggles.
  • face shields.
    hazardous materials gear.

Immediately upon arriving at the scene of an emergency call involving a traumatic injury, you should notify the dispatcher of your arrival and then:

  • observe the scene for safety hazards.
  • carefully assess the mechanism of injury.
  • determine if additional units are needed.
  • quickly gain access to the patient.
    observe the scene for safety hazards.

Which of the following is an example of regional equipment or supplies?

  • DuoDote Auto-Injector
  • Oral glucose
  • Vacuum splint
  • Inhaled bronchodilator
    DuoDote Auto-Injector

The MOST common and usually the most serious ambulance crashes occur at:

  • intersections.
  • railroad crossings.
  • stop signs.
  • stop lights.
  • intersections.

Maintaining a cushion of safety when operating an ambulance means:

  • driving at the posted speed limit, regardless of the patient’s condition, and routinely using your lights and siren when driving on a freeway.
  • remaining in the far right-hand lane when transporting a critical patient and refraining from passing other motorists on the left side.
  • keeping a safe distance between your ambulance and the vehicles in front of you and remaining aware of vehicles potentially hiding in your mirrors’ blind spots.
  • driving about 2 to 3 seconds behind any vehicles in front of you and exceeding the posted speed limit by no more than 20 to 25 mph.
    keeping a safe distance between your ambulance and the vehicles in front of you and remaining aware of vehicles potentially hiding in your mirrors’ blind spots.

Equipment and supplies that are carried on an ambulance should be stored:

  • as directed by the EMS system’s medical director.
  • according to the urgency and frequency of their use.
  • based on recommendations of the health department.
  • in locked or secured cabinets in order to prevent theft.
    according to the urgency and frequency of their use.

When approaching a helicopter, whether the rotor blades are moving or not, you should:

  • carefully approach the aircraft from the rear unless a crew member instructs you to do otherwise.
  • remember that the main rotor blade is flexible and can dip as low as 5¢ to 6¢ from the ground.
  • never duck under the body or the tail boom because the pilot cannot see you in these areas.
  • approach the aircraft from the side because this will make it easier for you to access the aircraft doors.
    never duck under the body or the tail boom because the pilot cannot see you in these areas.

During the transport phase of an ambulance call, it is MOST important to:

  • converse with the patient and provide reassurance.
  • complete the run form before arrival at the hospital.
  • reassess the patient only if he or she deteriorates.
  • reassess unstable patients at least every 15 minutes.
    converse with the patient and provide reassurance.

Where should you park the ambulance when responding to a vehicle crash involving a hazardous material spill?

  • Downhill and downwind from the hazard
  • Uphill and downwind from the hazard
  • Uphill and upwind from the hazard
  • Downhill and upwind from the hazard
    Uphill and upwind from the hazard

As an EMT, what is your main role at the scene of an accident involving extrication of the patient?

  • To extricate the patients
  • To assess and provide immediate medical care
  • To provide safe entrance and access to the patients
  • To provide traffic control and direction
    To assess and provide immediate medical care

Which of the following is an example of simple access?

  • Removing the windshield
  • Opening a rear door
  • Breaking a side window
  • Prying open a door
    Opening a rear door

In an instance of multiple casualties, how should patient care be initiated?

  • Only after all patients have been triaged
  • In the order you reach each patient
  • Based on the mechanism of injury
  • In the order of age beginning with the youngest
    Only after all patients have been triaged

During which phase of extrication should you don proper protective gear (PPE)?

  • En route
  • Patient removal
  • Upon arrival
  • During hazard control
    Upon arrival

You are assessing a female patient as the extrication team gets ready to roll up the dashboard to remove her. The patient has multiple fractures in her left arm. What should you do?

  • Apply a rigid splint before she is removed.
  • Apply an air splint before she is removed.
  • Leave the arm alone until the patient has been extricated.
  • Secure the injured arm against her body.
    Secure the injured arm against her body.

You have tried to gain simple access to a crashed vehicle but the doors are all jammed. What is your next step?

  • Break the window farthest away from the patient.
  • Break the front windshield.
  • Break the window closest to the patient.
  • Use the “jaws of life” to jimmy the driver’s side door.
    Break the window farthest away from the patient.

At what point should you begin your primary assessment of a patient who has been entrapped?

  • After the patient has been extricated from the wreckage
  • Before extrication begins
  • Once the patient has been moved to the safe zone
  • En route to the ED
    Before extrication begins

What should you do if you find a patient entrapped in a vehicle who is in need of CPR?

  • Perform CPR during the extrication process.
  • Apply a bag-valve mask (BVM).
  • Perform rapid extrication techniques.
  • Apply manual stabilization until the patient can be extricated.
    Perform rapid extrication techniques.

How far away should you park your ambulance from the site of a cave-in or trench collapse?

  • 1000 feet
  • 20 feet
  • 500 feet
  • 100 feet
    500 feet

Once entrance and access to the patient have been provided, you should:

  • perform a primary assessment.
  • allow extrication to commence.
  • administer high-flow oxygen.
  • begin treating his or her injuries.
    perform a primary assessment.

When functioning at the scene of a motor vehicle crash in which a patient will require complex extrication, you should enter the vehicle and provide care to the patient:

  • after receiving approval from the incident commander.
  • only after you believe the vehicle has been stabilized.
  • only if the patient has experienced a major injury.
  • after rescue personnel have cut the battery cables.
    after receiving approval from the incident commander.

Upon arrival at a search-and-rescue incident, the EMS crew should _.

  • prepare the equipment to carry to the patient.
  • split up and request a search grid
  • provide the incident commander with the crew’s names and certification level(s)
  • begin searching for the patient
    prepare the equipment to carry to the patient.

If a technical rescue team is required at the scene, but is not present when you arrive, you should:

  • check with the incident commander to ensure that the team is en route.
  • remain with your ambulance until the rescue team arrives at the scene.
  • don personal protective equipment and begin the rescue process.
  • have fire personnel initiate the rescue process if they are at the scene.
    check with the incident commander to ensure that the team is en route.

Upon arriving at the scene of a motor vehicle crash, you can see three patients, one who is entrapped in his car and two who have been ejected from their vehicle. You should:

  • immediately request additional resources.
  • call medical control for further direction.
  • request law enforcement for traffic control.
  • begin triage to determine injury severity.
    immediately request additional resources.

The rescue team is in the process of extricating a 40-year-old male from his truck. The patient’s wife, who was uninjured in the crash, is calmly observing the extrication and asks you if her husband will be all right. You should:

  • ensure that she is in a safe area, away from the scene.
  • allow her to observe the extrication and keep her calm.
  • allow her to talk to her husband during the extrication.
  • ask her follow-up questions about the details of the crash.
    ensure that she is in a safe area, away from the scene.

At a scene with downed electrical lines, the EMT should _.

  • stabilize the patient in the danger (hot) zone
  • enter the danger (hot) zone for patient removal only
  • relocate the danger (hot) zone away from the patient
  • remain outside the danger (hot) zone
    remain outside the danger (hot) zone

The use of special tools to remove an entrapped patient from a vehicle is known as _.

  • vehicle stabilization
  • complex access
  • simple access
  • incident management
    complex access

You are attempting to gain access to a patient who was injured when his truck struck another vehicle from behind. The patient is conscious and alert, but is screaming in pain. You try to open the door, but it is locked. You should:

  • break the window and unlock the door.
  • use a pry bar to attempt to open the door.
  • ask the patient if he can unlock the door.
  • request the rescue team to extricate him.
    ask the patient if he can unlock the door.

A 19-year-old female has just been extricated from her severely damaged car. She is on a long backboard and has been moved to a place of safety. As your partner maintains manual stabilization of her head, you perform a rapid assessment. The patient is unresponsive, has slow and shallow respirations, and has bilaterally closed femur deformities. You should:

  • obtain baseline vital signs and transport at once.
  • stabilize her legs with long board splints.
  • direct your partner to begin ventilatory assistance.
  • apply oxygen via a nonrebreathing mask.
    direct your partner to begin ventilatory assistance.

You arrive at an accident scene to find an alternative-fuel vehicle leaking an unknown substance. You should immediately _.

  • disconnect the car battery
  • search the vehicle for occupants
  • apply retardant to the leaking fuel
  • call for additional resources and keep bystanders away from the vehicle
    call for additional resources and keep bystanders away from the vehicle

Trench collapses usually involve large areas of falling dirt that weigh approximately _ per cubic foot.

  • 200 lb
  • 100 lb
  • 150 lb
  • 50 lb
    100 lb

Upon arrival at a trench rescue operation, you should _.

  • park as close to the incident as possible
  • assist with the rescue operation
  • determine if rescuers are operating safely
  • park at least 500 feet from the incident
    park at least 500 feet from the incident

While staged at the scene of a structure fire, the EMT should _.

  • stay with the incident commander
  • remain with the ambulance
  • locate the safety officer
  • assess firefighters for signs of fatigue
    remain with the ambulance

Returning the emergency unit to service is part of the _.

  • termination phase
  • hazard-control phase
  • support phase
  • transfer phase
    termination phase

The EMT’s responsibility during search-and-rescue operations is to _.

  • assist in the search on foot
  • assume medical command
  • wait at the staging area until the patient is located
  • direct other incoming EMS units
    wait at the staging area until the patient is located

When removing a critically injured patient from his or her vehicle, you should:

  • remove him or her using a short backboard.
  • move him or her in one fast, continuous step.
  • release c-spine control to facilitate rapid removal.
  • protect the cervical spine during the entire process.
    protect the cervical spine during the entire process.

You are standing by at the scene of a hostage situation when the incident commander advises you that one of his personnel has been shot. The patient is lying supine in an open area and is not moving. As the SWAT team escorts you to the patient, you should:

  • perform a rapid assessment and move him to a place of safety.
  • treat only critical injuries before moving him to a safe place.
  • grab him by the clothes and immediately move him to safety.
  • limit your primary assessment to airway and breathing only.
    grab him by the clothes and immediately move him to safety.

A 33-year-old restrained driver of a motor vehicle crash is awake and alert, complaining only of neck pain and left leg pain. The vehicle is stable and no hazards are present. When removing this patient from his vehicle, you should:

  • apply a full leg splint prior to extrication.
  • use the rapid extrication technique.
  • maintain slight traction to his neck area.
  • immobilize him with a vest-style device.
    immobilize him with a vest-style device.

Extrication is defined as:

  • dismantling an automobile to remove a victim.
  • using heavy equipment to access a patient.
  • removal from a dangerous situation or position.
  • immobilizing a patient before moving him or her.
    removal from a dangerous situation or position.

Which type of drum is used for the storage of corrosives such as acids, bases, or oxidizers?

  • Cardboard
  • Polyethylene
  • Steel
  • Nickel
    Polyethylene

Which of the following is a requirement when labeling pesticide bags?

  • The EPA establishment number
  • Routes of entry
  • Permissible exposure limits
  • Contact information for the responsible party
    The EPA establishment number

In which control zone would you find the decontamination area?

  • Hot zone
  • Warm zone
  • Cold zone
  • Outer perimeter
    Warm zone

Which toxicity level includes materials that are extremely hazardous to health?

  • Level 2
  • Level 3
  • Level 1
  • Level 4
    Level 4

The two important underlying principles of the NIMS are standardization and what?

  • Maintenance
  • Resource allocation
  • Flexibility
  • Chain of command
    Flexibility

Which of the following is a member of the command staff?

  • Logistics officer
  • Planning officer
  • Public information officer
  • Operations chief
    Public information officer

You are triaging victims of a train derailment. Patient A has significant bleeding from an abdominal injury. Patient B has several simple fractures of the femur and ulna. Patient C is conscious and complaining of head and neck pain. Patient D is in cardiac arrest. How would you prioritize these patients?

  • A-yellow; B-yellow; C-green; D-red
  • A. A-red; B-yellow; C-green; D-red
  • A-red; B-green; C-yellow; D-black
  • A-yellow; B-green; C-green; D-yellow
    A-red; B-green; C-yellow; D-black

Which level of PPE do you need to wear when dealing with a Level 1 or 2 toxicity level?

  • C
  • A
  • B
  • D
    C

What is the primary difference between a disaster and an MCI?

  • MCIs are usually short term and disasters are long term.
  • EMTs can declare a disaster but not an MCI.
  • The number of casualties is higher with an MCI.
  • Casualty collection areas are only erected in MCIs.
    MCIs are usually short term and disasters are long term.

During triage, how should you tag a child who is breathing fewer than 15 breaths/min?

  • Black
  • Yellow
  • Red
  • Green
    Red

Placards and labels on a storage container are intended to:

  • broadly classify chemicals as being explosive or nonexplosive.
  • provide specific information about the chemical being carried.
  • advise responders of the appropriate neutralizing measures.
  • give a general idea of the hazard inside that particular container.
    give a general idea of the hazard inside that particular container.

Which of the following is probably NOT a multiple-casualty incident?

  • A rollover crash of a school bus with eight children who have injuries of varying severity
  • An apartment fire with one patient and the possibility of others trapped in the building
  • A motor vehicle crash with two critically injured patients and two ambulances
  • A loss of power to a hospital or nursing home with ventilator-dependent patients
    A motor vehicle crash with two critically injured patients and two ambulances

Upon arriving at the scene of a possible hazardous materials (HazMat) incident involving several patients, you should:

  • rope off the entire perimeter.
  • retrieve all critical patients.
  • divert traffic away from the scene.
  • carefully assess the situation.
    carefully assess the situation.

The term used when individual units or different organizations make independent, and often inefficient, decisions regarding an incident is called:

-single command.

  • undermining.
  • logistical chaos.
  • freelancing.
    freelancing.

Which of the following duties or responsibilities does NOT fall within the realm of the medical branch of the incident command system?

  • Transport
  • Triage
  • Treatment
  • Extrication
    Extrication

A supervisor who has more than seven people reporting to him or her:

  • is more beneficial to the overall effort than a supervisor with fewer personnel because his or her team can accomplish more tasks.
  • has exceeded an effective span of control and should divide tasks and delegate the supervision of some tasks to another person.
  • should assign a specific task to each person reporting to him or her and regularly follow up to ensure that the tasks were carried out.
  • should regularly report to the incident commander (IC) to inform him or her of the functions that his or her team is performing.
    has exceeded an effective span of control and should divide tasks and delegate the supervision of some tasks to another person.

After primary triage, the triage supervisor should communicate all of the following information to the medical branch officer, EXCEPT:

  • recommendations for movement to the treatment area.
  • the total number of patients that have been triaged.
  • the number of patients in each triage category.
  • the recommended transport destination for each patient.
    the recommended transport destination for each patient.

In preparing for a disaster, EMS systems should have enough supplies for at least a __ period of self-sufficiency.

  • 48-hour
  • 24-hour
  • 96-hour
  • 72-hour
    72-hour

The JumpSTART triage system is intended to be used for children younger than _ years or who appear to weigh less than _.

  • 6; 70 lb
  • 8; 100 lb
  • 5; 50 lb
  • 7; 90 lb
    8; 100 lb

Which of your senses can be safely used to identify a HazMat incident?

  • Sound and smell
  • Smell and sight
  • Sight and sound
  • Sight and touch
    Sight and sound

You are triaging four patients who were involved in a head-on motor vehicle crash. Which of the following patients should be assigned the highest (red) triage category?

  • A 50-year-old male with an open head injury and no pulse
  • A 49-year-old female with diabetes and difficulty breathing
  • A 36-year-old female with back pain and numb extremities
  • A 29-year-old male with bilaterally closed femur deformities
    A 49-year-old female with diabetes and difficulty breathing

According to the START triage system, what should you do if a patient is found to have a respiratory rate of 24 breaths/min?

  • Triage the patient as delayed.
  • Administer high-flow oxygen.
  • Assess his or her neurologic status.
  • Assess for bilateral radial pulses.
    Assess for bilateral radial pulses.

The development of an incident action plan is the responsibility of the:

-finance section.

  • logistics section.
  • operations section.
  • planning section.
    planning section.

You are approaching an overturned tanker truck to assess the driver, who appears to be unresponsive. As you get closer to the vehicle, you note the smell of noxious fumes and find that you are in the midst of a vapor cloud. What should you do?

  • Realize that you are in the danger zone and prevent others from entering.
  • Exit the area immediately and gather information for the HazMat team.
  • Cover your face with your shirt and quickly extricate the injured driver.
  • Remain where you are and perform a visual assessment of the patient.
    Exit the area immediately and gather information for the HazMat team.

Following proper decontamination, a 30-year-old male is brought to you. He is semiconscious and has rapid, shallow respirations. A quick visual assessment reveals no obvious bleeding. You should:

  • perform a rapid assessment to locate critical injuries.
  • ask a firefighter what the patient was exposed to.
  • begin some form of positive-pressure ventilation.
  • administer high-flow oxygen via a nonrebreathing mask.
    begin some form of positive-pressure ventilation.

Unlike a multiple-casualty incident, a natural disaster:

  • usually does not require the incident command system process.
  • is typically short-lived and does not require as much manpower.
  • often requires personnel to remain on scene for several days.
  • exists when there are more than 100 critically injured patients.
    often requires personnel to remain on scene for several days.

The function of the National Incident Management System (NIMS) is to:

  • educate city and county governments regarding foreign terrorist attacks.
  • prepare for, prevent, respond to, and recover from domestic incidents.
  • prepare for the potential of a nuclear attack against the United States.
  • facilitate a standard method of incident command for natural disasters.
    prepare for, prevent, respond to, and recover from domestic incidents.

As a triage supervisor, you:

  • must not begin treatment until all patients have been triaged.
  • should communicate with area hospitals regarding their capabilities.
  • must prepare patients for transport before they leave the triage area.
  • are responsible for providing initial treatment to all patients.
    must not begin treatment until all patients have been triaged.

While triaging patients at the scene of a building collapse, you encounter a young child who is conscious, alert, and breathing; has bilateral radial pulses; and has a severely angulated leg, which is not bleeding. According to the JumpSTART triage system, you should:

  • evacuate him to a designated area and assign him a minimal (green) category.
  • apply high-flow oxygen, obtain baseline vital signs, and continue triaging.
  • assign him a delayed (yellow) category and continue triaging the other patients.
  • quickly bind his legs together to stabilize the fracture, and continue triaging.
    assign him a delayed (yellow) category and continue triaging the other patients.

What is mustard gas considered to be?

  • G-agent
  • Mutagen
  • Metabolic agent
  • Neurotoxin
    Mutagen

In which group of agents is miosis a sign of exposure?

  • Respiratory agents
  • Nerve agents
  • Metabolic agents
  • Vesicants
    Nerve agents

Which of the following requires a host to survive?

  • Plague
  • Botulinum
  • Anthrax
  • Smallpox
    Smallpox

Which is the most deadly route of entry for anthrax?

  • Injection
  • Absorption
  • Inhalation
  • Ingestion
    Inhalation

Which of the following is the means by which an agent is spread?

  • Dissemination
  • Volatility
  • Incubation
  • Contamination
    Dissemination

Which of the following is true regarding a dirty bomb?

  • It is a highly effective WMD.
  • The destructive capability is limited to the explosives that are attached to it.
  • The most significant cause of injury is a result of the radioactive material dispersed.
  • It is difficult to make because the procurement of radioactive material is tightly regulated.
    The destructive capability is limited to the explosives that are attached to it.

Which type of blast injury is most likely to produce penetrating trauma?

  • Primary
  • Tertiary
  • Secondary
  • Quaternary
    Secondary

Why is it important to follow the requests of the incident commander?

  • The incident commander leads the fire department, not EMS.
  • It is federal law to follow the ICS structure for patient care.
  • The incident commander best understands incident needs.
  • The incident commander represents a higher level of care.
    The incident commander best understands incident needs.

You are on scene at a suspected terror attack in which a RDD has detonated. A 67-year-old patient reports tightness in his chest. The patient is coughing up blood and is in respiratory distress. During the physical assessment, you notice subcutaneous emphysema. What is the condition most likely to be?

  • Pulmonary embolism
  • Exacerbation of asthma
  • Myocardial infarction
  • Pulmonary blast injury
    Pulmonary blast injury

Petechiae are the result of what?

  • Neurologic injury
  • Psychologic injury
  • Solid organ injury
  • Hollow organ injury
    Hollow organ injury

The EMT should expect that a patient who was exposed to cyanide will have:

  • an abnormally slow pulse rate.
  • a normal pulse oximetry reading.
  • an abnormally slow respiratory rate.
  • skin that is cherry red and hot.
    a normal pulse oximetry reading.

To date, the preferred weapons of mass destruction for terrorists have been:

  • chemical weapons.
  • nuclear weapons.
  • explosive weapons.
  • biologic weapons.
    explosive weapons.

All of the following are vesicant agents, EXCEPT:

  • phosgene oxime.
  • sarin.
  • lewisite.
  • sulfur mustard.
    sarin.

In determining the potential for a terrorist attack, you should routinely observe all of the following on every call, EXCEPT:

  • victim’s statements.
  • the type of call.
  • weather conditions.
  • the location type.
    weather conditions.

Most cases of anthrax begin with:

  • signs of shock.
  • pulmonary edema.
  • respiratory distress.
  • flulike symptoms.
  • flulike symptoms.

The Centennial Park bombing during the 1996 Summer Olympics is an example of:

  • domestic terrorism.
  • an ecoterrorist attack.
  • apocalyptic violence.
  • state-sponsored terrorism.
    domestic terrorism.

Unlike viral agents, bacterial agents:

  • do not replicate in the body.
  • respond to antibiotics.
  • are far less infectious.
  • are usually not treatable.
    respond to antibiotics.

You and your partner arrive at the scene of a fire at a large office complex. Witnesses tell you that they heard a loud explosion shortly before the building caught fire. You should:

  • tell the witnesses that you suspect that the explosion was the work of a terrorist.
  • carefully document the witnesses’ statements and report them immediately.
  • take standard precautions and begin searching for critically injured patients.
  • ensure that your ambulance is parked upwind and uphill from the building.
    ensure that your ambulance is parked upwind and uphill from the building.

Continual reassessment of the scene at a suspected terrorist or weapon of mass destruction incident is MOST important because:

  • a secondary explosive device may detonate.
  • bystanders may destroy the evidence.
  • weather conditions may change quickly.
  • terrorists are often at the scene after an attack.
    a secondary explosive device may detonate.

Points of distribution (PODs) are strategically placed facilities where:

  • chemical weapons are manufactured and distributed.
  • chemical and biologic weapons are stockpiled.
  • antidotes, antibiotics, and vaccines are distributed.
  • weapons of mass destruction are distributed.
    antidotes, antibiotics, and vaccines are distributed.

At present, the likelihood of a nuclear attack against the United States is very low because:

  • the United States has an effective early warning system to detect an incoming nuclear missile.
  • terrorist nations do not have the ability to deliver a nuclear weapon via missile or bomb.
  • other than the United States, no other countries are currently in possession of nuclear weapons.
  • all nuclear devices or weapons that different countries are in possession of are currently accounted for.
    terrorist nations do not have the ability to deliver a nuclear weapon via missile or bomb.

As you and your partner report for duty, you check your ambulance and begin talking about the possibility of a terrorist attack. The MOST effective and appropriate way to determine the likelihood of this happening is to:

  • check with local businesses to see if they have received any terrorist threats.
  • ask your immediate supervisor if he or she has been watching the local news.
  • know the current threat level issued by the Department of Homeland Security.
  • ascertain the current situation overseas with regard to the number of casualties.
    know the current threat level issued by the Department of Homeland Security.

After eating at a local restaurant, a 20-year-old male complains of blurred vision, difficulty speaking, and difficulty breathing. He is conscious; however, his respirations are profoundly labored and producing minimal tidal volume. You should:

  • assist his ventilations with high-flow oxygen.
  • apply oxygen via a nonrebreathing mask.
  • position him supine and elevate his legs.
  • request a paramedic to administer atropine.
    assist his ventilations with high-flow oxygen.

The incubation period for Ebola can be up to:

  • 21 days.
  • 6 months.
  • 72 hours.
  • 1 week.
    21 days.

An attack on an abortion clinic would MOST likely be carried out by a(n):

  • violent religious group.
  • extremist political group.
  • single-issue group.
  • doomsday cult.
    single-issue group.

The type and severity of wounds sustained from incendiary and explosive devices primarily depend on the:

  • type of material used to manufacture the device.
  • size of the structure that was involved in the explosion.
  • patient’s distance from the epicenter of the explosion.
  • pressure that is generated from the explosion itself.
    patient’s distance from the epicenter of the explosion.

__ rays easily penetrate through the human body and require lead or several inches of concrete to prevent penetration.

  • Beta
  • Neutron
  • Gamma
  • Alpha
    Gamma

The means by which a terrorist will spread a particular agent is called:

  • weaponization.
  • aerosolization.
  • incubation.
  • dissemination.
    dissemination.

Which of the following statements regarding blast injuries is correct?

  • Solid organs such as the middle ear, lungs, and gastrointestinal tract are the most susceptible to pressure changes.
  • Solid organs are relatively protected from shock wave injury but may be injured during the secondary or tertiary blast phase.
  • Tertiary blast injuries are penetrating or nonpenetrating injuries that result from flying debris, such as ordnance projectiles.
  • The gastrointestinal tract is the organ system most sensitive to blast injuries and is the leading cause of death following an explosion.
    Solid organs are relatively protected from shock wave injury but may be injured during the secondary or tertiary blast phase.

Multiple people in a small town began experiencing abdominal cramps, excessive salivation and urination, and muscle twitching shortly after a small crop duster plane made several passes over the community. As you are assessing the patients, you further determine that most of them are bradycardic and have miosis. In addition to high-flow oxygen, the MOST appropriate treatment for these patients includes:

  • atropine and pralidoxime chloride.
  • epinephrine and hyperbaric oxygen.
  • activated charcoal and glucose.
  • amyl nitrate and naloxone.
    atropine and pralidoxime chloride.

What is the best group dynamic for an EMS situation?

  • Dependent
  • Independent
  • Interdependent
  • Co-dependent
    Interdependent

Which of the following is an example of closed loop communication?

  • Speaking concisely
  • Telling a team member when he/she is doing something wrong
  • Asking “yes” or “no” questions
  • Repeating a message back to the speaker
    Repeating a message back to the speaker

What is the purpose of crew resource management (CRM)?

  • It provides clear guidelines for providing support to ALS providers in the field.
  • It empowers team members to speak up when they detect a potential problem.
  • It clearly defines the roles and responsibilities of each team member prior to engaging in a potentially hazardous event.
  • It ensures that the team leader has the ultimate decision-making power in an emergency situation.
    It empowers team members to speak up when they detect a potential problem.

What is the best way to ensure the effectiveness of a team?

  • Carefully select team members who can collaborate well with others.
  • Ensure that the team leader can deliver direct and timely feedback to team members.
  • Make sure each team member is knowledgeable about how to do critical procedures.
  • Train and practice together.
    Train and practice together.

What is the first step in performing an endotracheal intubation?

  • Determine if trauma is present.
  • Position the patient.
  • Preoxygenate with a BVM.
  • Evaluate the airway.
    Preoxygenate with a BVM.

What is the best position for a patient who needs endotracheal intubation?

  • Prone
  • Sniffing
  • Semi-Fowler
  • Supine
    Sniffing

Which technique should you use when helping ALS spike an IV bag?

  • Pure
  • Clean
  • Aseptic
  • Sterile
    Aseptic

Which mnemonic is used in CRM?

  • PACE (Probe, Alert, Challenge, Emergency)
  • PUSH (Placate, Urge, Suggest, Help)
  • PRONE (Problem, Report, Object, Negotiate, Evaluate)
  • PIE (Plan, Interventions, Evaluation)
    PACE (Probe, Alert, Challenge, Emergency)

You are dispatched to a private residence where the patient is unresponsive, not breathing, and has a weak pulse. You secure her airway with an oral airway and ventilate her with a BVM at a rate of one breath every 5 seconds. An ALS transport ambulance arrives. The paramedic and her AEMT partner enter the house and approach you and the patient. Now that the ambulance has arrived, who will be the team leader?

  • The AEMT, because the paramedic will be too busy intubating the patient
  • The hospital physician, because he provides medical control
  • The paramedic, because she is the highest-level provider on the scene
  • You, because you were the first provider on the scene
    The paramedic, because she is the highest-level provider on the scene

You are assisting with an endotracheal intubation. After the tube is placed, you notice increased resistance as you ventilate with a BVM. What does this indicate?

  • Vascular collapse
  • Esophageal intubation
  • The ET tube is not secured at the proper depth marking.
  • Apneic oxygenation
    Esophageal intubation

Hyperventilation during the preoxygenation phase of endotracheal intubation:

  • is acceptable if done for fewer than 2 minutes.
  • can cause gastric distention and hypotension.
  • will decrease the likelihood of aspiration.
  • provides a better oxygen reserve for the patient.
    can cause gastric distention and hypotension.

You are attending to a 46-year-old male patient complaining of chest pain. Shortly after you begin to render care, an ALS unit arrives and the transfer of care is made. The decision is made to start an intravenous line and administer medication. You should:

  • acknowledge that it is no longer your patient and stand back, allowing the ALS provider to work.
  • move out of the ALS provider’s way and clear the scene.
  • clear space and, if necessary, explain the procedure to the patient.
  • ensure that enough assessments have been done to justify the decision.
    clear space and, if necessary, explain the procedure to the patient.

A team of EMTs is caring for a critically injured patient. The team leader advises the EMT that transport will not begin until the patient’s closed forearm fracture is splinted. Utilizing the crew resource management model, the EMT should:

  • repeat the request back to the team leader and then splint the patient’s arm.
  • advise the team leader that immediate transport is more important than splinting.
  • ensure that the entire team is aware that transport will be delayed for splinting.
  • disregard the team leader’s request and contact medical control for guidance.
    advise the team leader that immediate transport is more important than splinting.

When the EMT assists a paramedic with an advanced intervention, he or she should recall that the focus of the intervention is on:

  • solving a clinical problem.
  • completing the procedure.
  • following local protocol.
  • learning to perform the skill.
    solving a clinical problem.

When working as an independent health care group member, the EMT should expect that he or she:

  • will receive no support or guidance from an EMS supervisor.
  • will be specifically instructed on how to perform a specific task.
  • does not have to wait for an assignment before performing a task.
  • will rely on the group leader for making virtually all decisions.
    does not have to wait for an assignment before performing a task.

You are attending to a 68-year-old female patient in cardiac arrest. An ALS provider arrives shortly after you do and the transfer of care is made. The ALS provider asks that you assist in the endotracheal intubation. As part of this process, you may be required to:

  • visualize the airway and look for any potential complications in advance of the intubation.
  • perform the intubation with assistance.
  • help position the patient for a better view of the airway during the procedure.
  • ventilate and preoxygenate the patient but not handle any of the equipment required for the intubation.
    help position the patient for a better view of the airway during the procedure.

You are organizing a group of EMTs to provide triage in a mass-casualty exercise. In order for the group to be successful, it is essential that:

  • individuals understand that their roles will all be accomplished the same way.
  • individuals have an understanding of how the group will accomplish its goals.
  • individual goals are accomplished.
  • individuals understand that this is a one-time exercise and that things will change in the field.
    individuals have an understanding of how the group will accomplish its goals.

The effectiveness of pit crew CPR is dependent on:

  • a team leader who is capable of performing all of the patient care tasks.
  • defining clear roles and responsibilities before the call is received.
  • rapidly assessing the patient before assigning roles and responsibilities.
  • protocols that allow the EMT to function without medical control.
    defining clear roles and responsibilities before the call is received.

After spiking a bag of IV fluid for the paramedic, the EMT notices that the drip chamber is too full. The EMT should:

  • invert the IV bag and squeeze the drip chamber.
  • replace the administration set with a new one.
  • squeeze the IV bag to force fluid into the tubing.
  • let the IV flow rapidly for 20 to 30 seconds.
    invert the IV bag and squeeze the drip chamber.

You are at a mass-casualty scene and in the process of organizing a group to be responsible for ongoing patient care. In order for the group to be successful:

  • individuals see this as a one-time exercise and that the group may change.
  • individuals have an understanding of how the group will accomplish its goals.
  • individual goals are accomplished.
  • all individuals have the same roles.
    individuals have an understanding of how the group will accomplish its goals.

You are attending to a patient who was experiencing abdominal pain at a local shopping mall. The patient sitting on a bench and appears in moderate distress. You are met by a first responder at the scene who wishes to provide you with a patient care report. Your best course of action is to:

  • listen to the report while you perform a primary survey and render care to the patient.
  • listen to the responder while your partner performs a primary survey.
  • move the responder out of the way and begin patient care.
  • ask the responder to wait and take their report after you have completed a primary survey.
    listen to the responder while your partner performs a primary survey.

Which of the following would indicate that the endotracheal tube is NOT in the trachea?

  • Absence of sounds over the epigastrium
  • Decreased resistance when ventilating
  • Steady increase in the oxygen saturation
  • Absence of an end-tidal CO2 waveform
    Absence of an end-tidal CO2 waveform

The Microdrip administration delivers 1 mL of fluid for every __ drops.

  • 45
  • 60
  • 10
  • 15
    60

If a problem with a team member is not directly or immediately impacting patient care, the team leader should:

  • ignore the problem to avoid conflict.
  • engage the team member at once.
  • discuss the problem after the call.
  • contact the medical director at once.
    discuss the problem after the call.

You are called to the scene of a 56-year-old female patient who was the driver of a car that struck a telephone pole. The patient is sitting in her vehicle with her seat belt on and the air bag has deployed. A bystander approaches as you arrive and informs you that he is an off-duty firefighter and was first on the scene. The bystander wishes to provide you with a patient care report. You should:

  • ask the bystander for any formal identification and listen to the report once it is verified.
  • ask your partner to perform the primary survey while you listen to the report.
  • ask the bystander to wait and you will take their report after you have finished your assessment.
  • listen to the report while you perform a primary survey and render care to the patient.
    ask your partner to perform the primary survey while you listen to the report.

You are attending to a 66-year-old male patient in cardiac arrest. As you gather history and begin the resuscitation, an Advanced Life Support (ALS) provider arrives on scene. You should:

  • stay focused on the resuscitation and allow the ALS provider to make his or her own assessment.
  • continue the resuscitation and provide a patient care report while you work.
  • pause the resuscitation and provide a patient care report as quickly as possible.
  • ensure that CPR is ongoing by the other providers present and then provide a patient care report off to the side.
    ensure that CPR is ongoing by the other providers present and then provide a patient care report off to the side.

The concept of consistent care across the entire health care team from first patient contact to patient discharge is called:

  • the continuum of care.
  • patient care advocacy.
  • the standard of care.
  • the scope of practice.
    the continuum of care.

Health care teams that infrequently train and work together:

  • can create delays in patient care.
  • are unable to accomplish their tasks.
  • often work better under pressure.
  • need less-explicit verbal direction.
    can create delays in patient care.

EMTs arrive at the scene of an ill person. The EMR, who arrived before the EMTs, advises that the patient had a syncopal episode. The patient is conscious and alert and remains so throughout transport. When transferring patient care to the emergency department nurse, the EMT should advise the nurse that:

  • she should contact the EMR about the incident.
  • the patient had a reported syncopal episode.
  • the EMR was probably mistaken about the episode.
  • there is no evidence to support the syncopal episode.
    the patient had a reported syncopal episode.

Which of the following would the EMT MOST likely be asked to do when assisting a paramedic with endotracheal intubation?

  • Visualization of the vocal cords
  • Placement of the endotracheal tube
  • Preoxygenation with a BVM
  • Suction under direct laryngoscopy
    Preoxygenation with a BVM

Leave a Comment

Scroll to Top